Acute - Module 7

Ace your homework & exams now with Quizwiz!

_____: A general term for a loss of sensory comprehension; may include an inability to write, comprehend reading material, or use an object correctly.

Agnosia

_____: Inability to find words.

Anomia

A client has just returned from cerebral angiography. Which symptom does the client display that causes the nurse to act immediately? A) Bleeding. B) Increased temperature. C) Severe headache. D) Urge to void.

Answer: A. After a cerebral angiography, the nurse would immediately react if the client had any bleeding. If bleeding is present at the puncture site, manual pressure on the site is maintained along with immediate notification of the primary care provider.Increased temperature or the urge to void are not typical complications of cerebral angiography. Severe headache is a typical complication of a lumbar puncture, but not of cerebral angiography.

_____: Overmedication with cholinesterase inhibitors.

Cholinergic crisis

_____: Surgical incision into the cranium.

Craniotomy

_____: Specific areas of the skin that receive sensory input from spinal nerves.

Dermatomes

_____: Slurred speech.

Dysarthria

_____: Removal of a blood clot.

Embolectomy

_____: Weakness on one side of the body.

Hemiparesis

_____: Abnormal sensitivity to sound.

Phonophobia

_____: Drooping of the eyelid.

Ptosis

The client comes to the clinic and reports a sudden drooping of the left side of the face and complains of pain in that area. The nurse notes that the client cannot wrinkle the forehead or close the left eye. Which condition should the nurse suspect? 1. Bell's palsy. 2. Right-sided stroke. 3. Tetany. 4. Mononeuropathy

ANSWER: 1. 1. Bell's palsy, called facial paralysis, is a disorder of the 7th cranial nerve (facial nerve) characterized by unilateral paralysis of facial muscles. 2. These are symptoms of a left-sided stroke. 3. Tetany is due to low calcium levels. In this disorder, the face twitches when touched; this is known as a positive Chvostek's sign. 4. Mononeuropathy is limited to a single peripheral nerve and its branches and occurs because the trunk of the nerve is compressed, such as in carpal tunnel syndrome.

The nurse is caring for a client diagnosed with encephalitis. Which is an expected outcome for the client? 1. The client will regain as much neurological function as possible. 2. The client will have no short-term memory loss. 3. The client will have improved renal function. 4. The client will apply hydrocortisone cream daily.

ANSWER: 1. 1. Clients diagnosed with encephalitis have neurological deficits while the inflammation is present. The therapeutic plan is to treat the disease process, decrease the edema, and return the client to an optimal level of wellness. 2. The client may have short-term memory loss from a previous condition. 3. Renal function is not affected by encephalitis. Only immobility would affect this system. 4. There is no reason to apply hydrocortisone cream for encephalitis. TEST TAKING HINT: The test taker should look at the option that reflects the body system that is involved with the disease. Refer to medical terminology; encephalon means "the brain."

The nurse is developing a plan of care for a client diagnosed with West Nile virus. Which intervention should the nurse include in this plan? 1. Monitor the client's respirations frequently. 2. Refer to a dermatologist for treatment of maculopapular rash. 3. Treat hypothermia by using ice packs under the client's arms. 4. Teach the client to report any swollen lymph glands.

ANSWER: 1. 1. Clients with West Nile virus should be continuously assessed for alteration in gas exchanges or patterns. 2. A rash will resolve when the disease causing the rash is treated. 3. Hypothermia is not treated with ice packs but with warming blankets. 4. Lymph glands are edematous early in the disease process. There is no reason to teach the client to report this condition. TEST TAKING HINT: The test taker needs to read words carefully. Prefixes such as "hypo-" and "hyper-" are important in determining if an option is correct. Even if the test taker did not know if the client is hypothermic or hyperthermic, "hypo-" means "less than normal" so hypothermia would not be treated with ice packs. A client with West Nile virus usually has a fever that should be reduced. Thus, a treatment for hypothermia is not needed.

The client is in the terminal stage of ALS. Which intervention should the nurse implement? 1. Perform passive ROM every two (2) hours. 2. Maintain a negative nitrogen balance. 3. Encourage a low-protein, soft-mechanical diet. 4. Turn the client and have him cough and deep breathe every shift.

ANSWER: 1. 1. Contractures can develop within a week because extensor muscles are weaker than flexor muscles. If the client cannot perform ROM exercises, then the nurse must do it for him—passive ROM. 2. The client should maintain a positive nitrogen balance to promote optimal body functioning. 3. Adequate protein is required to maintain osmotic pressure and prevent edema. 4. The client is usually on bedrest in the last stages and should be turned and told to cough and deep breathe more often than every shift. TEST TAKING HINT: "Terminal stage" is the key term in the stem that should cause the test taker to look for an option addressing immobility issues—option "1." An intervention implemented only once in every shift should be eliminated as a possible answer when addressing immobility issues.

The client is diagnosed with ALS. Which client problem would be most appropriate for this client? 1. Disuse syndrome. 2. Altered body image. 3. Fluid and electrolyte imbalance. 4. Alteration in pain.

ANSWER: 1. 1. Disuse syndrome is associated with complications of bedrest. Clients with ALS cannot move and reposition themselves, and they frequently have altered nutritional and hydration status. 2. The client does not usually have a change in body image. 3. ALS is a disease affecting the muscles, not the kidneys or circulatory system. 4. ALS is not painful. TEST TAKING HINT: The test taker would have to be knowledgeable about ALS to answer this question. This disease is chronic and debilitating over time and leads to wasting of the muscles

The client is scheduled for an MRI of the brain to confirm a diagnosis of Creutzfeldt-Jakob disease. Which intervention should the nurse implement prior to the procedure? 1. Determine if the client has claustrophobia. 2. Obtain a signed informed consent form. 3. Determine if the client is allergic to egg yolks. 4. Start an intravenous line in both hands.

ANSWER: 1. 1. For an MRI scan, the client is placed in a very narrow tube. If the client is claustrophobic, he or she may need medication or an open MRI machine may need to be considered. 2. An MRI scan is not an invasive procedure; therefore, informed consent is not needed. 3. The nurse would need to determine allergies to shellfish or iodine, not to egg yolks. 4. The client will need one saline lock, not two intravenous lines. Often the MRI tech is the person who inserts the IV line.

The charge nurse is making assignments. Which client should be assigned to the new graduate nurse? 1. The client diagnosed with aseptic meningitis who is complaining of a headache and the light bothering his eyes. 2. The client diagnosed with Parkinson's disease who fell during the night and is complaining of difficulty walking. 3. The client diagnosed with a cerebrovascular accident whose vitals signs are P 60, R 14, and BP 198/68. 4. The client diagnosed with a brain tumor who has a new complaint of seeing spots before the eyes.

ANSWER: 1. 1. Headache and photophobia are expected clinical manifestations of meningitis. The new graduate could care for this client. 2. This client has had an unusual occurrence (fall) and now has a potential complication (a fracture). The experienced nurse should take care of this client. 3. These vital signs indicate increased intracranial pressure. The more experienced nurse should care for this client. 4. This could indicate a worsening of the tumor. This client is at risk for seizures and herniation of the brainstem. The more experienced nurse should care for this client. TEST TAKING HINT: The test taker should determine if the clinical manifestations are expected as part of the disease process. If they are, a new graduate can care for the client; if they are not expected occurrences, a more experienced nurse should care for the client.

The nurse is assessing the client diagnosed with meningococcal meningitis. Which assessment data would warrant notifying the HCP? 1. Purpuric lesions on the face. 2. Complaints of light hurting the eyes. 3. Dull, aching, frontal headache. 4. Not remembering the day of the week.

ANSWER: 1. 1. In clients with meningococcal meningitis, purpuric lesions over the face and extremity are the signs of a fulminating infection that can lead to death within a few hours. 2. Photophobia is a common clinical manifestation of all types of meningitis and would be expected. 3. Inflammation of the meninges results in increased intracranial pressure, which causes a headache. This would be an expected occurrence and would not warrant notifying the HCP. 4. A client not being able to identify the day of the week would not in itself warrant notifying the HCP. TEST TAKING HINT: The stem is asking the nurse to identify which assessment data are abnormal for the disease process and require an immediate medical intervention to prevent the client from experiencing a complication or possible death

The public health department nurse is preparing a lecture on prevention of West Nile virus. Which information should the nurse include? 1. Change water daily in pet dishes and birdbaths. 2. Wear thick, dark clothing when outside to avoid bites. 3. Apply insect repellent over face and arms only. 4. Explain that mosquitoes are more prevalent in the morning.

ANSWER: 1. 1. Mosquitoes breed in standing water, even pet dishes and birdbaths. All areas that collect water should be emptied, removed, covered, or turned over. Rain gutters should be cleaned. 2. Light-colored, long-sleeved, and loosefitting clothing should be worn to avoid mosquito bites. 3. Insect repellent may irritate the eyes, but it should be applied over clothing and on all exposed areas. 4. Mosquitoes are more prevalent at dusk, dawn, and early evening. TEST TAKING HINT: Words such as "only" (option "3") should clue the test taker to eliminate that option. Rarely are these absolute terms correct.

In assessing a client with a Thoracic SCI, which clinical manifestation would the nurse expect to find to support the diagnosis of neurogenic shock? 1. No reflex activity below the waist. 2. Inability to move upper extremities. 3. Complaints of a pounding headache. 4. Hypotension and bradycardia.

ANSWER: 1. 1. Neurogenic shock associated with SCI represents a sudden depression of reflex activity below the level of the injury. T12 is just above the waist; therefore, no reflex activity below the waist would be expected. 2. Assessment of the movement of the upper extremities would be more appropriate with a higher level injury; an injury in the cervical area might cause an inability to move the upper extremities. 3. Complaints of a pounding headache are not typical of a T12 spinal injury. 4. Hypotension (low blood pressure) and tachycardia (rapid heart rate) are signs of hypovolemic or septic shock, but these do not occur in spinal shock. TEST TAKING HINT: If the test taker does not have any idea what the answer is, an attempt to relate the anatomical position of keywords in the question stem to words in the answer options is appropriate. In this case, T12, mentioned in the stem, is around the waist, so answer options involving the anatomy above that level (e.g., the upper extremities) can be eliminated.

The chief executive officer (CEO) of a large manufacturing plant presents to the occupational health clinic with chronic rhinitis and requesting medication. On inspection, the nurse notices holes in the septum that separates the nasal passages. The nurse also notes dilated pupils and tachycardia. The facility has a "No Drug" policy. Which intervention should the nurse implement? 1. Prepare to complete a drug screen urine test. 2. Discuss the client's use of illegal drugs. 3. Notify the client's supervisor about the situation. 4. Give the client an antihistamine and say nothing.

ANSWER: 1. 1. No employee of a facility is above certain rules. In a company with a "No Drugs" policy, this includes the CEO. This client is exhibiting symptoms of cocaine abuse. 2. The nurse does not have a definitive knowledge that the client is using drugs until a positive drug screen result is obtained. If the nurse is not a trained substance abuse counselor, this intervention would be out of the realm of the nurse's expertise. 3. The client is the CEO of the facility; only the board of directors or parent company is above this client in supervisory rank. 4. Giving an antihistamine is prescribing without a license, and the nurse is obligated to intervene in this situation. TEST TAKING HINT: The title of the client— CEO—eliminates option "3." The nurse has noted a potential illegal situation

The nurse enters the room as the client is beginning to have a tonic-clonic seizure. What action should the nurse implement first? 1. Note the first thing the client does in the seizure. 2. Assess the size of the client's pupils. 3. Determine if the client is incontinent of urine or stool. 4. Provide the client with privacy during the seizure.

ANSWER: 1. 1. Noticing the first thing the client does during a seizure provides information and clues as to the location of the seizure in the brain. It is important to document whether the beginning of the seizure was observed. 2. Assessment is important, but during the seizure the nurse should not attempt to restrain the head to assess the eyes; muscle contractions are strong, and restraining the client could cause injury. 3. This should be done, but it is not the first intervention when walking into a room where the client is beginning to have a seizure. 4. The client should be protected from onlookers, but the nurse should always address the client first. TEST TAKING HINT: This is a prioritizing question that asks the test taker which intervention to implement first. All four interventions would be appropriate, but only one should be implemented first. If the test taker cannot decide between two choices, always select the one that directly affects the client or the condition; privacy is important, but helping determine the origin of the seizure is priority.

The client diagnosed with a gunshot wound to the head assumes decorticate posturing when the nurse applies painful stimuli. Which assessment data obtained three (3) hours later would indicate the client is improving? 1. Purposeless movement in response to painful stimuli. 2. Flaccid paralysis in all four extremities. 3. Decerebrate posturing when painful stimuli are applied. 4. Pupils that are 6 mm in size and nonreactive on painful stimuli.

ANSWER: 1. 1. Purposeless movement indicates that the client's cerebral edema is decreasing. The best motor response is purposeful movement, but purposeless movement indicates an improvement over decorticate movement, which, in turn, is an improvement over decerebrate movement or flaccidity. 2. Flaccidity would indicate a worsening of the client's condition. 3. Decerebrate posturing would indicate a worsening of the client's condition. 4. The eyes respond to light, not painful stimuli, but a 6-mm nonreactive pupil indicates severe neurological deficit. TEST TAKING HINT: The test taker must have strong assessment skills and know what specific signs/symptoms signify for each of the body systems—in this case, the significance of different stages of posturing/ movement in assessing neurological status.

The nurse identifies the concept of intracranial regulation disturbance in a client diagnosed with Parkinson's Disease. Which priority intervention should the nurse implement? 1. Keep the bed low and call light in reach. 2. Provide a regular diet of three (3) meals per day. 3. Obtain an order for home health to see the client. 4. Perform the Braden scale skin assessment.

ANSWER: 1. 1. Safety is always a priority intervention when working with a client whose physical functioning is impaired or when the client's cognitive judgment is compromised. 2. The client should receive six (6) small meals each day. The client's swallowing ability may be impaired and the client will be unable to consume the meal before it gets cold. The consistency should be soft to not require extended chewing. 3. Home health may be needed but the priority intervention is safety. 4. A skin assessment is not priority over keeping the patient safe. TEST TAKING HINT: The test taker should remember that basic nursing care is appropriate for client protection. Maslow's hierarchy of needs lists safety in the second highest priority tier. Physiological needs that involve life-threatening or life-altering complications are the only things that are more important than safety.

The client with a history of migraine headaches comes to the emergency department complaining of a migraine headache. Which collaborative treatment should the nurse anticipate? 1. Administer an injection of sumatriptan (Imitrex), a triptan. 2. Prepare for a computed tomography (CT) of the head. 3. Place the client in a quiet room with the lights off. 4. Administer propranolol (Inderal), a beta blocker.

ANSWER: 1. 1. Sumatriptan is a medication of choice for migraine headaches. It constricts blood vessels and reduces inflammation. The nurse administering the medication is part of a collaborative effort because the nurse must act on the order or prescription of a physician or other health-care provider who has prescriptive authority. 2. This is a collaborative intervention, but it is not routinely ordered because the client reports having a history of migraine headaches. 3. This is an appropriate independent nursing intervention. 4. Propranolol is not used for acute migraine headaches; it is prescribed for long-term prophylaxis of migraines, so the nurse should not anticipate its use in this situation.

The nurse caring for a client who has been abusing amphetamines writes a problem of "cardiovascular compromise." Which nursing interventions should be implemented? 1. Monitor the telemetry and vital signs every four (4) hours. 2. Encourage the client to verbalize the reason for using drugs. 3. Provide a quiet, calm atmosphere for the client to rest. 4. Place the client on bedrest and a low sodium diet.

ANSWER: 1. 1. Telemetry and vital signs would be done to monitor cardiovascular compromise. Amphetamine use causes tachycardia, vasoconstriction, hypertension, and arrhythmias. 2. This might be an intervention for a problem of altered coping. 3. This would be an intervention for a problem of insomnia. 4. These are interventions for heart failure. TEST TAKING HINT: The correct answer must address the problem of cardiovascular compromise, which eliminates options "2" and "3."

The nurse is working with several clients in a substance abuse clinic. Client A tells the nurse that another client, Client B, has "started using again." Which action should the nurse implement? 1. Tell Client A the nurse cannot discuss Client B with him. 2. Find out how Client A got this information. 3. Inform the HCP that Client B is using again. 4. Get in touch with Client B and have the client come to the clinic

ANSWER: 1. 1. The Health Insurance Portability and Accountability Act (HIPAA) requires that a health-care professional not divulge information about one person to an unauthorized person. 2. This would be discussing Client B and a violation of HIPAA. 3. The nurse does not know Client B is using drugs, so notifying the HCP is not appropriate. 4. Client B would require an explanation for coming to the clinic, for which, if the nurse has not violated HIPAA, there is no explanation. TEST TAKING HINT: Nurses are required to practice within the laws of the state and within federal laws. HIPAA is a federal law and applies to all health-care professionals in the United States. Legally the nurse cannot use the information provided by Client A, but morally the nurse might try to identify behavior in Client B that would warrant the nurse's intervention

The client diagnosed with septic meningitis is admitted to the medical floor at noon. Which health-care provider's order would have the highest priority? 1. Administer an intravenous antibiotic. 2. Obtain the client's lunch tray. 3. Provide a quiet, calm, and dark room. 4. Weigh the client in hospital attire.

ANSWER: 1. 1. The antibiotic has the highest priority because failure to treat a bacterial infection can result in shock, systemic sepsis, and death. 2. The lunch tray is important and may actually arrive prior to the antibiotic, but the priority for the nurse must be the medication. 3. The client's room should be kept dark because of photophobia, but photophobia is a symptom that is not life threatening. 4. Knowledge of the client's weight is necessary, but initial antibiotic therapy can be initiated without knowing the client's admission weight. TEST TAKING HINT: The nurse must know how to prioritize care. Which intervention has the potential to avoid a complication related to the disease process? Remember the word "priority."

A client diagnosed with a subarachnoid hemorrhage has undergone a craniotomy for repair of a ruptured aneurysm. Which intervention will the intensive care nurse implement? 1. Administer a stool softener bid. 2. Encourage the client to cough hourly. 3. Monitor neurological status every shift. 4. Maintain the dopamine drip to keep BP at 160/90.

ANSWER: 1. 1. The client is at risk for increased intracranial pressure whenever performing the Valsalva maneuver, which will occur when straining during defecation. Therefore, stool softeners would be appropriate. 2. Coughing increases intracranial pressure and is discouraged for any client who has had a craniotomy. The client is encouraged to turn and breathe deeply but not to cough. 3. Monitoring the neurological status is appropriate for this client, but it should be done much more frequently than every shift. 4. Dopamine is used to increase blood pressure or to maintain renal perfusion, and a BP of 160/90 is too high for this client. TEST TAKING HINT: The test taker should always notice if an answer option has a time frame—every shift, every four (4) hours, or daily. Whether or not the time frame is correct may lead the test taker to the correct answer.

The client is diagnosed with a closed head injury and is in a coma. The nurse writes the client problem as "high risk for immobility complications." Which intervention would be included in the plan of care? 1. Position the client with the head of the bed elevated at intervals. 2. Perform active range-of-motion (ROM) exercises every four (4) hours. 3. Turn the client every shift and massage bony prominences. 4. Explain all procedures to the client before performing them.

ANSWER: 1. 1. The head of the client's bed should be elevated to help the lungs expand and prevent stasis of secretions that could lead to pneumonia, a complication of immobility. 2. Active range-of-motion exercises require that the client participate in the activity. This is not possible because the client is in a coma. 3. The client is at risk for pressure ulcers and should be turned more frequently than every shift, and research now shows that massaging bony prominences can increase the risk for tissue breakdown. 4. The nurse should always talk to the client, even if he or she is in a coma, but this will not address the problem of immobility. TEST TAKING HINT: Whenever a client problem is written, interventions must address the specific problem, not the disease. Positioning the client addresses the possibility of immobility complications, whereas talking to a comatose client addresses communication deficit and psychosocial needs, not immobility issues

The unlicensed assistive personnel (UAP) is caring for a client who is having a seizure. Which action by the UAP would warrant immediate intervention by the nurse? 1. The assistant attempts to insert an oral airway. 2. The assistant turns the client on the right side. 3. The assistant has all the side rails padded and up. 4. The assistant does not leave the client's bedside.

ANSWER: 1. 1. The nurse must intervene to stop the UAP because the client's jaws are clenched. Attempting to insert anything into the mouth could cause injury to the client or to the UAP. 2. Side-lying positions help to prevent aspiration and are an appropriate intervention. 3. The client's safety is priority, and this will help protect the client from injury. 4. Staying with the client is an appropriate behavior that would not warrant intervention by the nurse.

The nurse and the unlicensed assistive personnel (UAP) are caring for clients on a medicalsurgical unit. Which task should not be assigned to the UAP? 1. Feed the 69-year-old client diagnosed with Parkinson's disease who is having difficulty swallowing. 2. Turn and position the 89-year-old client diagnosed with a pressure ulcer secondary to Parkinson's disease. 3. Assist the 54-year-old client diagnosed with Parkinson's disease with toilet-training activities. 4. Obtain vital signs on a 72-year-old client diagnosed with pneumonia secondary to Parkinson's disease

ANSWER: 1. 1. The nurse should not delegate feeding a client who is at risk for complications during feeding. This requires judgment that the UAP is not expected to possess. 2. Unlicensed assistive personnel can turn and position clients with pressure ulcers. The nurse should assist in this at least once during the shift to assess the wound area. 3. The UAP can assist the client to the bathroom every two (2) hours and document the results of the attempt. 4. The UAP can obtain the vital signs on a stable client. TEST TAKING HINT: When reading the answer options in a question in which the nurse is delegating to an unlicensed-assistive personnel, read the stem carefully. Is the question asking what to delegate or what not to delegate? Anything requiring professional judgment should not be delegated.

The client diagnosed with atrial fibrillation has experienced a transient ischemic attack (TIA). Which medication would the nurse anticipate being ordered for the client on discharge? 1. An oral anticoagulant medication. 2. A beta blocker medication. 3. An anti-hyperuricemic medication. 4. A thrombolytic medication.

ANSWER: 1. 1. The nurse would anticipate an oral anticoagulant, warfarin (Coumadin), to be prescribed to help prevent thrombi formation in the atria secondary to atrial fibrillation. The thrombi can become embolic and may cause a TIA or CVA (stroke). 2. Beta blockers slow the heart rate and decrease blood pressure but would not be an anticipated medication to help prevent a TIA secondary to atrial fibrillation. 3. An anti-hyperuricemic medication is administered for a client experiencing gout and decreases the formation of tophi. 4. A thrombolytic medication is administered to dissolve a clot, and it may be ordered during the initial presentation for a client with a CVA but not on discharge. TEST TAKING HINT: In the stem of this question, there are two disease processes mentioned—atrial fibrillation and TIA. The reader must determine how one process affects the other before answering the question. In this question, the test taker must know atrial fibrillation predisposes the client to the formation of thrombi, and, therefore, the nurse should anticipate the health-care provider ordering a medication to prevent clot formation, an anticoagulant.

Which priority goal would the nurse identify for a client diagnosed with Parkinson's Disease (PD)? 1. The client will be able to maintain mobility and swallow without aspiration. 2. The client will verbalize feelings about the diagnosis of Parkinson's Disease. 3. The client will understand the purpose of medications administered for PD. 4. The client will have a home health agency for monitoring at home.

ANSWER: 1. 1. The priority goal is for the client to maintain functional ability. This improves quality and quantity of life. 2. Verbalizing feelings is a good goal but feeling will not impact stabilizing the physiological deterioration of the client. 3. There is no way to measure the client's understanding. 4. Having a home health agency does not ensure that functional ability is maintained. TEST TAKING HINT: Using Maslow's hierarchy of needs, physiological needs are higher than psychosocial needs, so the test taker can eliminate option "2." The nurse cannot determine or measure "understanding," so option "3" can be eliminated.

The home health nurse is caring for a 28-yearold client with a T10 SCI who says, "I can't do anything. Why am I so worthless?" Which statement by the nurse would be the most therapeutic? 1. "This must be very hard for you. You're feeling worthless?" 2. "You shouldn't feel worthless—you are still alive." 3. "Why do you feel worthless? You still have the use of your arms." 4. "If you attended a work rehab program you wouldn't feel worthless."

ANSWER: 1. 1. Therapeutic communication addresses the client's feelings and attempts to allow the client to verbalize feelings; the nurse should be a therapeutic listener. 2. This is belittling the client's feelings. 3. The client does not owe the nurse an explanation of his feelings; "why" is never therapeutic. 4. This is advising the client and is not therapeutic. TEST TAKING HINT: When the question requests a therapeutic response, the test taker should select the answer option that has "feelings" in the response.

The client diagnosed with delirium tremens when trying to quit drinking cold turkey is admitted to the medical unit. Which medications would the nurse anticipate administering? 1. Thiamine (vitamin B6) and librium, a benzodiazepine. 2. Dilantin, an anticonvulsant, and Feosol, an iron preparation. 3. Methadone, a synthetic narcotic, and Depakote, a mood stabilizer. 4. Mannitol, an osmotic diuretic, and Ritalin, a stimulant.

ANSWER: 1. 1. Thiamine is given in high doses to decrease the rebound effect on the nervous system as it adjusts to the absence of alcohol, and a benzodiazepine is given in high doses and titrated down over several days for the tranquilizing effect to prevent delirium tremens. 2. The client may have seizures, but Valium would control this. The client does not need a long-term anticonvulsant medication (Dilantin), and it is not known that the client needs an iron preparation (Feosol). The vitamin deficiency associated with delirium tremens is lack of thiamine, not iron. 3. Methadone is used for withdrawing clients from heroin, and Depakote can be used as a mood stabilizer in bipolar disorder or as an anticonvulsant. 4. The client does not need a diuretic, and a stimulant would produce an effect opposite to what is desired. TEST TAKING HINT: Option "3" could be eliminated if the test taker knew the treatment for heroin withdrawal, and option "4" could be reasoned out because a stimulant would produce an undesired effect.

The nurse is caring for clients on the rehabilitation unit. Which clients should the nurse assess first after receiving the change of-shift report? 1. The client with a C6 SCI who is complaining of dyspnea and has crackles in the lungs. 2. The client with an L4 SCI who is crying and very upset about being discharged home. 3. The client with an L2 SCI who is complaining of a headache and feeling very hot. 4. The client with a T4 SCI who is unable to move the lower extremities.

ANSWER: 1. 1. This client has signs/symptoms of a respiratory complication and should be assessed first. 2. This is a psychosocial need and should be addressed, but it does not have priority over a physiological problem. 3. A client with a lower SCI would not be at risk for autonomic dysreflexia; therefore, a complaint of headache and feeling hot would not be a priority over an airway problem. 4. The client with a T4 SCI would not be expected to move the lower extremities. TEST TAKING HINT: The nurse should assess the client who is at risk for dying or having some type of complication that requires intervention. Remember Maslow's hierarchy of needs, in which physiological problems are always priority and airway is the top physiological problem.

The charge nurse is making client assignments for a neuro-medical floor. Which client should be assigned to the most experienced nurse? 1. The elderly client who is experiencing a stroke in evolution. 2. The client diagnosed with a transient ischemic attack 48 hours ago. 3. The client diagnosed with Guillain-Barré syndrome who complains of leg pain. 4. The client with Alzheimer's disease who is wandering in the halls.

ANSWER: 1. 1. This client is experiencing a progressing stroke, is at risk for dying, and should be cared for by the most experienced nurse. 2. A TIA by definition lasts less than 24 hours, so this client should be stable at this time. 3. Pain is expected in clients with Guillain-Barré syndrome, and symptoms are on the lower half of the body, which does not affect the airway. Therefore, a less experienced nurse could care for this client. 4. The charge nurse could delegate much of the care of this client to a UAP.

The nurse is preparing the male client for an electroencephalogram (EEG). Which intervention should the nurse implement? 1. Explain that this procedure is not painful. 2. Premedicate the client with a benzodiazepine drug. 3. Instruct the client to shave all facial hair. 4. Tell the client it will cause him to see "floaters."

ANSWER: 1. 1. This procedure is not painful, although electrodes are attached to the scalp. The client will need to wash the hair after the procedure. 2. Antianxiety medication would make the client drowsy and could cause a false EEG reading. 3. There is no reason for facial hair to be shaved. 4. This procedure measures the electrical conductivity in the brain and does not cause the client to see "floaters" (spots before the eyes). Flashing bright lights may be used in an attempt to evoke a seizure.

The client is prescribed phenytoin (Dilantin), an anticonvulsant, for a seizure disorder. Which statement indicates the client understands the discharge teaching concerning this medication? 1. "I will brush my teeth after every meal." 2. "I will check my Dilantin level daily." 3. "My urine will turn orange while on Dilantin." 4. "I won't have any seizures while on this medication."

ANSWER: 1. 1. Thorough oral hygiene after each meal, gum massage, daily flossing, and regular dental care are essential to prevent or control gingival hyperplasia, which is a common occurrence in clients taking Dilantin. 2. A serum (venipuncture) Dilantin level is checked monthly at first and then, after a therapeutic level is attained, every six (6) months. 3. Dilantin does not turn the urine orange. 4. The use of Dilantin does not ensure that the client will not have any seizures, and, in some instances, the dosage may need to be adjusted or another medication may need to be used. TEST TAKING HINT: The test taker should realize that monitoring blood glucose levels using a glucometer is about the only level that is monitored daily; therefore, option "2," which calls for daily monitoring of Dilantin levels, could be eliminated. Remember, there are very few absolutes in the health-care field; therefore, option "4" could be ruled out because "won't have any" is an absolute.

The 28-year-old client is on the rehabilitation unit post spinal cord injury at level T10. Which collaborative team members should participate with the nurse at the case conference? Select all that apply. 1. Occupational Therapist (OT). 2. Physical therapist (PT). 3. Registered dietitian (RD). 4. Rehabilitation physician. 5. Social Worker (SW). 6. Patient care tech (PCT).

ANSWER: 2, 3, 4, 5. 1. This client is 28 years old and needs to learn how to function in the home to be able to manage activities of daily living. The OT works with clients to help them attain the highest level of functionality. 2. The PT will work with the client to develop upper body strength. 3. The RD will make sure that nutritional needs are being met. 4. The rehabilitation physician (physiatrist) is a rehabilitation specialist and an expert in bone, muscle, and nerves and treats injuries or illnesses that affect how a client moves. 5. The SW can assist the client with financial matters and can direct the client to programs that will help the client to receive training in a skill(s) that will assist in job placement. 6. The RN represents nursing in the case conference, not the PCT. TEST TAKING HINT: The test taker can eliminate option "6" by understanding the roles of the staff members. Nurses are with the client 24 hours a day in an inpatient care facility and frequently are the coordinators of the client's care. The nurse must know which discipline should be consulted.

The nurse is caring for a client diagnosed with an epidural hematoma. Which nursing interventions should the nurse implement? Select all that apply. 1. Maintain the head of the bed at 60 degrees of elevation. 2. Administer stool softeners daily. 3. Ensure the pulse oximeter reading is higher than 93%. 4. Perform deep nasal suction every two (2) hours. 5. Administer mild sedatives

ANSWER: 2, 3, 5. 1. The head of the bed should be elevated no more than 30 degrees to help decrease cerebral edema by gravity. 2. Stool softeners are initiated to prevent the Valsalva maneuver, which increases intracranial pressure. 3. Oxygen saturation higher than 93% ensures oxygenation of the brain tissues; decreasing oxygen levels increase cerebral edema. 4. Noxious stimuli, such as suctioning, increase intracranial pressure and should be avoided. 5. Mild sedatives will reduce the client's agitation; strong narcotics would not be administered because they decrease the client's level of consciousness. TEST TAKING HINT: In "select all that apply" questions, the test taker should look at each answer option as a separate entity. In option "1" the test taker should attempt to get a mental picture of the client's position in the bed. A 60-degree angle is almost upright in the bed. Would any client diagnosed with a head injury be placed this high? The client would be at risk for slumping over because of the inability to control the body position. Nasal suctioning, option "4," which increases intracranial pressure, should also be avoided.

Which diagnostic evaluation tool would the nurse use to assess the client's cognitive functioning? Select all that apply. 1. The Geriatric Depression Scale (GDS). 2. The St. Louis University Mental Status (SLUMS) scale. 3. The Mini-Mental Status Examination (MMSE) scale. 4. The Manic Depression vs Elderly Depression (MDED) scale. 5. The Functional Independence Measurement Scale (FIMS).

ANSWER: 2, 3. 1. The GDS assesses the older client for depression, not cognitive functioning. 2. The SLUMS scale is a measurement tool for cognitive functioning. 3. The MMSE scale is another tool to assess cognitive functioning. 4. There is no MDED scale and, in addition, depression is not cognitive functioning. 5. The FIMS measures how well the client can perform activities of daily living, not cognitive functioning. TEST TAKING HINT: The test taker could eliminate options "1," "4," and "5" based on the word "cognitive" in the stem of the question. The test taker should highlight any word that gives a clue as to what the question is asking. Words matter

The client has undergone a craniotomy for a brain tumor. Which data indicate a complication of this surgery? 1. The client complains of a headache at "3" to "4" on a 1-to-10 scale. 2. The client has an intake of 1,000 mL and an output of 3,500 mL. 3. The client complains of a raspy, sore throat. 4. The client experiences dizziness when trying to get up too quickly.

ANSWER: 2. 1. A headache after this surgery would be an expected occurrence, not a complication. 2. An output much larger than the intake could indicate the development of diabetes insipidus. Pressure on the pituitary gland can result in decreased production of vasopressin, the antidiuretic hormone (ADH). 3. A raspy sore throat is common after surgery due to the placement of the endotracheal tube during anesthesia. 4. Dizziness on arising quickly is expected; the client should be taught to rise slowly and call for assistance for safety. TEST TAKING HINT: The test taker could eliminate options "1" and "3" as expected occurrences following the surgery and not complications. Option "4" can also be expected.

The nurse is assessing a client experiencing motor loss as a result of a left-sided cerebrovascular accident (CVA). Which clinical manifestation would the nurse document? 1. Hemiparesis of the client's left arm and apraxia. 2. Paralysis of the right side of the body and ataxia. 3. Homonymous hemianopsia and diplopia. 4. Impulsive behavior and hostility toward family.

ANSWER: 2. 1. A left-sided cerebrovascular accident (CVA) will result in right-sided motor deficits; hemiparesis is weakness of one-half of the body, not just the upper extremity. Apraxia, the inability to perform a previously learned task, is a communication loss, not a motor loss. 2. The most common motor dysfunction of a CVA is paralysis of one side of the body, hemiplegia; in this case with a left-sided CVA, the paralysis would affect the right side. Ataxia is an impaired ability to coordinate movement. 3. Homonymous hemianopsia (loss of half of the visual field of each eye) and diplopia (double vision) are visual field deficits that a client with a CVA may experience, but they are not motor losses. 4. Personality disorders occur in clients with a right-sided CVA and are cognitive deficits; hostility is an emotional deficit. TEST TAKING HINT: Be sure to always notice adjectives describing something. In this case, "left-sided" describes the type of CVA. Also be sure to identify exactly what the question is asking—in this case, about "motor loss," which will help rule out many of the possible answer options.

The ambulance brings the client with a head injury to the emergency department. The client responds to painful stimuli by opening the eyes, muttering, and pulling away from the nurse. How would the nurse rate this client on the Glasgow Coma Scale? (Eye opening: 4=spontaneous, 3=to voice, 2=to pain, 1=none; Verbal response: 5=normal conversation, 4=disoriented conversation, 3=words, but not coherent, 2=no words, only sounds, 1=none; Motor response: 6=normal, 5=localizes to pain, 4=withdraws to pain,3=decorticate posture, 2=decerebrate, 1=none). 1. 3 2. 8 3. 10 4. 15

ANSWER: 2. 1. A score of 3 is the lowest score and indicates deep coma and impending brain death. 2. A score of 8 indicates severe increased intracranial pressure, but with appropriate care the client may survive. The nurse would rate the client at an 8: 1 for opening the eyes; 3 for verbal response; and 4 for motor response. 3. A score of 10 indicates moderately increased intracranial pressure. 4. A score of 15 is the highest score a client can receive, indicating normal function.

The nurse writes the nursing diagnosis "altered body temperature related to damaged temperature regulating mechanism" for a client with a head injury. Which would be the most appropriate goal? 1. Administer acetaminophen (Tylenol) for elevated temperature. 2. The client's temperature will remain less than 100°F. 3. Maintain the hypothermia blanket at 99°F for 24 hours. 4. The basal metabolic temperature will fluctuate no more than two (2) degrees.

ANSWER: 2. 1. Administering acetaminophen is an intervention, which is not a goal. 2. This is an appropriate goal. It addresses the client, addresses the problem (temperature elevation), and is measurable. 3. Maintaining the blanket temperature is a nursing intervention, which should eliminate this as a possible answer. 4. The basal metabolic temperature is evaluated for a woman trying to get pregnant; it helps indicate ovulation.

Which problem is the highest priority for the client diagnosed with West Nile virus? 1. Alteration in body temperature. 2. Altered tissue perfusion. 3. Fluid volume excess. 4. Altered skin integrity.

ANSWER: 2. 1. An alteration in body temperature in a client with West Nile virus would not be the highest priority. 2. Altered tissue perfusion would be the highest priority because it could be life threatening. 3. A problem of fluid volume excess would not apply for the client with West Nile virus. These clients are at risk for fluid volume deficit from nausea, vomiting, and hyperthermia. 4. A problem with skin integrity could apply to the client with immobility caused by West Nile virus, but it would not be the highest priority problem. TEST TAKING HINT: When prioritizing client problems, oxygenation is the highest priority problem according to Maslow, and tissue perfusion is oxygenation.

Which statement best describes the scientific rationale for alternating a nonnarcotic antipyretic and a nonsteroidal anti-inflammatory drug (NSAID) every two (2) hours to a female client diagnosed with bacterial meningitis? 1. This regimen helps to decrease the purulent exudate surrounding the meninges. 2. These medications will decrease intracranial pressure and brain metabolism. 3. These medications will increase the client's memory and orientation. 4. This will help prevent a yeast infection secondary to antibiotic therapy

ANSWER: 2. 1. Antibiotics would help decrease the bacterial infection in meningitis, which would cause the exudate. The drugs mentioned in the question would not. 2. Fever increases cerebral metabolism and intracranial pressure. Therefore, measures are taken to reduce body temperature as soon as possible, and alternating Tylenol and Motrin would be appropriate. 3. A nonnarcotic antipyretic (Tylenol) and an NSAID (Motrin) will not address the client's memory or orientation. 4. These medications do not prevent or treat a yeast infection. TEST TAKING HINT: The test taker must have a basic knowledge of the disease process and medications that are prescribed to treat a disease. Purulent drainage would require an antibiotic. Therefore, option "1" should be eliminated as a possible answer because the question is asking about NSAIDS and a non-narcotic antipyretic (Tylenol)

The client is diagnosed with a metastatic brain tumor, and radiation therapy is scheduled. The client asks the nurse, "Why not try chemotherapy first? It has helped my other tumors." The nurse's response is based on which scientific rationale? 1. Chemotherapy is only used as a last resort in caring for clients with brain tumors. 2. The blood-brain barrier prevents medications from reaching the brain. 3. Radiation therapy will have fewer side effects than chemotherapy. 4. Metastatic tumors become resistant to chemotherapy and it becomes useless.

ANSWER: 2. 1. Chemotherapy is systemic therapy that is used extensively in the care of clients diagnosed with cancer. However, most drugs have difficulty in crossing the blood-brain barrier and are not useful in treating brain tumors unless delivered by direct placement into the spinal column or directly to the ventricles of the brain by a device called an Omaya reservoir. 2. The blood-brain barrier is the body's defense mechanism for protecting the brain from chemical effects; in this case, it prevents the chemotherapy from being able to work on the tumor in the brain. 3. Radiation has about the same amount of side effects as chemotherapy, but the effects of radiation tend to last for a much longer time. 4. Some tumors do become resistant to the chemotherapy agents used. When this happens, the oncologist switches to different drugs. TEST TAKING HINT: The test taker can eliminate option "1" as a possible answer because it states that chemotherapy is used to treat brain tumors, but it does not tell the client why it is not being used. Option "2" is the only one that actually informs the client of a medical reason for not administering chemotherapy for a brain tumor

The client has glossopharyngeal nerve (cranial nerve IX) paralysis secondary to a stroke. Which referral would be most appropriate for this client? 1. Hospice nurse. 2. Speech therapist. 3. Physical therapist. 4. Occupational therapist.

ANSWER: 2. 1. Clients are referred to hospice when there is a life expectancy of less than six (6) months. This client has difficulty swallowing, which is not life threatening. 2. Speech therapists address the needs of clients who have difficulty with the innervations and musculature of the face and neck. This includes the swallowing reflex. 3. The physical therapist assists the client to ambulate and transfer (e.g., from bed to chair) and with muscle strength training. 4. The occupational therapist focuses on cognitive disability and activities of daily living

The public health nurse is giving a lecture on potential outbreaks of infectious meningitis. Which population is most at risk for an outbreak? 1. Clients recently discharged from the hospital. 2. Residents of a college dormitory. 3. Individuals who visit a third world country. 4. Employees in a high-rise office building.

ANSWER: 2. 1. Clients who have been hospitalized are weakened, but they are not at risk for contracting any type of meningitis. 2. Outbreaks of infectious meningitis are most likely to occur in dense community groups such as college campuses, jails, and military installations. 3. Third world countries do not pose a risk factor for meningitis. They provide a risk for hepatitis or tuberculosis. 4. Employees in a high-rise building do not live together and they have their own space; therefore, they are not at risk for developing meningitis. TEST TAKING HINT: The test taker must remember that the NCLEX-RN tests all areas of nursing, so always notice the type of nurse if this is mentioned in the stem. A public health nurse would not be concerned with third world countries.

A family member brings the client to the emergency department reporting that the 78-year-old father has suddenly become very confused and thinks he is living in 1942, that he has to go to war, and that someone is trying to poison him. Which question should the nurse ask the family member? 1. "Has your father been diagnosed with dementia?" 2. "What medication has your father taken today?" 3. "What have you given him that makes him think it's poison?" 4. "Does your father like to watch old movies on television?"

ANSWER: 2. 1. Dementia involves behavior changes that are irreversible and occur over time. Delirium, however, occurs suddenly (as in this man's symptom onset), is caused by an acute event, and is reversible. 2. Drug toxicity and interactions are common causes of delirium in the elderly. 3. This is blaming the family member for the client's paranoid ideation. 4. Watching old movies on television will not cause delirium

. Which diagnostic test is used to confirm the diagnosis of Amyotropic Lateral Sclerosis (ALS)? 1. Electromyogram (EMG). 2. Muscle biopsy. 3. Serum creatine kinase (CK). 4. Pulmonary function test.

ANSWER: 2. 1. EMG is done to differentiate a neuropathy from a myopathy, but it does not confirm ALS. 2. Biopsy confirms changes consistent with atrophy and loss of muscle fiber, both characteristic of ALS. 3. CK may or may not be elevated in ALS so it cannot confirm the diagnosis of ALS. 4. This is done as ALS progresses to determine respiratory involvement, but it does not confirm ALS. TEST TAKING HINT: The test taker must be clear as to what the question is asking. The word "confirm" is the key to answering this question correctly. The test taker would need to know that this disease affects the muscle tissue to correctly identify the answer

The client with ALS is admitted to the medical unit with shortness of breath, dyspnea, and respiratory complications. Which intervention should the nurse implement first? 1. Elevate the head of the bed 30 degrees. 2. Administer oxygen via nasal cannula. 3. Assess the client's lung sounds. 4. Obtain a pulse oximeter reading.

ANSWER: 2. 1. Elevating the head of the bed will enhance lung expansion, but it is not the first intervention. 2. Oxygen should be given immediately to help alleviate the difficulty breathing. Remember that oxygenation is a priority. 3. Assessment is the first part of the nursing process and is a priority, but assessment will not help the client breathe easier. 4. This is an appropriate intervention, but obtaining the pulse oximeter reading will not alleviate the client's respiratory distress. TEST TAKING HINT: The test taker should not automatically select assessment. Make sure that there is not another intervention that will directly help the client, especially if the client is experiencing a life-threatening complication

Which assessment data indicate that the client with a traumatic brain injury (TBI) exhibiting decorticate posturing on admission is responding effectively to treatment? 1. The client has flaccid paralysis. 2. The client has purposeful movement. 3. The client has decerebrate posturing with painful stimuli. 4. The client does not move the extremities.

ANSWER: 2. 1. Flaccid paralysis indicates a worsening of the increased ICP. 2. Purposeful movement indicates the client is getting better and is responding to the treatment. 3. Decerebrate positioning indicates a worsening of the increased intracranial pressure. 4. This is the same as flaccid paralysis and indicates a worsening of the increased intracranial pressure.

The client diagnosed with Parkinson's disease (PD) is being admitted with a fever and patchy infiltrates in the lung fields on the chest x-ray. Which clinical manifestations of PD would explain these assessment data? 1. Masklike facies and shuffling gait. 2. Difficulty swallowing and immobility. 3. Pill rolling of fingers and flat affect. 4. Lack of arm swing and bradykinesia

ANSWER: 2. 1. Masklike facies is responsible for lack of expression and is part of the motor manifestations of Parkinson's disease but is not related to the symptoms listed. Shuffling is also a motor deficit and does pose a risk for falling, but fever and patchy infiltrates on a chest x-ray do not result from a gait problem. They are manifestations of a pulmonary complication. 2. Difficulty swallowing places the client at risk for aspiration. Immobility predisposes the client to pneumonia. Both clinical manifestations place the client at risk for pulmonary complications. 3. Pill rolling of fingers and flat affect do not have an impact on the development of pulmonary complications. 4. Arm swing and bradykinesia are motor deficits. TEST TAKING HINT: The nurse must recognize the clinical manifestations of a disease and the resulting bodily compromise. In this situation, fever and patchy infiltrates on a chest x-ray indicate a pulmonary complication. Options "1," "3," and "4" focus on motor problems and could be ruled out as too similar. Only option "2" includes dissimilar information.

The client is being admitted to rule out a brain tumor. Which classic triad of symptoms supports a diagnosis of brain tumor? 1. Nervousness, metastasis to the lungs, and seizures. 2. Headache, vomiting, and papilledema. 3. Hypotension, tachycardia, and tachypnea. 4. Abrupt loss of motor function, diarrhea, and changes in taste.

ANSWER: 2. 1. Nervousness is not a symptom of a brain tumor, and brain tumors rarely metastasize outside of the cranium. Brain tumors kill by occupying space and increasing intracranial pressure. Although seizures are not uncommon with brain tumors, seizures are not part of the classic triad of symptoms. 2. The classic triad of symptoms suggesting a brain tumor includes a headache that is dull, unrelenting, and worse in the morning; vomiting unrelated to food intake; and edema of the optic nerve (papilledema), which occurs in 70% to 75% of clients diagnosed with brain tumors. Papilledema causes visual disturbances such as decreased visual acuity and diplopia. 3. Hypertension and bradycardia, not hypotension and tachycardia, occur with increased intracranial pressure resulting from pressure on the cerebrum. Tachypnea does not occur with brain tumors. 4. Abrupt loss of motor function occurs with a stroke; diarrhea does not occur with a brain tumor, and the client with a brain tumor does not experience a change in taste. TEST TAKING HINT: The test taker can rule out option "4" because of the symptom of diarrhea, which is a gastrointestinal symptom, not a neurological one. Considering the other three possible choices, the symptom of "headache" would make sense for a client with a brain tumor.

The unlicensed assistive personnel (UAP) is attempting to put an oral airway in the mouth of a client having a tonic-clonic seizure. Which action should the primary nurse take? 1. Help the UAP to insert the oral airway in the mouth. 2. Tell the UAP to stop trying to insert anything in the mouth. 3. Take no action because the UAP is handling the situation. 4. Notify the charge nurse of the situation immediately

ANSWER: 2. 1. Once the seizure has started, no one should attempt to put anything in the client's mouth. 2. The nurse should tell the UAP to stop trying to insert anything in the mouth of the client experiencing a seizure. Broken teeth and injury to the lips and tongue may result from trying to place anything in the clenched jaws of a client having a tonic-clonic seizure. 3. The primary nurse is responsible for the action of the UAP and should stop the UAP from doing anything potentially dangerous to the client. No one should attempt to pry open the jaws that are clenched in a spasm to insert anything. 4. The primary nurse must correct the action of the UAP immediately, prior to any injury occurring to the client and before notifying the charge nurse. TEST TAKING HINT: The nurse is responsible for the actions of the unlicensed assistive personnel and must correct the behavior immediately.

The rehabilitation nurse caring for the client with an Lumbar SCI is developing the nursing care plan. Which intervention should the nurse implement? 1. Keep oxygen via nasal cannula on at all times. 2. Administer low-dose subcutaneous anticoagulants. 3. Perform active lower extremity ROM exercises. 4. Refer to a speech therapist for ventilatorassisted speech.

ANSWER: 2. 1. Oxygen is administered initially to maintain a high arterial partial pressure of oxygen (PaO2) because hypoxemia can worsen a neurological deficit to the spinal cord initially, but this client is in the rehabilitation department and thus not in the initial stages of the injury. 2. Deep vein thrombosis (DVT) is a potential complication of immobility, which can occur because the client cannot move the lower extremities as a result of the L1 SCI. Low-dose anticoagulation therapy (Lovenox) helps prevent blood from coagulating, thereby preventing DVTs. 3. The client is unable to move the lower extremities. The nurse should do passive ROM exercises. 4. A client with a spinal injury at C4 or above would be dependent on a ventilator for breathing, but a client with an L1 SCI would not. TEST TAKING HINT: The test taker should notice any adjectives such as "rehabilitation," which should clue the test taker into ruling out oxygen, which is for the acute phase. The test taker should also be very selective if choosing an answer with a definitive word such as "all" (option "1").

The client is diagnosed with expressive aphasia. Which psychosocial client problem would the nurse include in the plan of care? 1. Potential for injury. 2. Powerlessness. 3. Disturbed thought processes. 4. Sexual dysfunction.

ANSWER: 2. 1. Potential for injury is a physiological, not a psychosocial, problem. 2. Expressive aphasia means that the client cannot communicate thoughts but understands what is being communicated; this leads to frustration, anger, depression, and the inability to verbalize needs, which, in turn, causes the client to have a lack of control and feel powerless. 3. A disturbance in thought processes is a cognitive problem; with expressive aphasia the client's thought processes are intact. 4. Sexual dysfunction can have a psychosocial or physical component, but it is not related to expressive aphasia. TEST TAKING HINT: The test taker should always make sure that the choice selected as the correct answer matches what the question is asking. The stem has the adjective "psychosocial," so the correct answer must address psychosocial needs.

The client newly diagnosed with Parkinson's Disease (PD) asks the nurse, "Why can't I control these tremors?" Which is the nurse's best response? 1. "You can control the tremors when you learn to concentrate and focus on the cause." 2. "The tremors are caused by a lack of the chemical dopamine in the brain; medication may help." 3. "You have too much acetylcholine in your brain causing the tremors but they will get better with time." 4. "You are concerned about the tremors? If you want to talk I would like to hear how you feel."

ANSWER: 2. 1. Sometimes a client can temporarily overcome a freezing of motion or a tremor by making an intentional movement, but the issue is not enough of the neurotransmitter, dopamine, in the brain. Concentration or focusing will not increase the amount of dopamine available in the brain. 2. This is the cause of the tremors, cogwheel motion of movement, and bradykinesia, and so forth. It is also in layman's terms that the client can understand and provides some measure of hope that something can be done without giving false reassurance. 3. The issue is dopamine. The acetylcholine effects are caused by the dopamine not being available to counteract the acetylcholine. 4. This is a therapeutic response and the client is asking for information. TEST TAKING HINT: The test taker should read the stem of the question carefully and determine what the client is requesting. The client is newly diagnosed and wants to know about the disease. The nurse should respond to the client's question.

Which assessment data would make the nurse suspect that the client with a C7 spinal cord injury is experiencing autonomic dysreflexia? 1. Abnormal diaphoresis. 2. A severe throbbing headache. 3. Sudden loss of motor function. 4. Spastic skeletal muscle movement.

ANSWER: 2. 1. Sweating is not a sign of autonomic dysreflexia. 2. A throbbing headache is the classic sign of autonomic dysreflexia, which is caused by a stimulus such as a full bladder. 3. Sudden loss of motor function occurs with the original injury. Autonomic dysreflexia does not occur until spinal shock has resolved; it usually occurs in the rehabilitation phase. 4. Spastic skeletal muscle movement could be secondary to the reflex arc in lower motor neuron injuries.

The 34-year-old male client with an SCI is sharing with the nurse that he is worried about finding employment after being discharged from the rehabilitation unit. Which intervention should the nurse implement? 1. Refer the client to the American Spinal Cord Injury Association (ASIA). 2. Refer the client to the state rehabilitation commission. 3. Ask the social worker (SW) about applying for disability. 4. Suggest that the client talk with his significant other about this concern.

ANSWER: 2. 1. The ASIA is an appropriate referral for living with this condition, but it does not help find gainful employment after the injury. 2. The rehabilitation commission of each state will help evaluate and determine if the client can receive training or education for another occupation after injury. 3. The client is not asking about disability; he is concerned about employment. Therefore, the nurse needs to make a referral to the appropriate agency. 4. This does not address the client's concern about gainful employment. TEST TAKING HINT: If the question mentions a specific age for a client, the nurse should consider it when attempting to answer the question. This is a young person who needs to find gainful employment. Remember Erickson's stages of growth and development.

The client diagnosed with a closed head injury is admitted to the rehabilitation department. Which medication order would the nurse question? 1. A subcutaneous anticoagulant. 2. An intravenous osmotic diuretic. 3. An oral anticonvulsant. 4. An oral proton pump inhibitor

ANSWER: 2. 1. The client in rehabilitation is at risk for the development of deep vein thrombosis; therefore, this is an appropriate medication. 2. An osmotic diuretic would be ordered in the acute phase to help decrease cerebral edema, but this medication would not be expected to be ordered in a rehabilitation unit. 3. Clients with head injuries are at risk for post-traumatic seizures; thus an oral anticonvulsant would be administered for seizure prophylaxis. 4. The client is at risk for a stress ulcer; therefore, an oral proton pump inhibitor would be an appropriate medication. TEST TAKING HINT: The client is in the rehabilitation unit and therefore must be stable. The use of any intravenous medication should be questioned under those circumstances, even if the test taker is not sure why the medication may be considered.

The 29-year-old client who was employed as a forklift operator sustains a traumatic brain injury (TBI) secondary to a motor-vehicle accident. The client is being discharged from the rehabilitation unit after three (3) months and has cognitive deficits. Which goal would be most realistic for this client? 1. The client will return to work within six (6) months. 2. The client is able to focus and stay on task for 10 minutes. 3. The client will be able to dress self without assistance. 4. The client will regain bowel and bladder control.

ANSWER: 2. 1. The client is at risk for seizures and does not process information appropriately. Allowing him to return to his occupation as a forklift operator is a safety risk for him and other employees. Vocational training may be required. 2. "Cognitive" pertains to mental processes of comprehension, judgment, memory, and reasoning. Therefore, an appropriate goal would be for the client to stay on task for 10 minutes. 3. The client's ability to dress self addresses selfcare problems, not a cognitive problem. 4. The client's ability to regain bowel and bladder control does not address cognitive deficits. TEST TAKING HINT: The test taker must note adjectives closely. The question is asking about "cognitive" deficits; therefore, the correct answer must address cognition.

Which finding is considered to be one of the warning signs of developing Alzheimer's disease? 1. Difficulty performing familiar tasks. 2. Problems with orientation to date, time, and place. 3. Having problems focusing on a task. 4. Atherosclerotic changes in the vessels.

ANSWER: 2. 1. The client may experience minor difficulty in work or social activities but has adequate cognitive ability to hide the loss and continue to function independently. 2. Disorientation to time and place is a warning sign. 3. Not being able to focus on a task is more likely a sign of attention deficit-hyperactivity disorder. 4. Atherosclerotic changes are not warning signs of Alzheimer's disease. Amyloid protein plaques do appear to have something to do with the disease, but they are not found until autopsy.

The client with a cervical fracture is being discharged in a halo device. Which teaching instruction should the nurse discuss with the client? 1. Discuss how to correctly remove the insertion pins. 2. Instruct the client to report reddened or irritated skin areas. 3. Inform the client that the vest liner cannot be changed. 4. Encourage the client to remain in the recliner as much as possible.

ANSWER: 2. 1. The halo device is applied by inserting pins into the skull, and the client cannot remove them; the pins should be checked for signs of infection. 2. Reddened areas, especially under the brace, must be reported to the HCP because pressure ulcers can occur when wearing this appliance for an extended period. 3. The vest liner should be changed for hygiene reasons, but the halo part is not removed. 4. The client should be encouraged to ambulate to prevent complications of immobility. TEST TAKING HINT: The test taker would need basic knowledge about a halo device to answer this question easily, but some clues are in the stem. A cervical fracture is in the upper portion of the spine or neck area, and most people understand that a halo is something that surrounds the forehead or higher. So the test taker could get a mental image of a device that must span this area of the body and maintain alignment of the neck. If an HCP attaches pins into the head, then the test taker could assume that they were not to be removed by the client. Redness usually indicates some sort of problem with the skin.

The client with a history of migraine headaches comes to the clinic and reports that a migraine is coming because the client is experiencing bright spots before the eyes. Which phase of migraine headaches is the client experiencing? 1. Prodrome phase. 2. Aura phase. 3. Headache phase. 4. Recovery phase.

ANSWER: 2. 1. The prodrome phase occurs hours to days before the migraine headache. 2. This is the aura phase, which is characterized by focal neurological symptoms. 3. The headache phase occurs when vasodilation occurs in the brain, along with a decline in serotonin levels, causing a throbbing headache. 4. The recovery phase is when the pain begins to gradually subside

The resident in a long-term care facility fell during the previous shift and has a laceration in the occipital area that has been closed with steristrips. Which signs/symptoms would warrant transferring the resident to the emergency department? 1. A 4-cm area of bright red drainage on the dressing. 2. A weak pulse, shallow respirations, and cool pale skin. 3. Pupils that are equal, react to light, and accommodate. 4. Complaints of a headache that resolves with medication.

ANSWER: 2. 1. The scalp is a very vascular area and a moderate amount of bleeding would be expected. 2. These signs/symptoms—weak pulse, shallow respirations, cool pale skin—indicate increased intracranial pressure from cerebral edema secondary to the fall, and they require immediate attention. 3. This is a normal pupillary response and would not warrant intervention. 4. A headache that resolves with medication is not an emergency situation, and the nurse would expect the client to have a headache after the fall; a headache not relieved with Tylenol would warrant further investigation. TEST TAKING HINT: The test taker is looking for an answer option that is not normal for the client's situation. Of the options listed, three would be expected and would not warrant a trip to the emergency department.

Which assessment data should the nurse expect to observe for the client diagnosed with Parkinson's disease? 1. Ascending paralysis and pain. 2. Masklike facies and pill rolling. 3. Diplopia and ptosis. 4. Dysphagia and dysarthria.

ANSWER: 2. 1. The spread of pain and paralysis are signs/ symptoms of Guillain-Barré syndrome. 2. Masklike facies and pill rolling are signs/ symptoms of Parkinson's disease, along with cogwheeling, postural instability, and stooped and shuffling gait. 3. Diplopia and ptosis are signs/symptoms of myasthenia gravis. 4. Dysphagia and dysarthria are signs/ symptoms of myasthenia gravis.

The public health nurse is discussing St. Louis encephalitis with a group in the community. Which instruction should the nurse provide to help prevent an outbreak? 1. Yearly vaccinations for the disease. 2. Advise that the city should spray for mosquitoes. 3. The use of gloves when gardening. 4. Not going out at night.

ANSWER: 2. 1. There is no vaccine for preventing encephalitis. 2. Mosquitoes are the vectors that spread the disease, and spraying to kill mosquito larvae will help prevent an outbreak in the community. 3. Gloves will not protect a person from being bitten by a mosquito. 4. Mosquitoes are more prevalent at night, but this is an unrealistic intervention and will not help prevent an outbreak.

A 20-year-old female client who tried lysergic acid diethylamide (LSD) as a teen tells the nurse that she has bad dreams that make her want to kill herself. Which is the explanation for this occurrence? 1. These occurrences are referred to as "holdover reactions" to the drug. 2. These are flashbacks to a time when the client had a "bad trip." 3. The drug is still in the client's body and causing these reactions. 4. The client is suicidal and should be on one-to-one precautions.

ANSWER: 2. 1. These reactions are called "flashbacks." 2. Flashback reactions occur after the use of hallucinogens in which the client relives a bad episode that occurred while using the drug. 3. The drug is gone from the body, but the mind-altering effects can occur at any time in the form of memory flashbacks. 4. The client stated that the dreams are causing her distress. She is asking for help with the dreams, not planning her suicide. TEST TAKING HINT: The client is 20 years old and took the drug in her teens; drugs do not stay in the body for extended periods. This eliminates option "3."

The nurse is developing a plan of care for a client diagnosed with aseptic meningitis secondary to a brain tumor. Which nursing goal would be most appropriate for the client problem "altered cerebral tissue perfusion"? 1. The client will be able to complete activities of daily living. 2. The client will be protected from injury if seizure activity occurs. 3. The client will be afebrile for 48 hours prior to discharge. 4. The client will have elastic tissue turgor with ready recoil

ANSWER: 2. 1. This goal is not related to altered cerebral tissue perfusion, but it would be a goal for selfcare deficit. 2. A client with a problem of altered cerebral tissue perfusion is at risk for seizure activity secondary to focal areas of cortical irritability; therefore, the client should be on seizure precautions. 3. This would be an appropriate goal for the client who has a problem of infection. 4. This would be an appropriate goal for the client who has a problem of dehydration. TEST TAKING HINT: The goal must be related to the problem—in this case, "altered cerebral tissue perfusion."

The client diagnosed with ALS asks the nurse, "I know this disease is going to kill me. What will happen to me in the end?" Which statement by the nurse would be most appropriate? 1. "You are afraid of how you will die?" 2. "Most people with ALS die of respiratory failure." 3. "Don't talk like that. You have to stay positive." 4. "ALS is not a killer. You can live a long life."

ANSWER: 2. 1. This is a therapeutic response, but the client is asking for specific information. 2. About 50% of clients die within two (2) to five (5) years from respiratory failure, aspiration pneumonia, or another infectious process. 3. The nurse should allow the client to talk freely about the disease process and should provide educational and emotional support. 4. This is incorrect information; ALS is a disease that results in death within 5 years in most cases. TEST TAKING HINT: When the client is asking for factual information, the nurse should provide accurate and truthful information. This helps foster a trusting client-nurse relationship. A therapeutic response (option "1") should be used when the client needs to ventilate feelings and is not asking specific questions about the disease process

The client diagnosed with a brain abscess has become lethargic and difficult to arouse. Which intervention should the nurse implement first? 1. Implement seizure precautions. 2. Assess the client's neurological status. 3. Close the drapes and darken the room. 4. Prepare to administer an IV steroid.

ANSWER: 2. 1. This is an appropriate intervention, but it is not the first. 2. Remember, assessment is the first step of the nursing process and should be implemented first whenever there is a change in the client's behavior. 3. This helps prevent stimulation that could initiate a seizure, but it is not the first intervention. 4. Steroids may be administered to clients with brain abscesses to decrease inflammation, but assessment is the first intervention.

The nurse is caring for the client with encephalitis. Which intervention should the nurse implement first if the client is experiencing a complication? 1. Examine pupil reactions to light. 2. Assess level of consciousness. 3. Observe for seizure activity. 4. Monitor vital signs every shift.

ANSWER: 2. 1. This is an important area to assess for neurological deterioration, but it is not the first indication of increased intracranial pressure. 2. This is the most important assessment data. A change in level of consciousness is usually the first sign of neurological deterioration. 3. Seizures can occur with inflammation from encephalitis, but their occurrence does not indicate that the client has increased intracranial pressure resulting from a worsening condition. 4. This is important information to assess, but changes in vital signs are not the first sign and symptom of increased intracranial pressure. TEST TAKING HINT: The word "first" asks the test taker to prioritize the interventions. Usually all the options are interventions that the nurse should do, but the question implies that the client may be deteriorating. Level of consciousness is the most sensitive indicator of neurological deficit.

The nurse is conducting a support group for clients diagnosed with Parkinson's disease and their significant others. Which information regarding psychosocial needs should be included in the discussion? 1. The client should discuss feelings about being placed on a ventilator. 2. The client may have rapid mood swings and become easily upset. 3. Pill-rolling tremors will become worse when the medication is wearing off. 4. The client may automatically start to repeat what another person says.

ANSWER: 2. 1. This is information that should be discussed when filling out an advance directive form. A ventilator is used to treat a physiological problem. 2. These are psychosocial manifestations of PD. These should be discussed in the support meeting. 3. The reduction in the unintentional pillrolling movement of the hands is controlled at times by the medication; this is a physiological problem. 4. Echolalia is a speech deficit in which the client automatically repeats the words or sentences of another person; this is a physiological problem. TEST TAKING HINT: Psychosocial problems should address the client's feelings or interactions with another person

Which information should be shared with the client diagnosed with stage I Alzheimer's disease who is prescribed donepezil (Aricept), a cholinesterase inhibitor? 1. The client must continue taking this medication forever to maintain function. 2. The drug may delay the progression of the disease, but it does not cure it. 3. A serum drug level must be obtained monthly to evaluate for toxicity. 4. If the client develops any muscle aches, the HCP should be notified.

ANSWER: 2. 1. This is not a true statement. The client will no longer be prescribed this medication as the disease progresses and it becomes ineffective. 2. This medication does not cure Alzheimer's disease, and at some point it will become ineffective as the disease progresses. 3. There is no monthly drug level to be monitored. Toxicity includes jaundice and gastrointestinal distress. 4. Muscle aches are an adverse effect of the lipid-lowering medications, not of Aricept.

The male client diagnosed with a brain tumor is scheduled for a magnetic resonance imaging (MRI) scan in the morning. The client tells the nurse that he is scared. Which response by the nurse indicates an appropriate therapeutic response? 1. "MRIs are loud but there will not be any invasive procedure done." 2. "You're scared. Tell me about what is scaring you." 3. "This is the least thing to be scared about— there will be worse." 4. "I can call the MRI tech to come and talk to you about the scan."

ANSWER: 2. 1. This is providing information and is not completely factual. MRIs are loud, but frequently the client will require an IV access (an invasive procedure) to be started for a contrast medium to be injected. 2. This is restating and offering self. Both are therapeutic responses. 3. This statement is belittling the client's feeling. 4. This is not dealing with the client's concerns and is passing the buck. The nurse should explore the client's feeling to determine what is concerning the client. The MRI may or may not be the problem. The client may be afraid of the results of the MRI. TEST TAKING HINT: When the question asks the test taker for a therapeutic response, the test taker should choose the response that directly addresses the client's feelings.

The client diagnosed with substance abuse is being discharged from a drug and alcohol rehabilitation facility. Which information should the nurse teach the client? 1. "Do not go anyplace where you can be tempted to use again." 2. "It is important that you attend a 12-step meeting regularly." 3. "Now that you are clean, your family will be willing to see you again." 4. "You should explain to all your coworkers what has happened."

ANSWER: 2. 1. This is unrealistic. Most restaurants serve some form of alcoholic beverage. It is good advice for the client to try to avoid situations that provide the temptation to use drugs or alcohol again. 2. The client will require a follow-up program such as 12-step meetings if the client is not to relapse. 3. The nurse does not know that this is true. 4. The client should discuss the history with the people the client chooses. TEST TAKING HINT: The test taker must notice descriptive words such as "all" or "do not go anywhere." These words or phrases are absolutes that should cause the test taker to eliminate the options containing them.

The client diagnosed with atrial fibrillation complains of numbness and tingling of her left arm and leg. The nurse assesses facial drooping on the left side and slight slurring of speech. Which nursing interventions should the nurse implement first? 1. Schedule a STAT Magnetic Resonance Imaging of the brain. 2. Call a Code STROKE. 3. Notify the health-care provider (HCP). 4. Have the client swallow a glass of water.

ANSWER: 2. 1. This may be needed once the client is stable, but the first action is to get the needed personnel to intervene to prevent lasting damage for the client. 2. A Code STROKE (for an RRT related to a stroke) has been instituted in most facilities to have personnel to respond so that there is no delay in initiating interventions, thus reducing the impact of a cerebrovascular accident (stroke) on a client. According to Reuters: The National Stroke Association suggests using the word FAST to help recognize the signs of a stroke. F stands for Face: ask the person to smile, and see whether one side of the face droops. A stands for Arms: if both arms are raised, does one drift to the side? S stands for Speech: is it slurred, or strange? And T stands for Time: don't waste time before calling 911 if someone has started to show any of these signs. The American Stroke Association says that during a stroke, "Time lost is brain lost." An estimated two million brain cells die every minute during a stroke, increasing risk of permanent brain damage, disability and death. (Gaitan, 2014) 3. The first nursing intervention is to call the Code STROKE, then the health-care provider would be notified next. 4. Having the client swallow could be an assessment step but not a glass of water and with standby suction available in case the client is unable to swallow. TEST TAKING HINT: The test taker should remember that certain physiological processes carry risks that have to be contended with. Atrial fibrillation can cause the blood to become stagnant and coat the atrial interior surfaces. If this coating of blood breaks loose, then the result can be an intracranial embolus.

The client is diagnosed with meningococcal meningitis. Which preventive measure would the nurse expect the health-care provider to order for the significant others in the home? 1. The Haemophilus influenzae vaccine. 2. Antimicrobial chemoprophylaxis. 3. A 10-day dose pack of corticosteroids. 4. A gamma globulin injection.

ANSWER: 2. 1. This vaccine must be administered prior to exposure to build up an immunity to prevent meningitis resulting from Haemophilus influenzae. 2. Chemoprophylaxis includes administering medication that will prevent infection or eradicate the bacteria and the development of symptoms in people who have been in close proximity to the client. Medications include rifampin (Rifadin), ciprofloxacin (Cipro), and ceftriaxone (Rocephin). 3. Steroids are used as an adjunct therapy in treatment of clients diagnosed with acute bacterial meningitis. They would not be given as a prophylactic measure to others in the home. 4. Gamma globulin provides passive immunity to clients who have been exposed to hepatitis. It is not appropriate in this situation. TEST TAKING HINT: The key word in the stem is "preventive." The test taker must pay close attention to the adjectives.

The client asks the nurse, "What causes Creutzfeldt-Jakob disease?" Which statement would be the nurse's best response? 1. "The person must have been exposed to an infected prion." 2. "It is mad cow disease, and eating contaminated meat is the cause." 3. "This disease is caused by a virus that is in stagnant water." 4. "A fungal spore from the lungs infects the brain tissue."

ANSWER: 2. 1. Would a layperson know what a prion is? This is using medical jargon, which is not the nurse's best response. 2. This is the cause of this disease and would be the best response. 3. A virus is not the cause of Creutzfeldt-Jakob disease. 4. Fungal spores do not cause this disease

The client is undergoing post-thrombolytic therapy for a stroke. The health-care provider has ordered heparin to be infused at 1,000 units per hour. The solution comes as 25,000 units of heparin in 500 mL of D5W. At what rate will the nurse set the pump? ___________

ANSWER: 20 mL/hr. To arrive at the answer, the test taker must divide 25,000 units by 500 mL = 50 units in 1 mL. Divide 1,000 units by 50 units = 20 mL/hr.

The client is to receive a 100-mL intravenous antibiotic over 30 minutes via an intravenous pump. At what rate should the nurse set the IV pump? ___________

ANSWER: 200 mL/hr. This is a basic math question. The IV pump is calculated in mL/hr, so the nurse must double the rate to infuse the IV solution in 30 minutes. TEST TAKING HINT: This is a basic calculation that the nurse should be able to make even without a calculator.

The client is withdrawing from a heroin addiction. Which interventions should the nurse implement? Select all that apply. 1. Initiate seizure precautions. 2. Check vital signs every eight (8) hours. 3. Place the client in a quiet, calm atmosphere. 4. Have a consent form signed for HIV testing. 5. Provide the client with sterile needles.

ANSWER: 3, 4. 1. Chills, sweats, and gooseflesh occur with heroin withdrawal, but seizures do not usually occur, so seizure precautions are not necessary. 2. Vital signs should be taken more frequently, every two (2) to four (4) hours, depending on the client's condition. 3. The client should be in an atmosphere with little stimulation. The client will be irritable and fearful. 4. Heroin is administered intravenously. Heroin addicts are at high risk for HIV as a result of shared needles and thus should be tested for HIV. 5. The client is withdrawing from heroin, so providing needles is inappropriate. Providing sterile needles to IV drug users is controversial, but it attempts to decrease the incidence of HIV among drug users. TEST TAKING HINT: A "select all that apply" question will usually have more than one correct answer. One option cannot eliminate another.

The nurse is caring for several clients. Which client would the nurse assess first after receiving the shift report? 1. The 22-year-old male client diagnosed with a concussion who is complaining someone is waking him up every two (2) hours. 2. The 36-year-old female client admitted with complaints of left-sided weakness who is scheduled for a magnetic resonance imaging (MRI) scan. 3. The 45-year-old client admitted with blunt trauma to the head after a motorcycle accident who has a Glasgow Coma Scale (GCS) score of 6. 4. The 62-year-old client diagnosed with a cerebrovascular accident (CVA) who has expressive aphasia.

ANSWER: 3. 1. A client with a head injury must be awakened every two (2) hours to determine alertness; decreasing level of consciousness is the first indicator of increased intracranial pressure. 2. A diagnostic test, MRI, would be an expected test for a client with left-sided weakness and would not require immediate attention. 3. The Glasgow Coma Scale is used to determine a client's response to stimuli (eyeopening response, best verbal response, and best motor response) secondary to a neurological problem; scores range from 3 (deep coma) to 15 (intact neurological function). A client with a score of 6 should be assessed first by the nurse. 4. The nurse would expect a client diagnosed with a CVA (stroke) to have some sequelae of the problem, including the inability to speak. TEST TAKING HINT: This is a prioritizing question that asks the test taker to determine which client has priority when assessing all four clients. The nurse should assess the client who has abnormal data for the disease process.

The nurse is caring for clients on a medicalsurgical floor. Which clients should be assessed first? 1. The 65-year-old client diagnosed with seizures who is complaining of a headache that is a "2" on a 1-to-10 scale. 2. The 24-year-old client diagnosed with a T10 spinal cord injury who cannot move his toes. 3. The 58-year-old client diagnosed with Parkinson's disease who is crying and worried about her facial appearance. 4. The 62-year-old client diagnosed with a cerebrovascular accident who has a resolving left hemiparesis.

ANSWER: 3. 1. A headache of "2" on a 1-to-10 scale is a mild headache. 2. A spinal cord injury at T10 involves deficits at approximately the waist area. Inability to move the toes would be expected. 3. Body image is a concern for clients diagnosed with PD. This client is the one client who is not experiencing expected sequelae of the disease. 4. This client is getting better; "resolving" indicates an improvement in the client's clinical manifestations. TEST TAKING HINT: At times a psychological problem can have priority. All the physical problems are expected and are not life threatening or life altering.

Which assessment data would make the nurse suspect that the client has amyotrophic lateral sclerosis? 1. History of a cold or gastrointestinal upset in the last month. 2. Complaints of double vision and drooping eyelids. 3. Fatigue, progressive muscle weakness, and twitching. 4. Loss of sensation below the level of the umbilicus.

ANSWER: 3. 1. A history of a cold or gastrointestinal upset in the last month would be assessment data that would make the nurse suspect Guillain-Barré syndrome. 2. Complaints of double vision and drooping eyelids would make the nurse suspect myasthenia gravis. 3. Fatigue, progressive muscle weakness, and twitching are signs of ALS, a progressive neurological disease in which there is a loss of motor neurons. There is no cure, but recently a medication to slow the deterioration of the motor neurons has been found. 4. Loss of sensation would make the nurse suspect some type of spinal cord injury.

The nurse writes the problem "high risk for impaired skin integrity" for the client with an L5-6 spinal cord injury. Which intervention should the nurse include in the plan of care? 1. Perform active range-of-motion exercise. 2. Massage the legs and trochanters every shift. 3. Arrange for a Roho cushion in the wheelchair. 4. Apply petroleum-based lotion to the extremities.

ANSWER: 3. 1. A patient with an L5-6 spinal cord injury is paralyzed and cannot perform active ROM exercises. 2. Massaging bony prominences can cause trauma to the underlying blood vessels and will increase the risk for skin breakdown. 3. The nurse must realize that the client is at risk for skin breakdown even when sitting in the chair. A Roho cushion is an air-filled cushion that provides reduced pressure on the ischium. 4. Lotion will not prevent skin breakdown and should be water based, not petroleum based.

The client is diagnosed with a pituitary tumor and is scheduled for a transsphenoidal hypophysectomy. Which preoperative instruction is important for the nurse to teach? 1. There will be a large turban dressing around the skull after surgery. 2. The client will not be able to eat for four (4) or five (5) days postop. 3. The client should not blow the nose for two (2) weeks after surgery. 4. The client will have to lie flat for 24 hours following the surgery

ANSWER: 3. 1. A transsphenoidal hypophysectomy is done by an incision above the gumline and through the sinuses to reach the sella turcica, where the pituitary is located. 2. The client will be given regular food when awake and able to tolerate food. 3. Blowing the nose creates increased intracranial pressure and could result in a cerebrospinal fluid leak. 4. The client will return from surgery with the head of the bed elevated to about 30 degrees; this allows for gravity to assist in draining the cerebrospinal fluid. TEST TAKING HINT: The test taker must know the procedures for specific disease processes to answer this question, but anatomical positioning of the pituitary gland (just above the sinuses) could help to eliminate option "1," which calls for a large turban dressing

The client admitted to the hospital to rule out encephalitis is being prepared for a lumbar puncture. Which instructions should the nurse teach the client regarding care postprocedure? 1. Instruct that all invasive procedures require a written permission. 2. Explain that this allows analysis of a sample of the cerebrospinal fluid. 3. Tell the client to increase fluid intake to 300 mL for the next 48 hours. 4. Discuss that lying supine with the head flat will prevent all hematomas.

ANSWER: 3. 1. A written consent is given for all invasive procedures, but this would reflect care before the lumbar puncture, not after. 2. This is information that would be shared with the client about the reason the procedure would be done but not care after. 3. The nurse should teach this information to prevent the severe, throbbing, "spinal headache" caused by the decrease in cerebrospinal fluid. 4. The client should lie with the head of the bed flat for four (4) to eight (8) hours after the lumbar puncture, but this position would not prevent all hematomas. TEST TAKING HINT: When the test taker is trying to eliminate options, any that have absolute words, such as "all," "never," and "always," are usually wrong and can be eliminated quickly. Rarely is any activity always or never done

The nurse educator is presenting an in-service on seizures. Which disease process is the leading cause of seizures in the elderly? 1. Alzheimer's disease. 2. Parkinson's disease (PD). 3. Cerebral Vascular Accident (CVA, stroke). 4. Brain atrophy due to aging

ANSWER: 3. 1. Alzheimer's disease does not lead to seizures. 2. Parkinson's disease does not cause seizures. 3. A CVA (stroke) is the leading cause of seizures in the elderly; increased intracranial pressure associated with the stroke can lead to seizures. 4. Brain atrophy is not associated with seizures. TEST TAKING HINT: All four answer options are associated with the brain, neurological system, and aging. However, options "1," "2," and "4" usually occur over time, with the condition gradually getting worse, and thus can be eliminated as a cause of seizures, which are usually sudden.

The client is scheduled for an electroencephalogram (EEG) to help diagnose a seizure disorder. Which preprocedure teaching should the nurse implement? 1. Tell the client to take any routine antiseizure medication prior to the EEG. 2. Tell the client not to eat anything for eight (8) hours prior to the procedure. 3. Instruct the client to stay awake for 24 hours prior to the EEG. 4. Explain to the client that there will be some discomfort during the procedure.

ANSWER: 3. 1. Antiseizure drugs, tranquilizers, stimulants, and depressants are withheld before an EEG because they may alter the brain wave patterns. 2. Meals are not withheld because altered blood glucose level can cause changes in brain wave patterns. 3. The goal is for the client to have a seizure during the EEG. Sleep deprivation, hyperventilating, or flashing lights may induce a seizure. 4. Electrodes are placed on the client's scalp, but there are no electroshocks or any type of discomfort. TEST TAKING HINT: The test taker should highlight the words "diagnose a seizure disorder" in the stem and ask which answer option would possibly cause a seizure.

The client is admitted to the intensive care unit (ICU) experiencing status epilepticus. Which collaborative intervention should the nurse anticipate? 1. Assess the client's neurological status every hour. 2. Monitor the client's heart rhythm via telemetry. 3. Administer an anticonvulsant medication by intravenous push. 4. Prepare to administer a glucocorticosteroid orally.

ANSWER: 3. 1. Assessment is an independent nursing action, not a collaborative one. 2. All clients in the ICD will be placed on telemetry, which does not require an order by another health-care provider or collaboration with one. 3. Administering an anticonvulsant medication by intravenous push requires the nurse to have an order or confer with another member of the health-care team. 4. A glucocorticoid is a steroid and is not used to treat seizures. TEST TAKING HINT: The keyword in the stem of this question is the adjective "collaborative." The test taker would eliminate the options "1" and "2" because these do not require collaboration with another member of the health-care team and would eliminate option "4" because it is not used to treat seizures.

The nurse is assessing the client diagnosed with bacterial meningitis. Which clinical manifestations would support the diagnosis of bacterial meningitis? 1. Positive Babinski's sign and peripheral paresthesia. 2. Negative Chvostek's sign and facial tingling. 3. Positive Kernig's sign and nuchal rigidity. 4. Negative Trousseau's sign and nystagmus.

ANSWER: 3. 1. Babinski's sign is used to assess brainstem activity, and paresthesia is tingling, which is not a clinical manifestation of bacterial meningitis. 2. Chvostek's sign is used to assess for hypocalcemia, and facial tingling is a sign of hypocalcemia. It is not used to assess for bacterial meningitis. 3. A positive Kernig's sign (client unable to extend leg when lying flat) and nuchal rigidity (stiff neck) are signs of bacterial meningitis, occurring because the meninges surrounding the brain and spinal column are irritated. 4. Trousseau's sign is used to assess for hypocalcemia, and nystagmus is abnormal eye movement. Neither of these is a clinical manifestation of bacterial meningitis. TEST TAKING HINT: If two answer options test for the same thing (Trousseau's and Chvostek's signs), then the test taker can rule out these as possible answers because there cannot be two correct answers in the question, unless the question tells the test taker that it is a "select all that apply" question.

The client diagnosed with PD is being discharged on carbidopa/levodopa (Sinemet), an antiparkinsonian drug. Which statement is the scientific rationale for combining these medications? 1. There will be fewer side effects with this combination than with carbidopa alone. 2. Dopamine D requires the presence of both of these medications to work. 3. Carbidopa makes more levodopa available to the brain. 4. Carbidopa crosses the blood-brain barrier to treat Parkinson's disease.

ANSWER: 3. 1. Carbidopa is never given alone. Carbidopa is given together with levodopa to help the levodopa cross the blood-brain barrier. 2. Levodopa is a form of dopamine given orally to clients diagnosed with PD. 3. Carbidopa enhances the effects of levodopa by inhibiting decarboxylase in the periphery, thereby making more levodopa available to the central nervous system. Sinemet is the most effective treatment for PD. 4. Carbidopa does not cross the blood-brain barrier. TEST TAKING HINT: The nurse must be knowledgeable of the rationale for administering a medication for a specific disease.

The client is diagnosed with Creutzfeldt-Jakob disease. Which referral would be the most appropriate? 1. Alzheimer's Association. 2. Creutzfeldt-Jakob Disease Foundation. 3. Hospice care. 4. A neurosurgeon.

ANSWER: 3. 1. Creutzfeldt-Jakob disease is not Alzheimer's disease, although the presenting symptoms may mimic Alzheimer's disease. 2. There is no foundation for CreutzfeldtJakob disease, but if there were, the significant other would not be referred to this organization because the disease progresses so rapidly that the client would not get any benefit from the organization. 3. This disease is usually fatal within a year, and the symptoms progress rapidly to dementia. 4. The nurse does not refer the client to a neurosurgeon and the primary HCP would not refer to a neurosurgeon because there is no surgical or medical treatment for this disease.

Which potential pituitary complication should the nurse assess for in the client diagnosed with a traumatic brain injury (TBI)? 1. Diabetes mellitus type 2 (DM 2). 2. Seizure activity. 3. Syndrome of inappropriate antidiuretic hormone (SIADH). 4. Cushing's disease.

ANSWER: 3. 1. Diabetes mellitus type 2 is a pancreatic disease that has nothing to do with the pituitary gland or head injury. 2. Seizure activity is a possible complication of TBI, but it is not a pituitary complication. 3. The pituitary gland produces vasopressin, the antidiuretic hormone (ADH), and any injury that causes increased intracranial pressure will exert pressure on the pituitary gland and can cause the syndrome of inappropriate antidiuretic hormone (SIADH). 4. Cushing's disease is caused by an excess production of glucocorticoids and mineralocorticoids from the adrenal gland.

Which is a common cognitive problem associated with Parkinson's disease? 1. Emotional lability. 2. Depression. 3. Memory deficits. 4. Paranoia.

ANSWER: 3. 1. Emotional lability is a psychosocial problem, not a cognitive one. 2. Depression is a psychosocial problem. 3. Memory deficits are cognitive impairments. The client may also develop a dementia. 4. Paranoia is a psychosocial problem. TEST TAKING HINT: The test taker must know the definitions of common medical terms. "Cognitive" refers to mental capacity to function.

The spouse of a recently retired man tells the nurse, "All my husband does is sit around and watch television all day long. He is so irritable and moody. I don't want to be around him." Which action should the nurse implement? 1. Encourage the wife to leave the client alone. 2. Tell the wife that he is probably developing Alzheimer's disease. 3. Recommend that the client see an HCP for an antidepressant medication. 4. Instruct the wife to buy him some arts and crafts supplies.

ANSWER: 3. 1. If the wife could leave the client alone, she would not be sharing her concerns with the nurse. The nurse needs to address the wife's concerns as well as the husband's. 2. This is not the typical signs/symptoms of stage I Alzheimer's disease. 3. This behavior indicates the client is depressed and should be treated with antidepressants. A major lifestyle change has occurred, and he may need short-term medication therapy, depending on how the client adjusts to retirement. 4. The client may not want to participate in arts and crafts.

The client is admitted with a diagnosis of trigeminal neuralgia. Which assessment data would the nurse expect to find in this client? 1. Joint pain of the neck and jaw. 2. Unconscious grinding of the teeth during sleep. 3. Sudden severe unilateral facial pain. 4. Progressive loss of calcium in the nasal septum.

ANSWER: 3. 1. Joint pain is usually associated with some type of arthritis. 2. Unconscious grinding of the teeth during sleep is usually associated with temporomandibular joint (TMJ) disorder. 3. Trigeminal neuralgia affects the 5th cranial nerve and is characterized by paroxysms of pain in the area innervated by the three branches of the nerve. The unilateral nature of the pain is an important diagnostic characteristic. The disorder is also known as tic douloureux. 4. The nasal structure is not made up of bone.

The nurse has written a care plan for a client diagnosed with a brain tumor. Which is an important goal regarding self-care deficit? 1. The client will maintain body weight within two (2) pounds. 2. The client will execute an advance directive. 3. The client will be able to perform three (3) ADLs with assistance. 4. The client will verbalize feeling of loss by the end of the shift.

ANSWER: 3. 1. Maintaining weight is a nutritional goal. 2. Completing an advance directive is an end-of-life or psychosocial goal. 3. Performing activities of daily living is a goal for self-care deficit. 4. Verbalizing feelings is a psychosocial goal. TEST TAKING HINT: The test taker should read the stem of the question carefully. All of the goals could be appropriate for a client diagnosed with a brain tumor, but only one applies to self-care deficit.

The client diagnosed with breast cancer has developed metastasis to the brain. Which prophylactic measure should the nurse implement? 1. Institute aspiration precautions. 2. Refer the client to Reach to Recovery. 3. Initiate seizure precautions. 4. Teach the client about mastectomy care.

ANSWER: 3. 1. Nothing in the stem indicates the client has a problem with swallowing, so aspiration precautions are not needed. 2. Reach to Recovery is an American Cancer Society-sponsored program for clients with breast cancer, but it is not prophylactic. 3. The client diagnosed with metastatic lesions to the brain is at high risk for seizures. 4. Teaching about mastectomy care is not prophylactic, and the stem did not indicate whether the client had a mastectomy. TEST TAKING HINT: The test taker should not read into a question—for example, about mastectomy care. The test taker should be careful to read the descriptive words in the stem; in this case, "prophylactic" is the key to answering the question correctly

Which client should the nurse assess first after receiving the shift report? 1. The client diagnosed with a stroke who has right-sided paralysis. 2. The client diagnosed with meningitis who complains of photosensitivity. 3. The client with a brain tumor who has projectile vomiting. 4. The client with epilepsy who complains of tender gums.

ANSWER: 3. 1. Paralysis is an expected occurrence with a client who has had a stroke. 2. Photosensitivity is an expected sign of meningitis. 3. Projectile vomiting indicates that increased intracranial pressure is exerting pressure on the vomiting center of the brain. 4. Tender gums could be secondary to medication given for epilepsy. The client may need to see a dentist, but this client does not need to be assessed first.

The nurse and an unlicensed assistive personnel (UAP) are caring for a client with right-sided paralysis. Which action by the UAP requires the nurse to intervene? 1. The assistant places a gait belt around the client's waist prior to ambulating. 2. The assistant places the client on the back with the client's head to the side. 3. The assistant places a hand under the client's right axilla to move up in bed. 4. The assistant praises the client for attempting to perform ADLs independently

ANSWER: 3. 1. Placing a gait belt prior to ambulating is an appropriate action for safety and would not require the nurse to intervene. 2. Placing the client in a supine position with the head turned to the side is not a problem position, so the nurse does not need to intervene. 3. This action is inappropriate and would require intervention by the nurse because pulling on a flaccid shoulder joint could cause shoulder dislocation; the client should be pulled up by placing the arm underneath the back or using a lift sheet. 4. The client should be encouraged and praised for attempting to perform any activities independently, such as combing hair or brushing teeth. TEST TAKING HINT: This type of question has three answer options that do not require a nurse to intervene to correct a subordinate. Remember to read every possible answer option before deciding on a correct one.

The client with a closed head injury has clear fluid draining from the nose. Which action should the nurse implement first? 1. Notify the health-care provider immediately. 2. Prepare to administer an antihistamine. 3. Test the drainage for presence of glucose. 4. Place a 2 × 2 gauze under the nose to collect drainage

ANSWER: 3. 1. Prior to notifying the HCP, the nurse should always make sure that all the needed assessment information is available to discuss with the HCP. 2. With head injuries, any clear drainage may indicate a cerebrospinal fluid leak; the nurse should not assume the drainage is secondary to allergies and administer an antihistamine. 3. The presence of glucose in drainage from the nose or ears indicates cerebrospinal fluid, and the HCP should be notified immediately once this is determined. 4. This would be appropriate, but it is not the first intervention. The nurse must determine where the fluid is coming from. TEST TAKING HINT: The question is asking which intervention should be implemented first, and the nurse should always assess the situation before calling the HCP or taking an action.

The nurse is caring for several clients on a medical unit. Which client should the nurse assess first? 1. The client with ALS who is refusing to turn every two (2) hours. 2. The client with abdominal pain who is complaining of nausea. 3. The client with pneumonia who has a pulse oximeter reading of 90%. 4. The client who is complaining about not receiving any pain medication.

ANSWER: 3. 1. Refusing to turn needs to be addressed by the nurse, but it is not a priority over a lifethreatening condition. 2. Nausea needs to be assessed by the nurse, but it is not a priority over an oxygenation problem. 3. A pulse oximeter reading of less than 93% indicates that the client is experiencing hypoxemia, which is a life-threatening emergency. This client should be assessed first. 4. The nurse must address the client's complaints, but it is not a priority over a physiological problem. TEST TAKING HINT: The test taker should apply Maslow's hierarchy of needs, in which oxygenation is a priority. The nurse must know normal parameters for diagnostic tools and laboratory data.

The nurse is working with clients and their families regarding substance abuse. Which statement is the scientific rationale for teaching the children new coping mechanisms? 1. The child needs to realize that the parent will be changing behaviors. 2. The child will need to point out to the parent when the parent is not coping. 3. Children tend to mimic behaviors of parents when faced with similar situations. 4. Children need to feel like they are a part of the parent's recovery

ANSWER: 3. 1. The child will realize the changed behaviors when and if they happen. 2. This could cause problems between the parent and child. 3. Most coping behaviors are learned from parents and guardians. Children of substance abusers tend to cope with life situations by becoming substance abusers unless taught healthy coping mechanisms. 4. Children can be a part of the parent's recovery, but this is not the rationale for teaching new coping mechanisms. TEST TAKING HINT: Most parents do not like to be corrected by their child; this could eliminate option "2." The correct answer must address a reason for teaching new coping strategies.

The client who just had a three (3)-minute seizure has no apparent injuries and is oriented to name, place, and time but is very lethargic and just wants to sleep. Which intervention should the nurse implement? 1. Perform a complete neurological assessment. 2. Awaken the client every 30 minutes. 3. Turn the client to the side and allow the client to sleep. 4. Interview the client to find out what caused the seizure.

ANSWER: 3. 1. The client is exhausted from the seizure and should be allowed to sleep. 2. Awakening the client every 30 minutes possibly could induce another seizure as a result of sleep deprivation. 3. During the postictal (after-seizure) phase, the client is very tired and should be allowed to rest quietly; placing the client on the side will help prevent aspiration and maintain a patent airway. 4. The client must rest, and asking questions about the seizure will keep the client awake, which may induce another seizure as a result of sleep deprivation. TEST TAKING HINT: Options "1," "2," and "4" all have something to do with keeping the client awake. This might lead the test taker to choose the option that is different from the other three.

Which intervention should the nurse take with the client recently diagnosed with amyotrophic lateral sclerosis (Lou Gehrig's disease)? 1. Discuss a percutaneous gastrostomy tube. 2. Explain how a fistula is accessed. 3. Provide an advance directive. 4. Refer to a physical therapist for leg braces.

ANSWER: 3. 1. The client was diagnosed recently and at some point may need a percutaneous endoscopic gastrostomy (PEG) tube, but it is too early for this discussion. 2. A fistula is used for hemodialysis, and ALS does not cause renal dysfunction. 3. It is never too early to discuss advance directives with a client diagnosed with a terminal illness. 4. A client with ALS does not have leg braces as part of the therapeutic regimen.

Which statement by the female client indicates that the client understands factors that may precipitate seizure activity? 1. "It is all right for me to drink coffee for breakfast." 2. "My menstrual cycle will not affect my seizure disorder." 3. "I am going to take a class in stress management." 4. "I should wear dark glasses when I am out in the sun."

ANSWER: 3. 1. The client with a seizure disorder should avoid stimulants, such as caffeine. 2. The onset of menstruation can cause seizure activity in the female client. 3. Tension states, such as anxiety and frustration, induce seizures in some clients, so stress management may be helpful in preventing seizures. 4. Bright flickering lights, television viewing, and some other photic (light) stimulation may cause seizures but sunlight does not. Wearing dark glasses or covering one eye during potential seizure-stimulating activities may help prevent seizures. TEST TAKING HINT: Caffeine is a stimulant and its use is not recommended in many disease processes. Menstrual cycle changes are known to affect seizure disorders. Therefore, options "1" and "2" can be eliminated, as can option "4" because sunlight does not cause seizures.

A 78-year-old client is admitted to the emergency department (ED) with numbness and weakness of the left arm and slurred speech. Which nursing intervention is priority? 1. Prepare to administer recombinant tissue plasminogen activator (rt-PA). 2. Discuss the precipitating factors that caused the symptoms. 3. Schedule for a STAT computed tomography (CT) scan of the head. 4. Notify the speech pathologist for an emergency consult.

ANSWER: 3. 1. The drug rt-PA may be administered, but a cerebrovascular accident (CVA) must be verified by diagnostic tests prior to administering it. The drug rt-PA helps dissolve a blood clot, and it may be administered if an ischemic CVA is verified; rt-PA is not given if the client is experiencing a hemorrhagic stroke. 2. Teaching is important to help prevent another CVA, but it is not the priority intervention on admission to the emergency department. Slurred speech indicates problems that may interfere with teaching. 3. A CT scan will determine if the client is having a stroke or has a brain tumor or another neurological disorder. If a CVA is diagnosed, the CT scan can determine if it is a hemorrhagic or an ischemic accident and guide treatment. 4. The client may be referred for speech deficits and/or swallowing difficulty, but referrals are not the priority in the emergency department. TEST TAKING HINT: When "priority" is used in the stem, all answer options may be appropriate for the client situation, but only one option is the priority. The client must have a documented diagnosis before treatment is started.

The client diagnosed with Parkinson's disease is being discharged. Which statement made by the significant other indicates an understanding of the discharge instructions? 1. "All of my spouse's emotions will slow down now just like his body movements." 2. "My spouse may experience hallucinations until the medication starts working." 3. "I will schedule appointments late in the morning after his morning bath." 4. "It is fine if we don't follow a strict medication schedule on weekends."

ANSWER: 3. 1. The emotions of a person diagnosed with PD are labile. The client has rapid mood swings and is easily upset. 2. Hallucinations are a sign that the client is experiencing drug toxicity. 3. Scheduling appointments late in the morning gives the client a chance to complete ADLs without pressure and allows the medications time to give the best benefits. 4. The client should take the prescribed medications at the same time each day to provide a continuous drug level. TEST TAKING HINT: The test taker could eliminate option "2" because hallucinations are never an expected part of legal medication administration.

Which nursing task would be most appropriate for the nurse to delegate to the unlicensed assistive personnel? 1. Teach Credé's maneuver to the client needing to void. 2. Administer the tube feeding to the client who is quadriplegic. 3. Assist with bowel training by placing the client on the bedside commode. 4. Observe the client demonstrating selfcatheterization technique.

ANSWER: 3. 1. The nurse cannot delegate assessment or teaching. 2. Tube feedings should be treated as if they were medications, and this task cannot be delegated. 3. The assistant can place the client on the bedside commode as part of bowel training; the nurse is responsible for the training but can delegate this task. 4. Evaluating the client's ability to selfcatheterize must be done by the nurse. TEST TAKING HINT: Although each state has its own delegation rules, teaching, assessing, evaluating, and medication administration are nursing interventions that cannot be delegated to unlicensed assistive personnel.

The nurse observes a coworker acting erratically. The clients assigned to this coworker don't seem to get relief when pain medications are administered. Which action should the nurse implement? 1. Try to help the coworker by confronting the coworker with the nurse's suspicions. 2. Tell the coworker that the nurse will give all narcotic medications from now on. 3. Report the nurse's suspicions to the nurse's supervisor or the facility's peer review. 4. Do nothing until the nurse can prove the coworker has been using drugs.

ANSWER: 3. 1. The nurse is not the coworker's supervisor, and confronting the coworker about the suspicions could lead to problems if the nurse is not trained to deal with substance abusers. 2. This is circumventing the problem. The coworker will find another source of drugs if needed, and it is finding the coworker guilty without due process. 3. The coworker's supervisor or peer review committee should be aware of the nurse's suspicions so that the suspicions can be investigated. This is a client safety and care concern. 4. The nurse is obligated to report suspicious behavior to protect the clients the coworker is caring for. TEST TAKING HINT: The test taker can eliminate option "4" based on "do nothing." In this instance, direct confrontation is not recommended, but the nurse must do something—namely, report the suspicions to the supervisor or peer review.

The nurse arrives at the site of a one-car motorvehicle accident and stops to render aid. The driver of the car is unconscious. After stabilizing the client's cervical spine, which action should the nurse take next? 1. Carefully remove the driver from the car. 2. Assess the client's pupils for reaction. 3. Assess the client's airway. 4. Attempt to wake the client up by shaking him.

ANSWER: 3. 1. The nurse should stabilize the client's neck prior to removal from the car. 2. The nurse must stabilize the client's neck before doing any further assessment. Most nurses don't carry penlights, and the client's pupil reaction can be determined after stabilization. 3. The nurse must maintain a patent airway. Airway is the first step in resuscitation. 4. Shaking the patient could cause further damage, possibly leading to paralysis. TEST TAKING HINT: Remember that, in a question asking about which action should be taken first, all of the answers are interventions, but only one should be implemented first. There are very few "always" situations in the health-care profession, but in this situation, unless the client's car is on fire or under water, stabilizing the client's neck is always the priority, followed by airway

The nurse stops at the scene of a motor-vehicle accident and provides emergency first aid at the scene. Which law protects the nurse as a first responder? 1. The First Aid Law. 2. Ombudsman Act. 3. Good Samaritan Act. 4. First Responder Law.

ANSWER: 3. 1. There is no such law known as the First Aid Law. 2. The Ombudsman Act addresses many areas, such as advance directives, elderly advocacy, and several other areas. 3. The Good Samaritan Act protects nurses from judgment against them when in an emergency situation in which the nurse is not receiving compensation for the skills and expertise rendered. The nurse is held to a different standard than a layperson; the nurse must act as any reasonable and prudent nurse would in the same situation. 4. There is no such law known as the First Responder Law.

The client with a C6 SCI is admitted to the emergency department complaining of a severe pounding headache and has a BP of 180/110. Which intervention should the emergency department nurse implement? 1. Keep the client flat in bed. 2. Dim the lights in the room. 3. Assess for bladder distention. 4. Administer a narcotic analgesic

ANSWER: 3. 1. This action will not address the client's pounding headache and hypertension. 2. Dimming the lights will not help the client's condition. 3. This is an acute emergency caused by exaggerated autonomic responses to stimuli and only occurs after spinal shock has resolved in the client with a spinal cord injury above T6. The most common cause is a full bladder. 4. The nurse should always assess the client before administering medication. TEST TAKING HINT: The test taker should apply the nursing process when answering questions, and assessing the client comes first, before administering any type of medication.

The husband of a client who is an alcoholic tells the nurse, "I don't know what to do. I don't know how to deal with my wife's problem." Which response would be most appropriate by the nurse? 1. "It must be difficult. Maybe you should think about leaving." 2. "I think you should attend Alcoholics Anonymous." 3. "I think that Alanon might be very helpful for you." 4. "You should not enable your wife's alcoholism."

ANSWER: 3. 1. This advice might be appropriate at some point from a professional counselor but not from the nurse. 2. Alcoholics Anonymous is the support group that alcoholics—in this case, the wife, not the husband—should attend. 3. Alanon is the support group for significant others of alcoholics. Al-A-Teen is for teenage children of alcoholics. 4. This statement is making a judgment that is not given in the stem and is not applicable to all husbands of alcoholic wives

Which assessment data would indicate to the nurse that the client would be at risk for a hemorrhagic stroke? 1. A blood glucose level of 480 mg/dL. 2. A right-sided carotid bruit. 3. A blood pressure (BP) of 220/120 mm Hg. 4. The presence of bronchogenic carcinoma.

ANSWER: 3. 1. This glucose level is elevated and could predispose the client to ischemic neurological changes due to blood viscosity, but it is not a risk factor for a hemorrhagic stroke. 2. A carotid bruit predisposes the client to an embolic or ischemic stroke but not to a hemorrhagic stroke. 3. Uncontrolled hypertension is a risk factor for hemorrhagic stroke, which is a ruptured blood vessel inside the cranium. 4. Cancer is not a precursor to developing a hemorrhagic stroke. TEST TAKING HINT: Both options "1" and "2" are risk factors for an ischemic or embolic type of stroke. Knowing this, the test taker can rule out these options as incorrect.

The wife of the client diagnosed with septic meningitis asks the nurse, "I am so scared. What is meningitis?" Which statement would be the most appropriate response by the nurse? 1. "There is bleeding into his brain causing irritation of the meninges." 2. "A virus has infected the brain and meninges, causing inflammation." 3. "It is a bacterial infection of the tissues that cover the brain and spinal cord." 4. "It is an inflammation of the brain parenchyma caused by a mosquito bite."

ANSWER: 3. 1. This is a definition of aseptic meningitis, which refers to irritated meninges from viral or noninfectious sources. 2. This is another example of aseptic meningitis, which refers to irritated meninges from viral or noninfectious sources. 3. Septic meningitis refers to meningitis caused by bacteria; the most common form of bacterial meningitis is caused by the Neisseria meningitides bacteria. 4. This is the explanation for encephalitis. TEST TAKING HINT: The nurse should explain the client's diagnosis in layperson's terms when the stem is identifying the significant other as asking the question. Be sure to notice that the adjective "septic" is the key to answering this question, ruling out options "1" and "2."

The client has sustained a severe closed head injury and the neurosurgeon is determining if the client is "brain dead." Which data support that the client is brain dead? 1. When the client's head is turned to the right, the eyes turn to the right. 2. The electroencephalogram (EEG) has identifiable waveforms. 3. No eye activity is observed when the cold caloric test is performed. 4. The client assumes decorticate posturing when painful stimuli are applied.

ANSWER: 3. 1. This is an oculocephalic test (doll's eye movement) that determines brain activity. If the eyes move with the head, it means the brainstem is intact and there is no brain death. 2. Waveforms on the EEG indicate that there is brain activity. 3. The cold caloric test, also called the oculovestibular test, is a test used to determine if the brain is intact or dead. No eye activity indicates brain death. If the client's eyes moved, that would indicate that the brainstem is intact. 4. Decorticate posturing after painful stimuli are applied indicates that the brainstem is intact; flaccid paralysis is the worse neurological response when assessing a client with a head injury. TEST TAKING HINT: The test taker needs to know what the results of the cold caloric test signify—in this case, no eye activity indicates brain death.

The 80-year-old male client on an Alzheimer's unit is agitated and asking the nurse to get his father to come and see him. Which is the nurse's best response? 1. Tell the client his father is dead and cannot come to see him. 2. Give the client the phone and have him attempt to call his father. 3. Ask the client to talk about his father with the nurse. 4. Call the family so they can tell the client why his father cannot come to see him

ANSWER: 3. 1. This is presumably true but it is not an appropriate response to someone with a cognitive impairment. Rational thought processes do not apply. 2. The client would become increasingly agitated when unable to utilize the phone and/or be unable to reach the father. 3. The client is focused on his father. Letting the client talk about his father will allow him to focus on his father while distracting him from his impossible-tofulfill request. 4. This is called "passing the buck." A nurse on an Alzheimer's unit should be able to assess and intervene in this type of situation. TEST TAKING HINT: Arguing with a client with cognitive impairments only produces frustrations for the client and nurse. The nurse must remember the disease process and respond accordingly

Which rationale explains the transmission of the West Nile virus? 1. Transmission occurs through exchange of body fluids when sneezing and coughing. 2. Transmission occurs only through mosquito bites and not between humans. 3. Transmission can occur from human to human in blood products and breast milk. 4. Transmission occurs with direct contact from the maculopapular rash drainage.

ANSWER: 3. 1. Transmission does not occur through exposure with sneezed or coughed secretions. 2. The most common transmission of the West Nile virus to humans is through the bite of an infected mosquito. 3. The West Nile virus can be transmitted through breast milk, blood products, and organ transplants. This is a vector-borne disease. It is transmitted to mosquitoes that bite infected birds. The incubation period is around 15 days. 4. Maculopapular rashes do not drain. Draining is a characteristic of a vesicle. TEST TAKING HINT: The test taker should eliminate option "2" because of the absolute word "only."

The client is diagnosed with a brain abscess. Which sign/symptom is the most common? 1. Projectile vomiting. 2. Disoriented behavior. 3. Headaches, worse in the morning. 4. Petit mal seizure activity.

ANSWER: 3. 1. Vomiting may occur, but it is not projectile and it is not the most common. 2. Disoriented behavior may occur, but it is not the most common. 3. The most common and prevailing symptom of a brain abscess is a headache that is worse in the morning because of increased intracranial pressure as a result of lying flat (gravity). 4. The client with a brain abscess may have seizure activity, but it is usually tonic-clonic (grand mal) and it is not the most common.

The nurse is caring for a client with increased intracranial pressure (ICP) who has secretions pooled in the throat. Which intervention should the nurse implement first? 1. Set the ventilator to hyperventilate the client in preparation for suctioning. 2. Assess the client's lung sounds and check for peripheral cyanosis. 3. Turn the client to the side to allow the secretions to drain from the mouth. 4. Suction the client using the in-line suction, wait 30 seconds, and repeat

ANSWER: 3. 1. When suctioning a client on a ventilator it is good to hyperventilate the client before suctioning because suctioning the secretions would also suction the oxygen from the client. However, suctioning a client who has ICP increases the ICP. The nurse should attempt to remove the secretions without having to suction the client. 2. The secretions pooling in the back of the throat would not be assessed by listening to lung sounds or checking for peripheral perfusion. 3. Secretions can drain if the client is turned to the side unless the secretions are too heavy. The first action is to attempt to relieve the situation without increasing the ICP even further. 4. If suctioning is absolutely needed, then a minimum of 1 minute is needed between attempts to suction. TEST TAKING HINT: The test taker can eliminate options or decide between two of the options based on the fact that options "1" and "4" both involve suctioning the client. Either the nurse will perform suctioning or it is contraindicated. Assessment is the first step of the nursing process but the test taker must decide if the nurse is assessing the correct situation. Pooled oral secretion is not lung sounds.

The client is diagnosed with ALS. As the disease progresses, which intervention should the nurse implement? 1. Discuss the need to be placed in a long-term care facility. 2. Explain how to care for a sigmoid colostomy. 3. Assist the client to prepare an advance directive. 4. Teach the client how to use a motorized wheelchair.

ANSWER: 3. 1. With assistance, the client may be able to stay at home. Therefore, placement in a long-term care facility should not be discussed until the family can no longer care for the client in the home. 2. There is no indication that a client with ALS will need a sigmoid colostomy. 3. A client with ALS usually dies within five (5) years. Therefore, the nurse should offer the client the opportunity to determine how he/she wants to die. 4. ALS affects both upper and lower extremities and leads to a debilitating state, so the client will not be able to transfer into and operate a wheelchair. TEST TAKING HINT: The nurse should always help the client prepare for death in disease processes that are terminal and should discuss advance directives, which include both a durable power of attorney for health care and a living will.

The client diagnosed with amyotrophic lateral sclerosis (Lou Gehrig's disease) is prescribed medications that require intravenous access. The HCP has ordered a primary intravenous line at a keep-vein-open (KVO) rate at 25 mL/hr. The drop factor is 10 gtts/mL. At what rate should the nurse set the IV tubing? ___________

ANSWER: 4 gtts/min. The nurse must know the formula for regulating IV drips: the amount to infuse (25 mL/hr) times the drop factor (10 gtts/mL) divided by the minutes. Thus, 25 × 10 = 250 ÷ 60 = 4.11 or 4 gtts/min.

The nurse researcher is working with clients diagnosed with Parkinson's disease. Which is an example of an experimental therapy? 1. Stereotactic pallidotomy/thalamotomy. 2. Dopamine receptor agonist medication. 3. Physical therapy for muscle strengthening. 4. Fetal tissue transplantation.

ANSWER: 4. 1. A stereotactic pallidotomy and/or thalamotomy are surgeries that use CT or MRI scans to localize specific areas of the brain in which to produce lesions in groups of brain cells through electrical stimulation or thermocoagulation. These procedures are done when medication has failed to control tremors. 2. Dopamine receptor agonists are medications that activate the dopamine receptors in the striatum of the brain. 3. Physical therapy is a standard therapy used to improve the quality of life for clients diagnosed with PD. 4. Fetal tissue transplantation has shown some success in PD, but it is an experimental and highly controversial procedure. TEST TAKING HINT: The test taker should not overlook the adjective "experimental." This would eliminate at least option "3," physical therapy, and option "2," which refers to standard dopamine treatment, even if the test taker was not familiar with all of the procedures.

The client has been diagnosed with a brain tumor. Which presenting signs and symptoms help to localize the tumor position? 1. Widening pulse pressure and bounding pulse. 2. Diplopia and decreased visual acuity. 3. Bradykinesia and scanning speech. 4. Hemiparesis and personality changes.

ANSWER: 4. 1. A widening pulse pressure and bounding pulse indicate increased intracranial pressure but do not localize the tumor. 2. Diplopia and decreased visual pressure are symptoms indicating papilledema, a general symptom in the majority of all brain tumors. 3. Bradykinesia is slowed movement, a symptom of Parkinson's disease, and scanning speech is symptomatic of multiple sclerosis. 4. Hemiparesis would localize a tumor to a motor area of the brain, and personality changes localize a tumor to the frontal lobe. TEST TAKING HINT: The test taker could arrive at the correct answer if the test taker realized that specific regions of the brain control motor function and hemiparesis and that other regions are involved in personality changes.

The nurse caring for a client diagnosed with Parkinson's disease writes a problem of "impaired nutrition." Which nursing intervention would be included in the plan of care? 1. Consult the occupational therapist for adaptive appliances for eating. 2. Request a low-fat, low-sodium diet from the dietary department. 3. Provide three (3) meals per day that include nuts and whole-grain breads. 4. Offer six (6) meals per day with a soft consistency

ANSWER: 4. 1. Adaptive appliances will not help the client's shaking movements and are not used for clients with Parkinson's disease. 2. Clients with Parkinson's disease are placed on high-calorie, high-protein, soft or liquid diets. Supplemental feedings may also be ordered. If liquids are ordered because of difficulty chewing, then the liquids should be thickened to a honey or pudding consistency. 3. Nuts and whole-grain food would require extensive chewing before swallowing and would not be good for the client. Three large meals would get cold before the client can consume the meal, and one-half or more of the food would be wasted. 4. The client's energy levels will not sustain eating for long periods. Offering frequent and easy-to-chew (soft) meals of small proportions is the preferred dietary plan. TEST TAKING HINT: The correct answer for a nursing problem question must address the actual problem.

The nurse is planning care for a client experiencing agnosia secondary to a cerebrovascular accident. Which collaborative intervention will be included in the plan of care? 1. Observe the client swallowing for possible aspiration. 2. Position the client in a semi-Fowler's position when sleeping. 3. Place a suction setup at the client's bedside during meals. 4. Refer the client to an occupational therapist for evaluation.

ANSWER: 4. 1. Agnosia is the failure to recognize familiar objects; therefore, observing the client for possible aspiration is not appropriate. 2. A semi-Fowler's position is appropriate for sleeping, but agnosia is the failure to recognize familiar objects; therefore, this intervention is inappropriate. 3. Placing suction at the bedside will help if the client has dysphagia (difficulty swallowing), not agnosia, which is failure to recognize familiar objects. 4. A collaborative intervention is an intervention in which another health-care discipline—in this case, occupational therapy—is used in the care of the client. TEST TAKING HINT: Be sure to look at what the question is asking and see if the answer can be determined even if some terms are not understood. In this case, note that the question refers to "collaborative intervention." Only option "4" refers to collaboration with another discipline.

The nurse is planning the care for a client diagnosed with Parkinson's disease. Which would be a therapeutic goal of treatment for the disease process? 1. The client will experience periods of akinesia throughout the day. 2. The client will take the prescribed medications correctly. 3. The client will be able to enjoy a family outing with the spouse. 4. The client will be able to carry out activities of daily living.

ANSWER: 4. 1. Akinesia is lack of movement. The goal in treating PD is to maintain mobility. 2. This could be a goal for a problem of noncompliance with the treatment regimen but not a goal for treating the disease process. 3. This might be a goal for a psychosocial problem of social isolation. 4. The major goal of treating PD is to maintain the ability to function. Clients diagnosed with PD experience slow, jerky movements and have difficulty performing routine daily tasks. TEST TAKING HINT: The test taker should match the goal to the problem. A "therapeutic goal" is the key to answering this question.

The nurse is caring for clients on a medical unit. Which client would be most at risk for experiencing a stroke? 1. A 92-year-old client who is an alcoholic. 2. A 54-year-old client diagnosed with hepatitis. 3. A 60-year-old client who has a Greenfield filter. 4. A 68-year-old client with chronic atrial fibrillation.

ANSWER: 4. 1. An alcoholic is not at risk for having a stroke anymore than someone in the general population. 2. A client with hepatitis is not at risk for having a stroke anymore than someone in the general population. 3. A Greenfield filter is positioned in the inferior vena cava to prevent an embolism resulting from deep vein thrombosis; these filters prevent strokes and pulmonary emboli. 4. A client with atrial fibrillation is at high risk to have a stroke and is usually given oral anticoagulants to prevent a stroke.

Which behavior is a risk factor for developing and spreading bacterial meningitis? 1. An upper respiratory infection (URI). 2. Unprotected sexual intercourse. 3. Chronic alcohol consumption. 4. Use of tobacco products.

ANSWER: 4. 1. An upper respiratory infection (URI) is not a behavior. The question asked which behavior was a risk factor, so this option can be ruled out. However, a URI is a risk factor for developing and spreading bacterial meningitis because of increased droplet production. 2. Unprotected sexual intercourse is a risk factor for sexually transmitted diseases (STDs) but not for meningitis. 3. Chronic alcohol consumption can cause pancreatitis or hepatitis but not meningitis. 4. Tobacco use increases respiratory secretions and droplet production and thus is a risk factor for developing and spreading bacterial meningitis.

The client is diagnosed with arboviral encephalitis. Which priority intervention should the nurse implement? 1. Place the client in strict isolation. 2. Administer IV antibiotics. 3. Keep the client in the supine position. 4. Institute seizure precautions.

ANSWER: 4. 1. Arboviral encephalitis is a viral infection transmitted by mosquito bites, and isolation is not required. 2. Antibiotics are prescribed for bacterial infections, not viral infections. There is no antiviral medication to treat this disease. 3. Keeping the client supine would increase intracranial pressure, which is a concern when caring for clients with brain diseases. 4. Seizure precautions should be instituted because any inflammation of the brain tissue will put the client at risk for seizures.

The client is admitted to the medical floor with a diagnosis of closed head injury. Which nursing intervention has priority? 1. Assess neurological status. 2. Monitor pulse, respiration, and blood pressure. 3. Initiate an intravenous access. 4. Maintain an adequate airway.

ANSWER: 4. 1. Assessing the neurological status is important, but ensuring an airway is a priority over assessment. 2. Monitoring vital signs is important, but maintaining an adequate airway is higher priority. 3. Initiating an IV access is an intervention the nurse can implement, but it is not the priority intervention. 4. The most important nursing goal in the management of a client with a head injury is to establish and maintain an adequate airway. TEST TAKING HINT: If the question asks for a priority intervention, it means that all of the options would be appropriate for the client but only one intervention is priority. Always apply Maslow's hierarchy of needs—an adequate airway is first.

The nurse is enjoying a day at the lake and witnesses a water skier hit the boat ramp. The water skier is in the water not responding to verbal stimuli. The nurse is the first health-care provider to respond to the accident. Which intervention should be implemented first? 1. Assess the client's level of consciousness. 2. Organize onlookers to remove the client from the lake. 3. Perform a head-to-toe assessment to determine injuries. 4. Stabilize the client's cervical spine.

ANSWER: 4. 1. Assessment is important, but with clients with head injury the nurse must assume spinal cord injury until it is ruled out with x-ray; therefore, stabilizing the spinal cord is the priority. 2. Removing the client from the water is an appropriate intervention, but the nurse must assume spinal cord injury until it is ruled out with x-ray; therefore, stabilizing the spinal cord is the priority 3. Assessing the client for further injury is appropriate, but the first intervention is to stabilize the spine because the impact was strong enough to render the client unconsciousness. 4. The nurse should always assume that a client with traumatic head injury may have sustained spinal cord injury. Moving the client could further injure the spinal cord and cause paralysis; therefore, the nurse should stabilize the cervical spinal cord as best as possible prior to removing the client from the water. TEST TAKING HINT: When two possible answer options contain the same directive word—in this case, "assess"—the test taker can either rule out these two as incorrect or prioritize between the two assessment responses.

The nurse is discussing seizure prevention with a female client who was just diagnosed with epilepsy. Which statement indicates the client needs more teaching? 1. "I will take calcium supplements daily and drink milk." 2. "I will see my HCP to have my blood levels drawn regularly." 3. "I should not drink any type of alcohol while taking the medication." 4. "I am glad that my periods will not affect my epilepsy."

ANSWER: 4. 1. Because of bone loss associated with longterm use of anticonvulsants, the client should increase calcium intake to reduce the risk of osteoporosis. 2. Anticonvulsant medications have a narrow range of therapeutic value and the levels should be checked regularly. 3. Alcohol interferes with anticonvulsant medication and should be avoided. 4. Women with epilepsy note an increase in the frequency of seizures during menses. This is thought to be linked to the increase in sex hormones that alter the excitability of the neurons in the brain.

The nurse is assessing the client admitted with encephalitis. Which data require immediate nursing intervention? 1. The client has bilateral facial palsies. 2. The client has a recurrent temperature of 100.6°F. 3. The client has a decreased complaint of headache. 4. The client comments that the meal has no taste.

ANSWER: 4. 1. Bilateral facial palsies are a common initial sign and symptom of encephalitis. 2. Fever is usually one of the first signs and symptoms the client experiences. 3. A decrease in the client's headache does not indicate that the client's condition is becoming worse and thus does not warrant immediate intervention. 4. The absence of smell and taste indicates that the cranial nerves may be involved. The client's condition is becoming more serious. TEST TAKING HINT: This question requires the test taker to select an option that indicates the disease is progressing and the client is at risk. Option "3" indicates that the client is improving, and options "1" and "2" are common early manifestations of the disease. The only option that reflects cranial nerve involvement, a sign that the client's condition is becoming worse and requires immediate intervention, is option "4."

The nurse is admitting a client with the diagnosis of Parkinson's disease. Which assessment data support this diagnosis? 1. Crackles in the upper lung fields and jugular vein distention. 2. Muscle weakness in the upper extremities and ptosis. 3. Exaggerated arm swinging and scanning speech. 4. Masklike facies and a shuffling gait

ANSWER: 4. 1. Crackles and jugular vein distention indicate heart failure, not PD. 2. Upper extremity weakness and ptosis are clinical manifestations of myasthenia gravis. 3. The client has very little arm swing, and scanning speech is a clinical manifestation of multiple sclerosis. 4. Masklike facies and a shuffling gait are two clinical manifestations of PD. TEST TAKING HINT: Option "3" refers to arm swing and speech, both of which are affected by PD. The test taker needs to decide if the adjectives used to describe these activities—"exaggerated" and "scanning"— are appropriate. They are not, but masklike facies and shuffling gait are.

The intensive care unit nurse is admitting a client with a traumatic brain injury. Which health-care provider medication order would the nurse question? 1. Dexamethasone. 2. 0.9% NS. 3. Nicotine patch. 4. Morphine sulfate

ANSWER: 4. 1. Dexamethasone is a steroid medication and is the steroid of choice to reduce cerebral edema. 2. An IV of NS at a keep-open rate (25 mL per hour is a keep-open rate) would not be questioned. It is needed for emergency access. 3. A nicotine patch would be administered if the client is a smoker and unable to smoke during hospitalization. 4. A narcotic analgesic is contraindicated until it is known that the client is neurologically stable. Narcotics, especially intravenous, can mask signs and symptoms of deterioration of the client's status. TEST TAKING HINT: The test taker must know the actions of medications in order to administer them safely. Maslow's hierarchy of needs lists safety as a high priority. Many medications will only be listed on the NCLEX-RN examination using only the generic name. Dexamethasone's trade name is Decadron.

The client is diagnosed with WernickeKorsakoff syndrome as a result of chronic alcoholism. For which symptoms would the nurse assess? 1. Insomnia and anxiety. 2. Visual or auditory hallucinations. 3. Extreme tremors and agitation. 4. Ataxia and confabulation.

ANSWER: 4. 1. Insomnia and anxiety are symptoms of alcohol withdrawal, not Wernicke-Korsakoff syndrome. 2. Visual and auditory hallucinations are symptoms of delirium tremens. 3. Extreme tremors and agitation are symptoms of delirium tremens. 4. Ataxia, or lack of coordination, and confabulation, making up elaborate stories to explain lapses in memory, are both symptoms of Wernicke-Korsakoff syndrome. TEST TAKING HINT: The test taker can eliminate options "2" and "3" if the test taker knows the symptoms of delirium tremens.

The client diagnosed with a brain tumor has a diminished gag response and weakness on the left side of the body. Which intervention should the nurse implement? 1. Make the client NPO until seen by the health-care provider. 2. Position the client in low Fowler's position for all meals. 3. Place the client on a mechanically ground diet. 4. Teach the client to direct food and fluid toward the right side.

ANSWER: 4. 1. Making the client NPO will not help the client to swallow. 2. A low Fowler's position would make it easier for the client to aspirate. 3. The consistency of the food is not an issue; the client will have difficulty swallowing this food as well as regular-consistency food. 4. To decrease the risk of aspiration, the client should direct food to the unaffected side of the throat; this helps the client to be able to use the side of the throat that is functioning. TEST TAKING HINT: The test taker should try to visualize the position of the client in the bed. A mostly recumbent position (low Fowler's) would increase the chance of aspiration; thus option "2" should be eliminated.

The client diagnosed with a brain tumor was admitted to the intensive care unit with decorticate posturing. Which indicates that the client's condition is becoming worse? 1. The client has purposeful movement with painful stimuli. 2. The client has assumed adduction of the upper extremities. 3. The client is aimlessly thrashing in the bed. 4. The client has become flaccid and does not respond to stimuli.

ANSWER: 4. 1. Purposeful movement following painful stimuli would indicate an improvement in the client's condition. 2. Adducting the upper extremities while internally rotating the lower extremities is decorticate positioning and would indicate that the client's condition had not changed. 3. Aimless thrashing would indicate an improvement in the client's condition. 4. The most severe neurological impairment result is flaccidity and no response to stimuli. This indicates that the client's condition has worsened. TEST TAKING HINT: Neurological assessment includes assessing the client for levels of consciousness; the nurse must memorize the stages of neurological progression toward a coma and death.

The nurse is discussing psychosocial implications of Huntington's chorea with the adult child of a client diagnosed with the disease. Which psychosocial intervention should the nurse implement? 1. Refer the child for genetic counseling as soon as possible. 2. Teach the child to use a warming tray under the food during meals. 3. Discuss the importance of not abandoning the parent. 4. Allow the child to talk about the fear of getting the disease.

ANSWER: 4. 1. Referring the child is not a psychosocial intervention. The gene that determines if a client has Huntington's chorea has been identified, and genetic counseling could rule out or confirm that the client's child will develop Huntington's chorea. 2. This is an appropriate intervention, but it is not a psychosocial intervention. (Read the stem closely.) 3. This is placing a lot of responsibility on the child concerning the parent's debilitating, chronic, and devastating disease. The client may need to be in a long-term care facility, and the child should not feel guilty if this is necessary. 4. The child will develop this disease if he or she inherited the gene. It can be frightening for children to watch a parent progress through this disease and understand that they too may get it.

Which type of precautions should the nurse implement for the client diagnosed with septic meningitis? 1. Standard Precautions. 2. Airborne Precautions. 3. Contact Precautions. 4. Droplet Precautions.

ANSWER: 4. 1. Standard Precautions are mandated for all clients, but a client with septic meningitis will require more than the Standard Precautions. 2. Airborne Precautions are for contagious organisms that are spread on air currents and require the hospital personnel to wear an ultra-high filtration mask; these precautions would be applied for diseases such as tuberculosis. 3. Contact Precautions are for contagious organisms that are spread by blood and body fluids, such as those that occur with wounds or diarrhea. 4. Droplet Precautions are respiratory precautions used for organisms that have a limited span of transmission. Precautions include staying at least four (4) feet away from the client or wearing a standard isolation mask and gloves when coming in close contact with the client. Clients are in isolation for 24 to 48 hours after initiation of antibiotics. TEST TAKING HINT: The test taker must know the types of isolation precautions used for different diseases and note the adjective— "septic"—in the stem of the question.

The client is brought to the emergency department by the police for public disorderliness. The client reports feeling no pain and is unconcerned that the police have arrested him. The nurse notes the client has epistaxis and nasal congestion. Which substance should the nurse suspect the client has abused? 1. Marijuana. 2. Heroin. 3. Ecstasy. 4. Cocaine.

ANSWER: 4. 1. Symptoms of marijuana use are apathy, delayed time, and not wanting to eat. 2. Heroin symptoms include pupil changes and respiratory depression. 3. Ecstasy is a hallucinogen that is an "upper." 4. Disorderly behavior and the symptoms of epistaxis and nasal congestion would make the nurse suspect cocaine abuse.

The concept of intracranial regulation is identified for a client diagnosed with a brain tumor. Which intervention should the nurse include in the client's plan of care? 1. Tell the client to remain on bedrest. 2. Maintain the intravenous rate at 150 mL/hour. 3. Provide a soft, bland diet with three (3) snacks per day. 4. Place the client on seizure precautions.

ANSWER: 4. 1. The client can be up as he/she wishes but the nurse should assess this and determine if the client has the functional ability to be able to accomplish this without assistance. 2. A client with a brain tumor would be at risk for increased intracranial pressure (ICP). Fluids should be limited to decrease the amount of cerebrospinal fluids produced by the body. 3. The client can have the diet of choice. The tumor occupies space and increases the pressure on the brain, which can cause vomiting. This vomiting is not associated with the diet; it is caused by the pressure. 4. Clients with brain issues are at risk for electrical misfiring of the neurons, a seizure. The nurse should institute measures to protect the client during a seizure. TEST TAKING HINT: The test taker should remember that basic nursing care is appropriate for client protection. Maslow's hierarchy of needs lists safety in the second highest priority tier. Physiological needs that involve life-threatening or life-altering complications are the only things that are more important than safety.

The 29-year-old client is admitted to the medical floor diagnosed with meningitis. Which assessment by the nurse has priority? 1. Assess lung sounds. 2. Assess the six cardinal fields of gaze. 3. Assess apical pulse. 4. Assess level of consciousness

ANSWER: 4. 1. The client's lung sounds should be clear with meningitis, and nothing in the question stem indicates a comorbid condition. Therefore, assessing lung sounds is not a priority. 2. The client may experience photophobia and visual disturbances, but assessing the six fields of gaze will not affect the client's condition. 3. The client's cardiac status is not affected by meningitis. Therefore, the apical pulse would not be a priority. 4. Meningitis directly affects the client's brain. Therefore, assessing the neurological status would have priority for this client. TEST TAKING HINT: The test taker should apply a systemic approach to discerning the priority response. Maslow's hierarchy of needs would put option "1" as correct, but the disease process of meningitis does not include signs or symptoms of a respiratory component. The next highest priority would be the neurological component, and meningitis definitely is a neurological disease.

The client recently has been diagnosed with trigeminal neuralgia. Which intervention is most important for the nurse to implement with the client? 1. Assess the client's sense of smell and taste. 2. Teach the client how to care for the eyes. 3. Instruct the client to have carbamazepine (Tegretol) levels monitored regularly. 4. Assist the client to identify factors that trigger an attack.

ANSWER: 4. 1. The client's sense of smell and taste are not affected. 2. The cornea is at risk for abrasions because of the twitching, which causes irritation. Therefore, the nurse must teach the client how to care for the eye, but the most important intervention is to prevent the attacks. 3. Tegretol is the treatment of choice for trigeminal neuralgia, but it is not the most important intervention when the client is first diagnosed with this condition. 4. Stimulating specific areas of the face, called trigger zones, many initiate the onset of pain. Therefore, the nurse should help the client identify situations that exacerbate the condition, such as chewing gum, eating, brushing the teeth, or being exposed to a draft of cold air.

The nurse is assessing a client who is experiencing anosmia on a neurological floor. Which area should the nurse assess for cranial nerve I that is pertinent to anosmia? 1. Eye. 2. Lip. 3. Cheek. 4. Nose.

ANSWER: 4. 1. The eyes, indicated by A, would be assessed if checking cranial nerves II, III, IV, or VI. 2. The tongue located in the mouth, indicated by B, would be assessed if checking cranial nerves IX, X, or XII. 3. The cheek, indicated by C, would be assessed if checking for cranial nerve V, the trigeminal nerve. 4. Anosmia, the loss of the sense of smell, would require the nurse to assess for cranial nerve I, the olfactory nerve, indicated by D.

The nurse is performing a Glascow Coma Scale (GCS) assessment on a client with a problem with intracranial regulation. The client's GCS one (1) hour ago was scored at 10. Which datum indicates the client is improving? 1. The current GSC rating is 3. 2. The current GSC rating is 9. 3. The current GSC rating is 10. 4. The current GSC rating is 12.

ANSWER: 4. 1. The lowest ranking possible on the GCS is 3. The client would be considered brain dead, not improving. 2. The lower the numbers are on the GCS, the worse the client's functioning; this client is not improving. 3. This GCS rating indicates the client is the same as one (1) hour ago. 4. The GCS rating is going up, which means the client is improving. TEST TAKING HINT: The test taker can eliminate options "1" and "2" because they both present a lower score; both cannot be correct in a multiple-choice question and, here, both indicate a worsening of the client's condition. Likewise, option "3" is the same score as 1 hour ago, so the test taker should be careful to note this. Therefore, because only one answer can show an improving condition, the test taker can deduce that it is option "4."

The client is diagnosed with an SCI and is scheduled for a magnetic resonance imaging (MRI) scan. Which question would be most appropriate for the nurse to ask prior to taking the client to the diagnostic test? 1. "Do you have trouble hearing?" 2. "Are you allergic to any type of dairy products?" 3. "Have you eaten anything in the last eight (8) hours?" 4. "Are you uncomfortable in closed spaces?"

ANSWER: 4. 1. The machine is very loud and the technician will offer the client ear plugs, but hearing difficulty will not affect the MRI scan. 2. Allergies to dairy products will not affect the MRI scan. 3. The client does not need to be NPO for this procedure. 4. MRI scans are often done in a very confined space; many people who have claustrophobia must be medicated or even rescheduled for the procedure in an open MRI machine, which may be available if needed. TEST TAKING HINT: The nurse must be knowledgeable of diagnostic tests to prepare the client for the tests safely. The test taker must be realistic in determining answers—is there any test in which a hearing problem would make the diagnostic test contraindicated?

The nurse is preparing to administer acetaminophen (Tylenol) to a client diagnosed with a stroke who is complaining of a headache. Which intervention should the nurse implement first? 1. Administer the medication in pudding. 2. Check the client's armband. 3. Crush the tablet and dissolve in juice. 4. Have the client sip some water.

ANSWER: 4. 1. The medication can be administered in pudding, but it is not the first intervention. 2. The armband should be checked but not before determining if the client can swallow. 3. Tylenol comes in liquid form, and the nurse should request this before crushing a very bitter tablet. 4. Asking the client to sip some water assesses the client's ability to swallow, which is a priority when placing anything in the mouth of the client who has had a stroke

The client diagnosed with ALS is prescribed an antiglutamate, riluzole (Rilutek). Which instruction should the nurse discuss with the client? 1. Take the medication with food. 2. Do not eat green, leafy vegetables. 3. Use SPF 30 when going out in the sun. 4. Report any febrile illness.

ANSWER: 4. 1. The medication should be given without food at the same time each day. 2. This medication is not affected by green, leafy vegetables. (The anticoagulant warfarin [Coumadin] is a well-known medication that is affected by eating green, leafy vegetables.) 3. This medication is not affected by the sun. 4. The medication can cause blood dyscrasias. Therefore, the client is monitored for liver function, blood count, blood chemistries, and alkaline phosphatase. The client should report any febrile illness. This is the first medication developed to treat ALS. TEST TAKING HINT: Blood dyscrasias occur with many medications, and this might prompt the test taker to select option "4" as the correct option. Otherwise, the test taker must be knowledgeable of medication administration.

The nurse in a long-term care facility has noticed a change in the behavior of one of the clients. The client no longer participates in activities and prefers to stay in his room. Which intervention should the nurse implement first? 1. Insist that the client go to the dining room for meals. 2. Notify the family of the change in behavior. 3. Determine if the client wants another roommate. 4. Complete a Geriatric Depression Scale.

ANSWER: 4. 1. The nurse cannot insist that the client do anything. The nurse can encourage but, remember, this is the client's home. 2. The family may need to be notified, but the nurse should first assess what is happening that is causing this change in behavior. 3. There is nothing that indicates the client is unhappy with the roommate. In fact, the client wants to stay in the room, which does not indicate a need for a room change. 4. A change in behavior may indicate depression. The Geriatric Depression Scale measures satisfaction with life's accomplishments. The elderly should be in Erikson's generativity versus stagnation stage of life.

The male client is sitting in the chair and his entire body is rigid with his arms and legs contracting and relaxing. The client is not aware of what is going on and is making guttural sounds. Which action should the nurse implement first? 1. Push aside any furniture. 2. Place the client on his side. 3. Assess the client's vital signs. 4. Ease the client to the floor.

ANSWER: 4. 1. The nurse needs to protect the client from injury. Moving furniture would help ensure that the client would not hit something accidentally, but this is not done first. 2. This is done to help keep the airway patent, but it is not the first intervention in this specific situation. 3. Assessment is important but when the client is having a seizure, the nurse should not touch him. 4. The client should not remain in the chair during a seizure. He should be brought safely to the floor so that he will have room to move the extremities. TEST TAKING HINT: All of the answer options are possible interventions, so the test taker should go back to the stem of the question and note that the question asks which intervention has priority. "In the chair" is the key to this question because the nurse should always think about safety, and a client having a seizure is not safe in a chair

The 85-year-old client diagnosed with a stroke is complaining of a severe headache. Which intervention should the nurse implement first? 1. Administer a nonnarcotic analgesic. 2. Prepare for STAT magnetic resonance imaging (MRI). 3. Start an intravenous infusion with D5W at 100 mL/hr. 4. Complete a neurological assessment.

ANSWER: 4. 1. The nurse should not administer any medication to a client without first assessing the cause of the client's complaint or problem. 2. An MRI scan may be needed, but the nurse must determine the client's neurological status prior to diagnostic tests. 3. Starting an IV infusion is appropriate, but it is not the action the nurse should implement when assessing pain, and 100 mL/hr might be too high a rate for an 85-year-old client. 4. The nurse must complete a neurological assessment to help determine the cause of the headache before taking any further action. TEST TAKING HINT: The test taker should always apply the nursing process when answering questions. If the test taker narrows down the choices to two possible answer options, always select the assessment option as the first intervention.

The client has been diagnosed with a cerebrovascular accident (stroke). The client's wife is concerned about her husband's generalized weakness. Which home modification should the nurse suggest to the wife prior to discharge? 1. Obtain a rubber mat to place under the dinner plate. 2. Purchase a long-handled bath sponge for showering. 3. Purchase clothes with Velcro closure devices. 4. Obtain a raised toilet seat for the client's bathroom.

ANSWER: 4. 1. The rubber mat will stabilize the plate and prevent it from slipping away from the client learning to feed himself, but this does not address generalized weakness. 2. A long-handled bath sponge will assist the client when showering hard-to-reach areas, but it is not a home modification nor will it help with generalized weakness. 3. Clothes with Velcro closures will make dressing easier, but they do not constitute a home modification and do not address generalized weakness. 4. Raising the toilet seat is modifying the home and addresses the client's weakness in being able to sit down and get up without straining muscles or requiring lifting assistance from the wife. TEST TAKING HINT: The test taker must read the stem of the question carefully and note that the intervention must be one in which the home is modified in some way. This would eliminate three of the options, leaving the correct answer

The client is reporting neck pain, fever, and a headache. The nurse elicits a positive Kernig's sign. Which diagnostic test procedure should the nurse anticipate the HCP ordering to confirm a diagnosis? 1. A computed tomography (CT). 2. Blood cultures times two (2). 3. Electromyogram (EMG). 4. Lumbar puncture (LP).

ANSWER: 4. 1. The symptoms and a positive Kernig's sign suggest meningitis, but a CT scan is not diagnostic of meningitis. 2. Blood cultures determine septicemia or infections of the bloodstream, not meningitis. 3. An electromyogram (EMG) evaluates electrical conductivity through the muscle. 4. The client's symptoms, along with a positive Kernig's sign, should make the nurse suspect meningitis. The definitive diagnostic test for meningitis is a lumbar puncture to obtain cerebrospinal fluid for culture.

The client is being evaluated to rule out ALS. Which signs/symptoms would the nurse note to confirm the diagnosis? 1. Muscle atrophy and flaccidity. 2. Fatigue and malnutrition. 3. Slurred speech and dysphagia. 4. Weakness and paralysis.

ANSWER: 4. 1. These signs and symptoms occur during the course of ALS, but they are not early symptoms. 2. These signs and symptoms will occur as the disease progresses. 3. These are late signs/symptoms of ALS. 4. ALS results from the degeneration and demyelination of motor neurons in the spinal cord, which results in paralysis and weakness of the muscles. TEST TAKING HINT: This is an application question in which the test taker must know that ruling out of ALS would result in the answer being early signs/symptoms. The test taker could rule out option "1" because of atrophy, which is a long-term occurrence; rule out option "2" because these symptoms will occur as the disease progresses; and rule out option "3" because these are late signs/ symptoms.

The nurse is caring for the client diagnosed with West Nile virus. Which assessment data would require immediate intervention from the nurse? 1. The vital signs are documented as T 100.2°F, P 80, R 18, and BP 136/78. 2. The client complains of generalized body aches and pains. 3. Positive results are reported from the enzyme-linked immunosorbent assay (ELISA). 4. The client becomes lethargic and is difficult to arouse using verbal stimuli.

ANSWER: 4. 1. These vital signs are within normal ranges. The temperature is slightly elevated and may require an antipyretic but not as an immediate need. 2. This is a common complaint requiring medication but not immediately. 3. This test is used to differentiate West Nile virus from other types of encephalitis and would not require immediate intervention. Supportive care is given for West Nile virus. No definitive treatment is available. 4. These assessment data may indicate that the client's condition is deteriorating and require immediate intervention to prevent complications. TEST TAKING HINT: The word "immediate" means that the nurse must recognize and intervene before complications occur. The test taker should eliminate any option that contains normal assessment data.

Which collaborative intervention should the nurse implement when caring for the client with West Nile virus? 1. Complete neurovascular examinations every eight (8) hours. 2. Maintain accurate intake and output at the end of each shift. 3. Assess the client's symptoms to determine if there is improvement. 4. Administer intravenous fluids while assessing for overload.

ANSWER: 4. 1. This intervention is independent, not collaborative. 2. This is an independent nursing intervention. 3. Assessment is an independent nursing intervention. 4. Administering an IV fluid is collaborative because it requires an order from a health-care provider. It does, however, require the nurse to assess the rate, fluid, and site for complications. TEST TAKING HINT: When reading test questions, the test taker should pay attention to adjectives. In this question, the word "collaborative" makes all the options incorrect except option "4." Collaborative interventions require an order from a health-care provider, but the nurse uses judgment and intuition within the scope of practice.

The wife of the client diagnosed with chronic alcoholism tells the nurse, "I have to call his work just about every Monday to let them know he is ill or he will lose his job." Which would be the nurse's best response? 1. "I am sure that this must be hard for you. Tell me about your concerns." 2. "You are afraid he will lose his source of income." 3. "Why would you call in for your husband? Can't he do this?" 4. "Are you aware that when you do this you are enabling him?"

ANSWER: 4. 1. This is a therapeutic response. The spouse is not expressing feelings but is stating a fact. The nurse should address the problem. 2. This is a therapeutic response. The spouse is not expressing feelings but is stating a fact. The nurse should address the problem. 3. The spouse is not required to give an explanation to the nurse. 4. The spouse's behavior is enabling the client to continue to drink until he cannot function. TEST TAKING HINT: The stem of the question did not ask for a therapeutic response but did ask for the nurse's best response. The best response is to address the problem.

Which should be the nurse's first intervention with the client diagnosed with Bell's palsy? 1. Explain that this disorder will resolve within a month. 2. Tell the client to apply heat to the involved side of the face. 3. Encourage the client to eat a soft diet. 4. Teach the client to protect the affected eye from injury.

ANSWER: 4. 1. This is correct information, but it is not a priority when discussing Bell's palsy. 2. Heat will help promote comfort and increase blood flow to the muscles, but safety of the client's eye is a priority. 3. The client may have difficulty chewing on the affected side, so a soft diet should be encouraged, but it is not a priority teaching. 4. Teaching the client to protect the eye is a priority because the eye does not close completely and the blink reflex is diminished, making the eye vulnerable to injury. The client should wear an eye patch at night and wraparound sunglasses or goggles during the day; he or she may also need artificial tears.

The intensive care nurse is caring for a client with a T1 SCI. When the nurse elevates the head of the bed 30 degrees, the client complains of light-headedness and dizziness. The client's vital signs are T 99.2°F, P 98, R 24, and BP 84/40. Which action should the nurse implement? 1. Notify the health-care provider as soon as possible (ASAP). 2. Calm the client down by talking therapeutically. 3. Increase the IV rate by 50 mL/hour. 4. Lower the head of the bed immediately.

ANSWER: 4. 1. This is not an emergency; therefore, the nurse should not notify the health-care provider. 2. A physiological change in the client requires more than a therapeutic conversation. 3. Increasing the IV rate will not address the cause of the problem. 4. For the first two (2) weeks after an SCI above T7, the blood pressure tends to be unstable and low; slight elevations of the head of the bed can cause profound hypotension; therefore, the nurse should lower the head of the bed immediately. TEST TAKING HINT: The test taker should notice that the only answer option that addresses the "bed" is the correct answer. This does not always help identify the correct answer, but it is a hint that should be used if the test taker has no idea what the correct answer is.

The student nurse asks the nurse, "Why do you ask the client to identify how many fingers you have up when the client hit the front of the head, not the back?" The nurse would base the response on which scientific rationale? 1. This is part of the routine neurological examination. 2. This is done to determine if the client has diplopia. 3. This assesses the amount of brain damage. 4. This is done to indicate if there is a rebound effect on the brain.

ANSWER: 4. 1. This is part of the neurological examination, but this is not the scientific rationale for why it is done. The nurse must understand what is being assessed to interpret the data. 2. Diplopia, double vision, is a sign of head injury, but it is not the scientific rationale. 3. The procedure does assess for brain damage, but this answer does not explain why. 4. When the client hits the front of the head, there is a rebound effect known as "coupcontrecoup" in which the brain hits the back of the skull. The occipital lobe is in the back of the head, and an injury to it may be manifested by seeing double.

The client comes to the clinic for treatment of a dog bite. Which intervention should the clinic nurse implement first? 1. Prepare the client for a series of rabies injections. 2. Notify the local animal control shelter. 3. Administer a tetanus toxoid in the deltoid. 4. Determine if the animal has had its vaccinations.

ANSWER: 4. 1. This may be needed if it is determined the dog has not had its shots or if the dog cannot be found, but it is not the first intervention. 2. This is an appropriate action if the client does not know who owns the dog so that the dog can be found and quarantined. 3. If the client has not had a tetanus booster in the last 10 years, one must be administered, but it is usually the last action taken before the client is discharged from the clinic. 4. This is a priority because if the dog has had its vaccinations, the client will not have to undergo a series of very painful injections. The nurse must obtain information about the dog, which is assessment of the situation.

The friend of an 18-year-old male client brings the client to the emergency department (ED). The client is unconscious and his breathing is slow and shallow. Which action should the nurse implement first? 1. Ask the friend what drugs the client has been taking. 2. Initiate an IV infusion at a keep-open rate. 3. Call for a ventilator to be brought to the ED. 4. Apply oxygen at 100% via nasal cannula.

ANSWER: 4. 1. This should be done so that appropriate care can be provided, but it is not a priority action. 2. This should be done before the client ceases breathing and a cardiac arrest follows, but it is not the first action. 3. This would be a good step to take to prepare for the worst-case scenario, but it can be done last among these answer options. 4. Applying oxygen would be the priority action for this client. The client's breathing is slow and shallow. The greater amount of inhaled oxygen, the better the client's prognosis. TEST TAKING HINT: When the test taker is deciding on a priority, some guidelines should be used. Maslow's hierarchy of needs places oxygen at the top of the priority list.

The nurse arrives at the scene of a motorvehicle accident and the car is leaking gasoline. The client is in the driver's seat of the car complaining of not being able to move the legs. Which actions should the nurse implement? List in order of priority. 1. Move the client safely out of the car. 2. Assess the client for other injuries. 3. Stabilize the client's neck. 4. Notify the emergency medical system. 5. Place the client in a functional anatomical position.

ANSWER: In order of priority: 3, 2, 1, 5, 4. 3. Stabilizing the client's neck is a priority action to prevent further injury to the client, and it must be done prior to moving the client. 2. The nurse should assess for any other injuries prior to moving the client from the vehicle. 1. Because the vehicle is leaking fuel and there is potential for an explosion or fire, the client should be moved to an area of safety. 5. Placing the client in a functional anatomical position is an attempt to prevent further spinal cord injury. 4. Because the vehicle is leaking fuel, the priority is to remove the client and then obtain emergency medical assistance.

The client diagnosed with a brain abscess is experiencing a tonic-clonic seizure. Which interventions should the nurse implement? Rank in order of performance. 1. Assess the client's mouth. 2. Loosen restrictive clothing. 3. Administer phenytoin IVP. 4. Turn the client to the side. 5. Protect the client's head from injury.

ANSWER: In order of priority: 4, 5, 2, 3, 1. 4. The client should be turned to the side to prevent the tongue from falling back into the throat and occluding the airway. (Padded tongue blades are NOT forced into the mouth because they can break teeth and cause aspiration of the teeth.) 5. The client's head should be protected from hitting the side rails or other objects. 2. Clothing should be loosened to prevent airway difficulties. 3. The medications to control the seizures should be administered to stop the seizure. 1. Assessment in this instance is last because of the crisis that is occurring. The nurse should assess the mouth to determine if the client bit the tongue or buccal mucosa during the seizure or if teeth were chipped or broken. TEST TAKING HINT: Rank order questions can be difficult to answer. The test taker should remember safety. Which intervention will keep the client safe the fastest? Also important is if in stress, do not assess: Perform an intervention

_____: A localized collection of pus caused by an inflammatory response to bacteria in tissues or organs.

Abscess

_____: Difficulty with math calculations; caused by brain injury or disease.

Acalculia

_____: Loss of the ability to write; caused by brain injury or disease.

Agraphia

_____: Slow or no movement, as seen in a patient with Parkinson disease.

Akinesia (bradykinesia)

_____: Awake, engaged, and responsive.

Alert

_____: Complete inability to understand written language; caused by brain injury or disease.

Alexia

_____: Loss of memory.

Amnesia

_____: A progressive and degenerative disease of the motor system that is characterized by atrophy of the hands, forearms, and legs and that results in paralysis and death. There is no known cause, no cure, no specific treatment, no standard pattern of progression, and no method of prevention. Also called Lou Gehrig's disease.

Amyotrophic lateral sclerosis (ALS)

_____: A permanent localized dilation of an artery (to at least 2 times its normal diameter) that forms when the middle layer (media) of the artery is weakened, stretching the inner (intima) and outer (adventitia) layers. As the artery widens, tension in the wall increases and further widening occurs, thus enlarging the aneurysm.

Aneurysm

Which are risk factors for stroke? (Select all that apply). A) High blood pressure. B) Previous stroke or transient ischemic attack (TIA). C) Smoking. D) Use of oral contraceptives. E) Female gender.

Answer: A, B, C, D. Common modifiable risk factors for developing a stroke include smoking and the use of oral contraceptives. Other risk factors include high blood pressure and history of a previous TIA.Gender is not a known risk factor for stroke; however, the female client is at risk for delayed recognition of early stroke symptoms.

The nurse prepares to assess a client with diabetes mellitus for sensory loss. Which equipment is the best choice for the nurse use to perform this assessment? A) Cotton-tipped applicator. B) Glucometer. C) Hammer. D) Safety pin.

Answer: A. A cotton-tipped applicator is the nurse's best choice to assess sensory loss on a client with diabetes mellitus. Sensory loss is assessed with any sharp or dull object, such as a cotton-tipped applicator. The client indicates whether the touch is sharp or dull. The soft and hard ends of the applicator would be interchanged at random so that the client does not anticipate the next type of sensation.A glucometer tests blood sugar. A hammer tests tendon reflexes. Although a safety pin could be used to test for sensory loss, a cotton-tipped applicator is safer in the event the client is taking anticoagulants.

The nurse is caring for a client who is scheduled to have a transcranial Doppler (TCD). What does this diagnostic test evaluate? A) Cerebral vasospasm. B) Cerebrospinal fluid. C) Evoked potentials. D) Intracranial pressure.

Answer: A. A transcranial Doppler (TCD) is used to evaluate cerebral vasospasm or narrowing of arteries. It is noninvasive.Cerebrospinal fluid is obtained and measured during a lumbar puncture (LP). Evoked potentials measure the electrical signals in the brain during an EEG. Intracranial pressure is a measurement of blood, brain tissue, and cerebral spinal fluid and is not measured by TCD.

A client newly diagnosed with Parkinson disease (PD) is being discharged. Which instruction is best for the nurse to provide to the client's spouse? A) Administer medications promptly on schedule to maintain therapeutic drug levels. B) Complete activities of daily living for the client. C) Provide high-fiber, high-carbohydrate foods. D) Speak loudly for better understanding.

Answer: A. Administering medications promptly on schedule is a correct statement.The best instruction the nurse can give to the spouse of a PD client about to be discharged is to give schedule medications promptly in order to keep drug levels therapeutic.

A client with severe muscle spasticity has been prescribed tizanidine (Zanaflex, Sirdalud). The nurse instructs the client about which adverse effect of tizanidine? A) Drowsiness. B) Hirsutism. C) Hypertension. D) Tachycardia.

Answer: A. Adverse effects of tizanidine include drowsiness and sedation. Tizanidine (Zanaflex, Sirdalud) is a centrally acting skeletal muscle relaxant.It does not cause hirsutism, hypertension, or tachycardia.

A client is scheduled for an electroencephalogram (EEG) in the morning. Which instruction does the nurse give the client? A) "Do not take any sedatives 12-24 hours before the test." B) "Please do not have anything to eat or drink after midnight." C) "You may bring some music to listen to for distraction." D) "You will need to have someone to drive you home."

Answer: A. Before an EEG, the client needs to be instructed not to use sedatives or stimulants for 12-24 hours prior to the test.A client would not fast prior to an EEG as hypoglycemia may alter results. Testing takes place in a quiet room, so music for distraction is not appropriate. Unless the EEG is for sleep disorder diagnosis, the client will not need to be driven home.

Which change in the cerebrospinal fluid (CSF) indicates to the nurse that a client may have bacterial meningitis? A) Cloudy, turbid CSF B) Decreased white blood cells C) Decreased protein D) Increased glucose

Answer: A. Cloudy, turbid CSF indicates to the nurse that the client may have bacterial meningitis.Clear fluid is a sign of viral meningitis. Increased white blood cells, increased protein, and decreased glucose are signs of bacterial meningitis.

A client will be receiving plasmapheresis for treatment of Guillain-Barre'syndrome (GBS). Which posttreatment test will the nurse anticipate to be ordered? A) Electrolyte panel. B) Electroencephalogram (EEG) C) Lumbar puncture. D) Urinalysis

Answer: A. For the client receiving plasmapheresis for treatment of GBS, the nurse expects that an electrolyte panel will be ordered. Electrolytes will be checked since citrate-induced hypocalcemia is a complication of plasmapheresis.An electroencephalogram evaluates brain waves and is useful in detecting seizure activity. It would not be beneficial in this situation. A lumbar puncture might have been performed as part of the diagnostic process initially but not as part of posttreatment. There is no role for a urinalysis after plasmapheresis.

The nurse is teaching a client and family about home care after a stroke. Which statement made by the client's spouse indicates a need for further teaching? A) "I should spend all my time with my husband in case I'm needed." B) "My husband may get depressed." C) "My husband must take his medicine every day to prevent another stroke." D) "The physical therapist will show us how to use the equipment so my husband can climb the stairs and get into and out of bed."

Answer: A. Further home care teaching is needed when the stroke client's wife says that "I need to spend all my time with my husband in case I'm needed." Although well intentioned, family members can start to feel socially isolated when caring for a loved one. The family may need to plan for regular respite care in a structured day-care respite program or through relief provided by a friend or neighbor.The life changes associated with stroke often cause a change in the client's self-esteem. The client who has had a stroke needs to maintain a regular medication regimen to help prevent another stroke. If it is determined necessary after a home assessment, the physical and occupational therapist will show the client and family how to use equipment so they are able to mobilize and function in the home setting.

The nurse is caring for a client with advanced Alzheimer's disease. Which communication technique is best to use with this client? A) Assuming that the client is not totally confused B) Providing the client with several options to choose from C) Waiting for the client to express a need D) Writing down instructions for the client

Answer: A. The best communication technique to use for a client with advanced Alzheimer's disease is to not assume that the client is totally confused and cannot understand what is being said.Choices need to be limited. Too many choices cause frustration and increased confusion in the client. Rather than waiting for the client to express a need, try to anticipate the client's needs and interpret nonverbal communication. Just writing down instructions may be confusing for the client. It is better to provide the client instructions with pictures, and put them in a highly visible place.

The nurse's friend fears that something is wrong with his grandmother, saying that she is becoming extremely forgetful and disoriented and is beginning to wander. What is the nurse's best response? A) "Have you taken her for a check-up?" B) "She has Alzheimer's disease." C) "That is a normal part of aging." D) "You should look into respite care."

Answer: A. The best response by the nurse to a friend whose grandmother is forgetful and wandering is to ask her friend if he/she has taken the grandmother for a check-up. The grandmother's symptoms could indicate possible Alzheimer's disease or some other physiologic imbalance, and she needs to be assessed further by the primary care provider.The nurse's role is not to diagnose Alzheimer's disease but to advocate for the friend's grandmother to be evaluated. Becoming extremely forgetful, disoriented, and wandering is not normal age related behavior. Respite care is for caregivers, not for clients.

Which client will the neurologic unit charge nurse assign to a registered nurse who has floated from the labor/delivery unit for the shift? A) Adult client who has just returned from having a cerebral arteriogram and needs vital sign checks every 15 minutes. B) Older adult client who was just admitted with a stroke and needs an admission assessment. C) Young adult client who has had a lumbar puncture and reports, "Light hurts my eyes." D) Middle-aged client who has a possible brain tumor and has questions about the scheduled magnetic resonance imaging.

Answer: A. The charge nurse would assign an RN with experience in labor and delivery to check vital signs and limbs on a client who just returned from a cerebral angiogram. This float nurse would also be able to recognize signs of bleeding.The older adult admitted with a stroke, the young adult post lumbar puncture, and the middle-aged client with a possible brain tumor all require a nurse with more experience with neurologic diagnoses and diagnostic procedures.

The spouse of the client with Alzheimer's disease is listening to the home health nurse explain the client's drug regimen. Which statement by the spouse indicates an understanding of the nurse's instruction? A) "Donepezil (Aricept) will treat the symptoms of Alzheimer's disease." B) "Memantine (Namenda) is indicated for treatment of early symptoms of Alzheimer's disease." C) "Rivastigmine (Exelon) is used to treat depression." D) "Sertraline (Zoloft) will treat the symptoms of Alzheimer's disease."

Answer: A. The comment that shows that the spouse understands the nurse's instructions is that Aricept will treat symptoms of Alzheimer's. Cholinesterase inhibitors (e.g., donepezil) are approved for the symptomatic treatment of Alzheimer's disease. This class of medication delays the destruction of acetylcholine (ACh) by the enzyme cholinesterase.Memantine (Namenda) is indicated for advanced Alzheimer's disease. Memantine blocks excess amounts of glutamate which can damage nerve cells. Rivastigmine (Exelon) is a cholinesterase inhibitor that is used to treat Alzheimer's symptoms. Selective serotonin reuptake inhibitors like sertraline (Zoloft) are antidepressants and may be used in Alzheimer's clients who develop depression.

The nursing instructor asks a nursing student to compare Bell's palsy and trigeminal neuralgia. Which statement by the nursing student is correct? A) "Difficulty chewing may occur in both disorders." B) "Both are disorders of the autonomic nervous system." C) "Facial twitching occurs in both disorders." D) "Both disorders are caused by the herpes simplex virus, which inflames and irritates cranial nerve V."

Answer: A. The correct statement about Bell's palsy and trigeminal neuralgia is that problems with chewing can happen in both disorders. Both Bell's palsy and trigeminal neuralgia can affect cranial nerve V, which affects facial expressions and chewing.Both Bell's Palsy and trigeminal neuralgia are disorders of the cranial nerves. Facial twitching can be a sign of trigeminal neuralgia, whereas Bell's palsy causes a unilateral facial paralysis. Bell's palsy is caused by the herpes simplex virus, unlike trigeminal neuralgia, which is thought to be caused by excessive firing of irritated nerve fibers in the trigeminal nerve.

Which statement correctly illustrates the commonality between Guillain-Barré syndrome (GBS) and myasthenia gravis (MG)? A) The client's respiratory status and muscle function are affected by both diseases. B) Both diseases are autoimmune diseases with ocular symptoms. C) Both diseases exhibit exacerbations and remissions of their signs and symptoms. D) Demyelination of neurons is a cause of both diseases.

Answer: A. The correct statement about the commonality between GBS and MG is that both diseases affect the respiratory and muscular system. Both GBS and MG affect clients' respiratory status and muscle function.Only MG is an autoimmune disease with ocular symptoms and is characterized by exacerbations and remissions, whereas GBS has three acute stages. GBS causes demyelination of the peripheral neurons.

The nurse is teaching a client about the risk factors of restless legs syndrome. Which statement by the client indicates a correct understanding of the nurse's instruction? A) "Cigarettes and alcohol must be avoided." B) "I need to exercise my legs before bedtime." C) "It is important to stay off my feet." D) "Over-the-counter drugs must not be taken."

Answer: A. The correct statement about the risks of restless legs syndrome is cigarettes and alcohol must be avoided. Clients with restless legs syndrome need to avoid as many risk factors as possible or make lifestyle modifications. Examples include avoiding caffeine and alcohol, quitting smoking, and losing weight.Clients with RLS need to be encouraged to exercise but not engage in strenuous activity within 2-3 hours before bedtime. Use of over-the-counter drugs is not contraindicated for clients with restless legs syndrome.

A client with Parkinson disease (PD) is being discharged home with his wife. To ensure success with the management plan, which discharge action is most effective? A) Involving the client and his wife in developing a plan of care B) Setting up visitations by a home health nurse C) Telling his wife what the client needs D) Writing up a detailed plan of care according to standards

Answer: A. The discharge plan most effective when discharging a client home with his spouse is to involve both the client and his wife in developing the plan of care. Involving the client and spouse in drawing up a plan of care is the best way to ensure success with the management plan.Home health nurse visitations are generally helpful but may not be needed for this client. The management plan must be collaborative and include not only the spouse but the client to ensure buy-in. Evidence-based guidelines would be utilized.

A client is admitted with bacterial meningitis. Which nursing intervention is the highest priority for this client? A) Assessing neurologic status at least every 2-4 hours B) Decreasing environmental stimuli C) Managing pain through drug and nondrug methods D) Strict monitoring of hourly intake and output

Answer: A. The highest priority nursing intervention for the newly admitted client with bacterial meningitis is to accurately monitor and record the client's neurologic status every 2-4 hours. The neurologic status, vital signs, and vascular status must be assessed at least every 4 hours or more often, if clinically indicated, to rapidly determine any deterioration in status.Decreasing environmental stimuli is helpful for the client with bacterial meningitis but is not the highest priority. Clients with bacterial meningitis report severe headaches requiring pain management which may be accomplished through both pharmacologic and nonpharmacologic methods. Assessing fluid balance while preventing overload is not the highest priority however intake and output must be monitored.

The wife of a client with Alzheimer's disease mentions to the home health nurse that, although she loves him, she is exhausted caring for her husband. What does the nurse do to alleviate caregiver stress? A) Arranges for respite care B) Provides positive reinforcement and support to the wife C) Restrains the client for a short time each day, to allow the wife to rest D) Teaches the client improved self-care

Answer: A. The home health nurse can help relieve caregiver stress for the wife caring for her husband with Alzheimer's disease by arranging for respite care for the wife. Respite care can give the wife some time to reenergize and will provide a social outlet for the client.Providing positive reinforcement and support is important but does not help provide a solution to the wife's situation. Restraints are almost never appropriate and are used only as a last resort. The client with Alzheimer's disease typically is unable to learn improved self-care.

The nurse is evaluating the collaborative care of a client with traumatic brain injury (TBI). What is the most important goal for this client? A) Achieving the highest level of functioning B) Increasing cerebral perfusion C) Preventing further injury D) Preventing skin breakdown

Answer: A. The most important nurse's goal for the client with TBI is to help him or her achieve the highest level of functioning possible.The nurse assesses cerebral perfusion, such as oxygenation status, but cannot increase cerebral perfusion. Prevention of injury from falls, infection, or further impairment of cerebral perfusion is part of a larger goal for this client. Prevention of skin breakdown is a goal for the care of any client.

A client presents to the clinic with a migraine and is lying in a darkened room with a wet cloth on the head after receiving treatment. In preparation for dismissal home, what does the nurse do next? A) Allow the client to remain undisturbed. B) Assess the client's vital signs. C) Remove the cloth because it can harbor microorganisms. D) Turn on the lights for a neurologic assessment.

Answer: A. The next action by the nurse is to allow the client to remain undisturbed. The client may be able to alleviate pain by lying down in a darkened room with a cool cloth on his or her forehead. If the client falls asleep, he or she would remain undisturbed until awakening.Assessing the client's vital signs, although important, will disturb the client unnecessarily. A cool cloth is helpful for the client with a migraine and does not present enough of a risk that it would be removed. Turning on the lights for a neurologic assessment is not appropriate because light can cause the migraine to worsen.

A client with trigeminal neuralgia is admitted for a percutaneous stereotactic rhizotomy in the morning. The client currently reports pain. What does the nurse do next? A) Administers pain medication as requested B) Ensures that the client has nothing by mouth (NPO) C) Ensures that the preoperative laboratory work is complete D) Performs a preoperative assessment

Answer: A. The next action the nurse needs to do is to give pain medication to the preoperative client with trigeminal neuralgia who is complaining of pain. Addressing the client's pain is the priority nursing intervention because pain is the main symptom of trigeminal neuralgia.After the client's pain has been addressed, the preoperative assessment can be completed, questions and concerns can be addressed, and any further testing can be completed. This client is not required to be NPO until after midnight.

The nurse is reviewing the history of a client who has been prescribed topiramate (Topamax) for prevention of migraines. The nurse plans to contact the primary care provider (PCP) if the client has which condition? A) Bipolar disorder. B) Diabetes mellitus. C) Glaucoma. D) Hypothyroidism.

Answer: A. The nurse contacts the PCP after reviewing the history of a client with bipolar disorder who has been prescribed topiramate. Cases of suicide have been associated with topiramate when it is used in larger doses of 400 mg daily, most often in clients with bipolar disorder.Topiramate is not contraindicated in clients with diabetes mellitus, glaucoma, or hypothyroidism.

A client with a traumatic brain injury from a motor vehicle crash is monitored for signs/symptoms of increased intracranial pressure (ICP). Which sign/symptoms does the nurse monitor for? A) Changes in breathing pattern. B) Dizziness. C) Increasing level of consciousness. D) Reactive pupils.

Answer: A. The nurse monitors for changes in breathing pattern. This may be indicative of increased intracranial pressure secondary to compression of areas of the brain responsible for respiratory control.Dizziness is a symptom of brain injury, not increased intracranial pressure. Increasing level of consciousness and reactive pupils are desired outcomes for this client.

A client is being evaluated for signs associated with myasthenic crisis or cholinergic crisis. Which symptoms lead the nurse to suspect that the client is experiencing a cholinergic crisis? A) Abdominal cramps, blurred vision, facial muscle twitching B) Bowel and bladder incontinence, pallor, cyanosis C) Increased pulse, anoxia, decreased urine output D) Restlessness, increased salivation and tearing, dyspnea

Answer: A. The nurse suspects a cholinergic crisis when the client experiences abdominal cramps, blurred vision, and facial muscle twitching. These are signs of an acute exacerbation of muscle weakness symptoms of cholinergic crisis caused by overmedication with cholinergic (anticholinesterase) drugs.Bowel and bladder incontinence, pallor, cyanosis, increased pulse, anoxia, and decreased urine output are symptoms indicating a myasthenic crisis. Restlessness, increased salivation and tearing, and dyspnea are symptoms indicating a mixed myasthenic-cholinergic crisis.

A client is admitted into the emergency department (ED) with frontal-temporal pain, preceded by a visual disturbance. The client is upset and thinks it is a stroke. What does the nurse suspect may be occurring? A) Classic migraine. B) Meningitis. C) Stroke. D) West Nile virus.

Answer: A. The nurse suspects that a classic migraine could be present when an ED client complains of frontal-temporal pain preceded by a visual disturbance. These symptoms are most typical of a classic migraine.Meningitis may present with a headache and visual disturbance but is usually accompanied by nuchal rigidity (neck stiffness) and fever. The symptoms of stroke will vary depending upon the area affected. Mild cases of West Nile virus may be asymptomatic or present with flu-like symptoms, whereas severe cases may lead to loss of consciousness and death.

A client's spouse expresses concern that the client, who has Guillain-Barré syndrome (GBS), is becoming very depressed and will not leave the house. What is the nurse's best response? A) "Contact the Guillain-Barré Syndrome Foundation International for resources. Here is their contact information." B) "Try inviting several people over so the client won't have to go out." C) "Let your spouse stay alone. Your spouse will get used to it." D) "This behavior is normal."

Answer: A. The nurse's best response to a client's spouse about the client with GBS being depressed is referring the client to the GBS Foundation for resources. The Guillain-Barré Syndrome Foundation International (www.gbs-cidp.org) provides resources and information for clients and their families. The Foundation may be able to help the spouse and family find local support groups to assist the family with the transition.Inviting one close friend over is appropriate, but more than one might overwhelm the client. Telling the spouse to let the client say alone and that the behavior is normal is not helpful and inappropriate. Although depression is expected initially, some action does need to be taken to prevent further deterioration.

A client with early-stage Alzheimer's disease is admitted to the surgical unit for a biopsy. Which client problem is the priority? A) Potential for injury related to chronic confusion and physical deficits B) Risk for reduced mobility related to progression of disability C) Potential for skin breakdown related to immobility and/or impaired nutritional status D) Lack of social contact related to personality and behavior changes

Answer: A. The priority client problem related to a client admitted to the surgical unit for biopsy is the potential for injury due to chronic confusion and physical deficits. The most important intervention for interdisciplinary care is safety. Chronic confusion and physical deficits place the client with Alzheimer's disease at high risk for injury.Reduced mobility, skin breakdown, and lack of social contact, although potential problems in this population, are more frequently observed in the long-term setting and not the top priority.

A client is being discharged to home with progressing stage I Alzheimer's disease. The family expresses concern to the nurse about caring for their parent. What is the priority for best continuity of care? A) Assigning a case manager B) Ensuring that all family questions are answered before discharge C) Providing a safe environment D) Referring the family to the Alzheimer's Association

Answer: A. The priority for the best continuity of care for a client about to be discharged with progressing Stage I Alzheimer's disease is to assign a case manager to the client and family. Whenever possible, the client and family need the services of a case manager who can assess their needs for health care resources and facilitate appropriate placement throughout the continuum of care. Continuity of care is important through all stages of Alzheimer's disease.Ensuring all questions are answered and providing a safe environment are necessary for family support. The Alzheimer's Association will also be able to help provide information and support to the family.

A client with myasthenia gravis (MG) is receiving cholinesterase inhibitor drugs to improve muscle strength. The nurse is educating the family about this therapy. Which statement by a family member indicates a correct understanding of the nurse's instruction? A) "I will call 911 if a sudden increase in weakness occurs." B) "I will increase the dose if a sudden increase in weakness occurs." C) "The medication must be taken with a large meal." D) "The medication must be taken on an empty stomach."

Answer: A. The statement about cholinesterase inhibitors that shows a correct understanding of the nurse's instructions is that the family member will call 911 if there is a sudden increase in weakness. A potential adverse effect of cholinesterase inhibitors is cholinergic crisis. Sudden increases in weakness and the inability to clear secretions, swallow, or breathe adequately indicate that the client is experiencing crisis. The family member must call 911 for emergency assistance.The dose of cholinesterase inhibitors would never be increased without provider supervision. The client needs to eat meals 45-60 minutes after taking cholinesterase inhibitors to avoid aspiration. Cholinesterase inhibitors must be taken with a small amount of food to help alleviate GI side effects.

A client has received preoperative teaching from the nurse for a microdiskectomy. Which statement by the client indicates a correct understanding of the nurse's instruction? A) "I can go home the day of the procedure." B) "I can go home 48 hours after the procedure." C) "I'll have a drain in place after the procedure." D) "I'll need to wear special stockings after the procedure."

Answer: A. The statement that indicates the client correctly understands preoperative teaching of a microdiskectomy is "I can go home the day of the procedure." A microdiskectomy is considered minimally invasive surgery (MIS) and does not typically require an inclient hospital stay.The client who undergoes a minimally invasive surgery does not have to wait 48 hours after the procedure to return home, will not have a drain in place after the procedure, and will not need to wear special stockings after the procedure. These steps are used in the case of traditional open laminectomy, not MIS.

The nurse encourages a ventilated client with advanced Guillain-Barré syndrome (GBS) to communicate by which simple technique? A) Blinking for "yes" or "no" B) Moving lips to speak C) Using sign language D) Using a laptop to write

Answer: A. To communicate, a ventilated client with advanced GBS needs to blink for "yes" or "no." A simple technique involving eye blinking or moving a finger to indicate "yes" and "no" is the best way for the ventilated client with GBS to communicate.Moving the lips is difficult to do around an endotracheal tube and is exhausting for the client. Sign language is very time-consuming to learn, unless the client and family already know it. Use of a laptop may prove too challenging for the client in advanced stages of GBS.

A client hospitalized for hypertension presses the call light and reports "feeling funny." When the nurse gets to the room, the client is slurring words and has right-sided weakness. What does the nurse do first? A) Assesses airway, breathing, and circulation B) Calls the provider C) Performs a neurologic check D) Assists the client to a sitting position

Answer: A. When a client reports "feeling funny" and then starts slurring words and has right-sided weakness, the nurse must first assess for airway, breathing, and circulation. The priority is assessment of the "ABCs"—airway, breathing, and circulation.Calling the Rapid Response Team, not the provider, after assessing ABCs would be appropriate. The first 10 minutes after onset of symptoms is crucial. A neurologic check will be performed rapidly but is not the top priority. The client would be placed in bed, easily accessible for healthcare providers to assess and begin treatment. This does not need to be a seated position.

The nurse is performing a neurologic assessment on an 81-year-old client. Which physiologic change does the nurse expect to find because of the client's age? A) Decreased coordination B) Increased sleeping during the night C) Increased touch sensation D) Nightly confusion

Answer: A. When performing a neurologic assessment on an elderly client, the nurse expects to find decreased coordination. Older adults experience decreased coordination as a result of the aging process.Older adults frequently go to bed earlier and arise earlier than younger adults. Sensation to touch is decreased not increased. Nightly confusion, sometimes referred to as "sundowning," is not an expected change with all older adults.

The nurse is caring for a client diagnosed with partial seizures after encephalitis, who is to receive carbamazepine (Tegretol). The nurse plans to monitor the client for which adverse effects? (Select all that apply). A) Alopecia. B) Headaches. C) Dizziness. D) Diplopia. E) Increased blood glucose.

Answer: B, C, D. Adverse effects the nurse must monitor for in a client taking carbamazepine for partial seizures after encephalitis include: headaches, dizziness, and diplopia. Carbamazepine affects the central nervous system, although it's mechanism of action is unclear.Carbamazepine does not cause alopecia and does not increase blood glucose. Divalproex (Depakote) and valproic acid (Depakene) may cause alopecia.

A client has Guillain-Barré syndrome. Which interdisciplinary health care team members does the nurse plan to collaborate with to help prevent pressure ulcers related to immobility in this client? (select all that apply) A) Certified hospital chaplain B) Family members C) Dietitian D) Occupational therapist (OT) E) Social worker

Answer: B, C, D. The nurse plans to collaborate with family members, the dietician, and OT to help prevent pressure ulcers in the client with GBS. Family members would help to develop interventions to prevent these ulcers, because the family will mostly likely be directly involved in the client's care. Malnutrition puts the client at greater risk for pressure ulcers, so the dietitian must be included as well. The OT can provide assistive devices that will help prevent ulcers.The certified hospital chaplain and the social worker can assist with providing additional psychosocial support but would not be involved with direct prevention of ulcers. The social worker would also assist with the discharge plan and reintegration into the community.

A client has been admitted with new-onset status epilepticus. Which seizure precautions does the nurse put in place? (Select all that apply). A) Bite block at the bedside B) Intravenous access (IV) C) Continuous sedation D) Suction equipment at the bedside E) Siderailes raised.

Answer: B, D, E. Seizure precautions the nurse institutes for an admitted client with new-onset status epilepticus include IV access, suctioning equipment at the bedside and raised siderails. IV access is needed to administer medications. Suctioning equipment must be available to suction secretions and facilitate an open airway during a seizure. Raised, padded siderails may be used to protect the client from falling out of bed during a seizure.Bite blocks or padded tongue blades would not be used because the client's jaw may clench, causing teeth to break and possibly obstructing the airway. Continuous sedation is a medical intervention and not a seizure precaution.

The nurse is performing a rapid neurologic assessment on a trauma client. Which assessment findings are normal? (Select all that apply). A) Decerebrate posturing. B) Glasgow Coma Score (GCS) 15. C) Lethargy. D) Minimal response to stimulation. E) Pupil constriction to light.

Answer: B, D. Normal rapid neurologic assessment findings include a GCS (Glasgow Coma Score) of 15 and pupil constriction to light. The GCS range is between 3 and 15. Pupil constriction is a function of cranial nerve III. The pupils would be equal in size and round and regular in shape and would react to light and accommodation (PERRLA).Decerebrate or decorticate posturing is not normal, as well as pinpoint or dilated and nonreactive pupils. Both of findings are a late sign of neurologic deterioration. In addition, minimal response to stimulation and increased lethargy are not normal findings.

The nurse has just received change-of-shift report about a group of clients on the neurosurgical unit. Which client does the nurse attend to first? A) Adult postoperative left craniotomy client whose hand grip is weaker on the right B) Middle-aged adult client who had a cerebral aneurysm clipping and is becoming increasingly confused C) Older adult client who had a carotid endarterectomy and is unable to state the day of the week D) Young adult client involved in a motor vehicle crash (MVC) who is yelling obscenities at the nursing staff

Answer: B. After a change-of-shift report, the neurosurgical nurse would first attend to the middle-aged client who had a clipping of a cerebral aneurysm and is now becoming increasingly confused. A change in level of consciousness is an early indication that central neurologic function has declined. The primary care provider must be notified immediately.The other clients are not the nurse's first priority. The young adult who is post-MVC does need to be assessed, but the client's behavior does not indicate a decline in neurologic function. The postoperative left craniotomy client and the older adult also need to be assessed, but these clients' neurologic assessment indicates better function.

The nurse has received report on a group of clients. Which client requires the nurse's attention first? A) Adult who is lethargic after a generalized tonic-clonic seizure B) Young adult who has experienced four tonic-clonic seizures within the past 30 minutes C) Middle-aged adult with absence seizures who is staring at a wall and does not respond to questions D) Older adult with a seizure disorder who has a temperature of 101.9° F (38.8° C)

Answer: B. After receiving report on a group of clients, the nurse first needs to attend to the young adult client who is experiencing repeated seizures over the course of 30 minutes. This client is in status epilepticus, which is a medical emergency and requires immediate intervention.The adult client who is lethargic and the middle-aged adult client with absence seizures do not require immediate attention. A fever of 101.9° F (38.8° C), although high, does not require immediate attention.

A client newly diagnosed with myasthenia gravis (MG) is being discharged, and the nurse is teaching about proper medication administration. Which statement by the client demonstrates a need for further teaching? A) "It is important to post my medicine schedule at home, so my family knows my schedule." B) "I can continue to take over-the-counter drugs like before." C) "An extra supply of medicine must be kept in my car." D) "Wearing a watch with an alarm will remind me to take my medicine."

Answer: B. Further teaching about medication administration is indicated when the client with MG says that he/she can still take over-the-counter drugs. Clients with MG must not take any over-the-counter medications without checking with their primary health care provider first.The client's medication schedule may be posted in the home for the benefit of family members. An extra supply of medication should be kept in the client's car or workplace to maintain therapeutic levels in case a dose was missed. The client may wear a watch with an alarm as a medication reminder to maintain therapeutic levels.

A client has been admitted with a diagnosis of stroke (brain attack). The nurse suspects that the client has had a right hemisphere stroke because the client exhibits which symptoms? A) Aphasia and cautiousness. B) Impulsiveness and smiling. C) Inability to discriminate words. D) Quick to anger and frustration.

Answer: B. Impulsiveness and smiling are signs and symptoms indicative of a right hemisphere stroke.Aphasia, cautiousness, the inability to discriminate words, quick to anger, and frustration are signs and symptoms indicative of a left hemisphere stroke.

An older client presents to the clinic after a ground level fall at home. What statement by the client indicates the need for more injury prevention education? A) "I always take my medicine as directed." B) "I only eat little snacks so I don't gain weight." C) "I will make sure I drink enough water." D) "I make sure to get as much sleep as I used to."

Answer: B. More fall injury prevention education is needed when the client says that he/she will only eat little snacks to prevent weight gain. The brain is sensitive to decreased glucose levels which can lead to falls. This is especially noted in older clients.Taking medication as directed, ensuring adequate hydration, nutrition, and sleep help promote nervous system health and decrease the risk for falls in the elderly.

A female client with newly diagnosed migraines is being discharged with a prescription for sumatriptan (Imitrex). Which comment by the client indicates an understanding of the nurse's discharge instructions? A) "Birth control is not needed while taking sumatriptan." B) "I must report any chest pain right away." C) "St. John's wort can also be taken to help my symptoms." D) "Sumatriptan can be taken as a last resort."

Answer: B. The client comment that shows that she understands the discharge instructions is that any chest pain must be reported right away. Chest pain must be reported immediately with the use of sumatriptan because triptans cause vasoconstriction.Remind the client to use contraception (birth control) while taking the drug because it may not be safe for women who are pregnant. Triptans would not be taken with selective serotonin reuptake inhibitors or St. John's wort, an herb used commonly for depression. Sumatriptan must be taken as soon as migraine symptoms appear.

The nurse is teaching a client, newly diagnosed with migraines, about trigger control. Which statement made by the client demonstrates good understanding of the teaching plan? A) "I can still eat Chinese food." B) "I must not miss meals." C) "It is okay to drink a few wine coolers." D) "I need to use fake sugar in my coffee."

Answer: B. The client understands the teaching plan about trigger control for migraines when the client states that he/she must not miss meals. Until triggers are identified, a headache diary would be considered. Missing meals is a trigger for many people suffering from migraines. The client must not skip any meals until the triggers are identified.Chinese food frequently contains monosodium glutamate. Monosodium glutamate-containing foods, alcohol, and artificial sweeteners are triggers for many people suffering from migraines and need to be eliminated until the triggers are identified.

The nurse is about to administer a contrast medium to the client undergoing diagnostic testing. Which question does the nurse first ask the client? A) "Are you in pain?" B) "Are you taking ibuprofen daily" C) "Are you wearing any metal?" D) "Do you know what this test is for?"

Answer: B. The first question the nurse asks is if the client uses Ibuprofen on a daily basis. Ibuprofen is an NSAID, and daily use may place the client's renal function at risk. The client would also be asked about allergies to contrast agents, daily use of Metformin, and any conditions that may compromise kidney function.Inquiring if a client is in pain is always part of nursing assessment but would not be the first question to ask. The nurse would use this opportunity for education to confirm the client knows the reason for the test and take this time to answer any questions. Diagnostic testing involving magnetic resonance imaging, not contrast medium, requires precautions around metal objects.

A client is being discharged home after treatment for a brain attack. What is the mnemonic that the nurse can teach the family and client to help recognize and act on another stroke? A) A-V-P-U. B) F-A-S-T. C) K-I-N-D. D) O-P-Q-R-S-T.

Answer: B. The mnemonic F-A-S-T is utilized to teach the client, family, and community how to recognize and respond to a stroke. The purpose is to observe the Face, Arms, Speech, and then Time of onset and knowing it's Time to call 9-1-1.A-V-P-U is the mnemonic for level of awareness (alert, verbal, painful, and unresponsive). K-I-N-D is a mnemonic for treatment of hyperkalemia (kayexalate, insulin, NaHCO3, diuretics). O-P-Q-R-S-T is a mnemonic for assessing pain (onset, provokes, quality, radiates, severity, time).

Which is the most effective way for a college student to minimize the risk for bacterial meningitis? A) Avoid large crowds. B) Get the meningococcal vaccine. C) Take a high dose vitamin C daily. D) Take prophylactic antibiotics.

Answer: B. The most effective way for a college student to minimize the risk for bacterial meningitis is to get the meningococcal vaccine. Individuals ages 16-21 years have the highest rates of meningococcal infection and need to be immunized against the virus.Avoiding large crowds is helpful, but is not practical for a college student. Taking a high dose of vitamin C every day does not minimize the risk of bacterial meningitis. However, maintaining a healthy lifestyle, with adequate sleep and nutrition, can improve immunity. Taking prophylactic antibiotics is inappropriate because it leads to antibiotic-resistant strains of microorganisms.

The nurse is reviewing the medication history of a client diagnosed with myasthenia gravis (MG) who has been prescribed a cholinesterase (ChE) inhibitor. The nurse contacts the primary health care provider (PHCP) if the client is taking which medication? A) Acetaminophen (Tylenol) B) Diazepam (Valium) C) Furosemide (Lasix) D) Ibuprofen (Motrin)

Answer: B. The nurse contacts the PHCP if the client with MG who has been prescribed a ChE is also taking diazepam. Diazepam (Valium) would be avoided because it may increase the client's weakness.Acetaminophen (Tylenol) is an analgesic and antipyretic. It does not interact with ChE inhibitors. Furosemide (Lasix) is a diuretic and does not interact with ChE inhibitors. Ibuprofen (Motrin) is a nonsteroidal analgesic and does not interact with ChE inhibitors.

Which task does the nurse plan to delegate to the unlicensed assistive personnel (UAP) caring for a group of clients in the neurosurgical unit? A) Assist the health care provider in performing a lumbar puncture on a confused client B) Attend to the care needs of a client who has had a transcranial Doppler study C) Educate a client about what to expect during an electroencephalogram (EEG) D) Prepare a client who is going to radiology for a cerebral arteriogram

Answer: B. The nurse delegates the UAP to care for the client who has had a transcranial Doppler study. Since transcranial Doppler studies are noninvasive and do not require any postprocedure monitoring or care the UAP can safely attend to this client.Assisting the primary care provider in performing a lumbar puncture and preparing a client for a cerebral arteriogram require assessments and interventions that would be done by licensed nursing staff. Client teaching would also be provided by licensed nursing staff.

The nurse admits a client with suspected Eaton-Lambert syndrome. The nurse anticipates that the primary health care provider (PHCP) will request which test to confirm the diagnosis? A) Doppler study. B) Electromyography (EMG) C) Magnetic resonance imaging (MRI) D) Tensilon test

Answer: B. The nurse expects the PHCP to request an electromyography for the client suspected of having Eaton-Lambert syndrome. EMG is used to confirm the diagnosis of Eaton-Lambert syndrome, which is a form of myasthenia gravis (MG) that is often seen with small cell carcinoma of the lung.Doppler study is used frequently in the diagnosis of vascular disorders. MRI is not used to confirm the diagnosis of Eaton-Lambert syndrome. The Tensilon test is used as a diagnostic test in MG, but it is not used to confirm the diagnosis of Eaton-Lambert syndrome even though it is a form of MG.

The nurse admits a client with suspected myasthenia gravis (MG). The nurse anticipates that the primary health care provider (PHCP) will request which medication to aid in the diagnosis of MG? A) Atropine. B) Edrophonium chloride (Tensilon) C) Methylprednisolone (Solu-Medrol) D) Ropinirole (Requip)

Answer: B. The nurse expects the PHCP to request edrophonium chloride for a newly admitted client suspected of having MG. Edrophonium chloride (Tensilon) and neostigmine bromide (Prostigmin) may be used for testing for MG. Tensilon is used most often because of its rapid onset and brief duration of action. This drug inhibits the breakdown of acetylcholine (ACh) at the postsynaptic membrane, which increases the availability of ACh for excitation of postsynaptic receptors.Atropine has parasympatholytic effects and is the antidote for edrophonium chloride. Methylprednisolone (Solu-Medrol) is a glucocorticoid that is used to treat inflammatory disorders. Ropinirole (Requip) is a dopamine agonist used in the treatment of restless leg syndrome (RLS).

The nurse has just received report on a group of clients. Which client does the nurse assess first? A) Client who was in a car accident and has a Glasgow Coma Scale score of 14 B) Client who had a cerebral arteriogram and has a cool, pale leg C) Client who has a headache after undergoing a lumbar puncture D) Client who has expressive aphasia after a left-sided stroke

Answer: B. The nurse first assesses the client with a cool, pale leg after an arteriogram. This assessment finding could indicate clot formation at the catheter insertion site and loss of blood flow to the extremity.The client with a GCS of 14, the client with a headache following a lumbar puncture, and the client with expressive aphasia need to be assessed as soon as possible.

A client recovering from a stroke reports double vision that is preventing the client from effectively completing activities of daily living. How does the nurse help the client compensate? A) Approaches the client on the affected side B) Covers the affected eye C) Encourages turning the head from side to side D) Places objects in the client's field of vision

Answer: B. The nurse helps the stroke client compensate with double vision by covering the affected eye. Covering the client's affected eye with a patch prevents diplopia.The client who is recovering from a stroke would always be approached on the unaffected side. The nurse may encourage side-to-side head turning for clients with hemianopsia (blindness in half of the visual field). Objects would be placed in the field of vision for the client with a decreased visual field.

The nurse is assessing a client with a traumatic brain injury after a skateboarding accident. Which sign/symptom is the nurse most concerned about? A) Amnesia. B) Asymmetric pupils. C) Headache. D) Head laceration.

Answer: B. The nurse is most concerned about asymmetric pupils in the client with traumatic brain injury. Asymmetric (uneven) pupils are treated as herniation of the brain from increased intracranial pressure (ICP) until proven otherwise. The nurse must report and document any changes in pupil size, shape, and reactivity to the primary health care provider immediately.Amnesia, a headache and a head laceration, can be signs of mild traumatic brain injuries and need to be investigated more thoroughly.

A client with a T6 spinal cord injury who is on the rehabilitation unit suddenly develops facial flushing and reports a severe headache. Blood pressure is elevated, and the heart rate is slow. Which action does the nurse take first? A) Check for fecal impaction. B) Help the client sit up. C) Insert a straight catheter. D) Loosen the client's clothing.

Answer: B. The nurse's first action for a T6 spinal cord injury client suddenly developing facial flushing and severe headache is to help the client sit up. The client is experiencing autonomic dysreflexia, which can produce severe and rapidly occurring hypertension. Getting the client to sit upright is the easiest and quickest action to take and has the most immediate chance of lowering blood pressure to the brain.Checking for fecal impaction, inserting a straight catheter, and loosening the clothing are important but will not immediately reduce blood pressure.

A client returns to the neurosurgical floor after undergoing an anterior cervical diskectomy and fusion (ACDF). What is the nurse's first action? A) Administer pain medication. B) Assess airway and breathing. C)Assist with ambulation. D) Check the client's ability to void.

Answer: B. The nurse's first action when a client returns to the neurosurgical floor after having an anterior cervical discectomy is to assess the airway and breathing. Assessment in the immediate postoperative period after an ACDF is maintaining an airway and ensuring that the client has no problem with breathing.Administration of pain medication, ambulation, and assessing the client's ability to void are important but are not the highest priority.

A client receiving propranolol (Inderal) as a preventative for migraine headaches is experiencing side effects after taking the drug. Which side effect is of greatest concern to the nurse? A) Dry mouth. B) Slow heart rate. C) Tingling feelings. D) Warm sensation

Answer: B. The side effect that is the greatest concern for a client taking propranolol for migraine headaches is a slow heart rate. Beta blockers such as propranolol (Inderal) may be prescribed as a preventive medication for migraines. Propranolol causes blood vessels to relax and improves blood flow although the exact mechanism of action in migraines is unclear. The client would be taught how to monitor his or her heart rate and appropriately report any deviations to the primary care provider.Dry mouth is typically associated with tricyclic antidepressants such as nortriptyline. Skin flushing, tingling feelings, and a warm sensation are common side effects with triptan medications and are not indications to avoid using this group of drugs. Nortriptyline may be used as a preventive medication. Triptans are utilized as abortive medications after a migraine begins.

A client has had a traumatic brain injury and is mechanically ventilated. Which technique does the nurse use to prevent increasing intracranial pressure (ICP)? A) Assessing for Grey Turner's sign B) Maintaining neutral head position C) Placing the client in the Trendelenburg position D) Suctioning the client frequently

Answer: B. To prevent ICP in a client with traumatic brain injury who is being mechanically ventilated, the nurse needs to maintain the patent's head in a neutral position. Maintaining the head in neutral alignments prevents obstruction of blood flow and is an important component of ICP.Grey Turner's sign is a bluish gray discoloration in the flank region caused by retroperitoneal hemorrhage. The head of the bed needs to be at 30 degrees. The Trendelenburg position will cause the client's ICP to increase. Although some suctioning is necessary, frequent suctioning would be avoided because it increases ICP.

Which client diagnosed with neurologic injury is typically at highest risk for depression? A) Older man with a mild stroke B) Older woman with a seizure C) Young man with a spinal cord injury D) Young woman with a minor closed head injury

Answer: C. A young man with a spinal cord injury is at highest risk for depression. Although each individual responds differently, young adults who experience a spinal cord injury and loss of independent movement are more likely to experience depression.Keeping in mind people's differences in personal experiences, the client with a mild stroke without long-term deficits, the client who had a seizure or the young woman who sustained a minor head injury are generally at a lower risk of depression.

Which cranial nerve allows a person to feel a light breeze on the face? A) I (olfactory). B) III (oculomotor). C) V (trigeminal). D) VII (facial).

Answer: C. Cranial nerve V (trigeminal) allows the person to feel a light breeze on the face. This nerve is responsible for sensation from the skin of the face and scalp and the mucous membranes of the mouth and nose.Cranial nerve I (olfactory) is responsible for smell. Cranial nerve III (oculomotor) is responsible for eye movement. Cranial nerve VII (facial) is responsible for pain and temperature from the ear area, deep sensations from the face, and taste from the anterior two thirds of the tongue.

The nurse is monitoring a postoperative craniotomy client with increased intracranial pressure (ICP). Which pharmacologic agent does the nurse expect to be requested to maintain the ICP within a specified range? A) Dexamethasone (Decadron) B) Hydrochlorothiazide (HydroDIURIL) C) Mannitol (Osmitrol) D) Phenytoin (Dilantin)

Answer: C. In a postoperative craniotomy client with ICP, the nurse expects Mannitol to be requested to keep the ICP within a certain range. Mannitol is an osmotic diuretic used specifically to treat cerebral edema.Glucocorticoids have no demonstrated benefit in reducing ICP. Hydrochlorothiazide is only a mild diuretic and is not beneficial in maintaining ICP. Dilantin is used to treat seizure activity caused by increased ICP.

A client has been diagnosed with Primary Progressive MS (PPMS) and the nurse is providing education at the clinic. What statement by the client indicates the need for more teaching? A) "I can alternate wearing my eye patch between eyes for double vision." B) "I should keep my home clutter free so I don't fall." C) "It's important I work out in the afternoon so my muscles are warmed up." D) "I always keep my medications in the same place."

Answer: C. More teaching is needed for the client with PPMS when the client says, "It's important I work out in the afternoon so my muscles are warmed up." Working out in the afternoon will increase body temperature and lead to fatigue. Fatigue is a key feature of MS. Working with a physical therapist to develop an appropriate exercise program tailored to the client's condition will be beneficial.If a client has diplopia, wearing an eye patch and alternating it between eyes every few hours may relieve the symptoms. Keeping the home organized and clutter free will decrease the risk of falls. Keeping medications and other important belongings in the same place and maintaining a routine may help with memory deficits that may occur with MS.

A client presents to the Emergency Department from an assisted living facility after a ground level fall with a head strike. The client has a Glasgow Coma Score (GCS) of 12, which is decreased for this client, and has projectile vomiting. What is the priority intervention for this client? A) Calling the Stroke Team B) Establishing an IV C) Positioning the client to prevent aspiration D) Preparing for thrombolytic administration

Answer: C. Positioning the client while maintaining cervical spine immobilization to prevent aspiration is the nurse's priority intervention. Maintaining a patent airway is essential especially since this client is vomiting.Calling the Stroke Team would not be necessary. Establishing an IV is important for this client but it is not the first priority. If this client was having a stroke, thrombolytics would be contraindicated because of the fall with head strike.

Which nursing intervention is best for preventing complications of immobility when caring for a client with spinal cord problems? A) Encouraging nutrition. B) Frequent ambulation. C) Regular turning and repositioning. D) Special pressure-relief devices.

Answer: C. Regular turning and repositioning are the best way to prevent complications of immobility in clients with spinal cord problems.A registered dietitian may be consulted to encourage nutrition to optimize diet for general health and to reduce osteoporosis. Frequent ambulation may not be possible for these clients. Use of special pressure-relief devices is important but is not the best way to prevent immobility complications.

A client is being discharged with paraplegia secondary to a motor vehicle crash and expresses concern over the ability to cope in the home setting after the injury. Which is the best resource for the nurse to provide for the client? A) Hospital library. B) Internet. C) National Spinal Cord Injury Association. D) Provider's office

Answer: C. The best resource for the nurse to provide is the National Spinal Cord Injury Association. The National Spinal Cord Injury Association will inform the client of support groups in the area and will assist in answering questions regarding adjustment in the home setting.The hospital library is not typically consumer-oriented. Most information available in the library is targeted to health care professionals. The Internet is not the best resource simply because of the unlimited volume of information available and its questionable quality. Although the provider's office may have information, the information may not be as comprehensive and current as other options.

A client in the emergency department (ED) has slurred speech, confusion, and visual problems and has been having intermittent episodes of worsening symptoms. The symptoms have a gradual onset. The client also has a history of hypertension and atherosclerosis. What does the nurse suspect that the client is probably experiencing? A) Embolic stroke. B) Hemorrhagic stroke. C) Thrombotic stroke. D) Transient ischemic attack.

Answer: C. The client's signs and symptoms fit the description of a thrombotic stroke due to its gradual onset.Signs and symptoms of embolic stroke have a sudden onset, unlike this client's symptoms. Hemorrhagic strokes more frequently present with sudden, severe headache. Intermittent episodes of slurred speech, confusion, and visual problems are transient ischemic attacks, which often are warning signs of an impending ischemic stroke.

The nurse is providing instructions to a client with a spinal cord injury about caring for the halo device. The nurse plans to include which instructions? A) "Avoid using a pillow under the head while sleeping." B) "Begin driving 1 week after discharge." C) "Keep straws available for drinking fluids." D) "Swimming is recommended to keep active."

Answer: C. The instructions the nurse include for a client with a halo device is to keep straws available for drinking fluids. The halo device makes it difficult to bring a cup or a glass to the mouth.The head would be supported with a small pillow when sleeping to prevent unnecessary pressure and discomfort. Driving must be avoided because vision is impaired with the device. Swimming must be avoided to prevent the risk for infection.

Which information is most important for the nurse to communicate to the primary care provider (PCP) about a client who is scheduled for CT angiography? A) Allergy to penicillin B) History of bacterial meningitis C) Poor skin turgor and dry mucous membranes D) The client's dose of metformin (Glucophage) held today

Answer: C. The most important information for the nurse to communicate to the PCP about a client scheduled for a CT angiography is the client with poor skin turgor and dry mucous membranes. This assessment indicates dehydration which places the client at risk for contrast induced nephropathy.Allergy to penicillin, history of bacterial meningitis, and withheld metformin will need to be reported as part of the client hand-off to Radiology (SAFETY).

The nurse is caring for a client in the emergency department (ED) whose spinal cord was injured at the level of C7 1 hour prior to arrival. Which assessment finding requires the most rapid action? A) After two fluid boluses, the client's systolic blood pressure remains 80 mm Hg. B) Cardiac monitor shows a sinus bradycardia at a rate of 50 beats/min. C) The client's chest moves very little with each respiration. D) The client demonstrates flaccid paralysis below the level of injury.

Answer: C. The most rapid action is needed for a spinal cord injury client injured one hour prior to arrival whose chest moves very little with each respiration. Airway and breathing are always of major concern in a spinal cord injury, especially in an injury near C3 to C5, where the spinal nerves control the diaphragm.Bradycardia and hypotension are indications neurogenic shock due to disruption of autonomic pathways. This will need to be addressed rapidly however airway and breathing are always the top priority. Flaccid paralysis below the level of the injury is to be expected.

The nurse is caring for a client with a spinal cord injury resulting from a diving accident, who has a halo fixator and an indwelling urinary catheter in place. The nurse notes that the blood pressure is elevated and that the client is reporting a severe headache. The nurse anticipates that the primary health care provider will prescribe which medication? A) Dopamine hydrochloride (Inotropin) B) Methylprednisolone (Solu-Medrol) C) Nifedipine (Procardia) D) Ziconotide (Prialt)

Answer: C. The nurse anticipates that the primary health care provider will prescribe nifedipine for a spinal cord injury client who has an elevated blood pressure and severe headache. This client is experiencing autonomic dysreflexia (AD). Nifedipine (Procardia), a calcium channel blocker, can be administered to treat AD and lower blood pressure. If AD is not treated, a hemorrhagic stroke can occur.Dopamine hydrochloride (Inotropin) is an inotropic agent used to treat severe hypotension. Methylprednisolone (Solu-Medrol) is a glucocorticoid and is not indicated because it may further increase blood pressure. Ziconotide (Prialt) is an N-type calcium channel blocker on those nerves that usually transmit pain signals to the brain.

A client arrives in the emergency department with new-onset ptosis, diplopia, and dysphagia. The nurse anticipates that the client will be tested for which neurologic disease? A) Bell's palsy. B) Guillain-Barre syndrome (GBS). C) Myasthenia gravis (MG). D) Trigeminal neuralgia

Answer: C. The nurse expects the client with these signs/symptoms will be tested for MG. Sudden-onset ptosis, diplopia, and dysphagia are classic signs/symptoms of MG. Laboratory studies and a cholinesterase inhibitor test (e.g., Tensilon challenge test) most likely will be done to confirm the diagnosis.Signs/symptoms of Bell's palsy include facial paralysis; the face appears masklike and sags. Signs/symptoms of GBS typically begin in the legs and spread to the arms and upper body. Trigeminal neuralgia is characterized by sharp, intense facial pain that is usually not associated with sensory or motor deficits.

The nurse is providing medication instructions for a client for whom phenytoin (Dilantin) has been ordered for treatment of epilepsy. The nurse instructs the client to avoid which beverage? A) Apple juice. B) Grape juice. C) Grapefruit juice. D) Prune juice.

Answer: C. The nurse instructs the client taking phenytoin for epilepsy to avoid taking grapefruit juice. Some citrus fruits and juices, like grapefruit juice, can interfere with the metabolism of phenytoin potentially leading to an increased blood level and toxicity.Apple, grape, and prune juices are not contraindicated for a client taking phenytoin (Dilantin).

A client on the neurosurgical floor who had a lumbar laminectomy is confused, agitated, and complaining of difficulty breathing. The client is normally alert and oriented. The nurse notices a pinpoint rash over the client's chest. What condition is the nurse concerned has occurred? A) Autonomic dysreflexia. B) CSF leak. C) Fat embolism syndrome D) Paralytic ileus

Answer: C. The nurse is concerned that fat embolism syndrome has occurred. Fat embolism syndrome (FES) is characterized by chest pain, dyspnea, anxiety, and mental status changes. Petechiae may develop around the neck, over the upper chest, buccal mucosa, and conjunctiva. This is an emergency. The nurse must notify the primary health care provider immediately.Autonomic dysreflexia is not associated with lumbar laminectomies. It is seen in spinal cord injuries. A cerebrospinal fluid (CSF) leak is a concern with laminectomy but would not present with these symptoms. Paralytic ileus may occur but is associated with abdominal pain and distention.

A client is considering treatments for a malignant brain tumor. Which statement by the client indicates a need for further instruction by the nurse? A) "A combination of treatments might be necessary." B) "In a craniotomy, holes are cut in the skull to access the tumor." C) "I can go home the day of my craniotomy." D) "The goal is to decrease tumor size and improve survival time."

Answer: C. The nurse knows that further instruction is needed when a client considering treatment for malignant brain tumor says, "I can go home the day of my craniotomy." Craniotomies are inclient procedures. The client will be admitted to critical care for monitoring after the procedure and may be mechanically ventilated for 24-48 hours postprocedure.Chemotherapy, radiation, and surgery are often used in conjunction with each other to treat malignancies. For a craniotomy, several burr holes are drilled into the skull, and a saw is used to remove a piece of bone (bone flap) to expose the tumor area. The goals of treatment of brain tumor are to decrease tumor size, improve quality of life, and improve survival time.

The nurse is assessing a client with a neurologic condition who is reporting difficulty chewing when eating. The nurse suspects that which cranial nerve has been affected? A) Abducens (CN VI). B) Facial (CN VII). C) Trigeminal (CN V). D) Trochlear (CN IV).

Answer: C. The nurse suspects that the trigeminal cranial nerve is affected when a client complains of difficulty chewing when eating. The trigeminal nerve affects the muscles of mastication.The abducens nerve affects eye movement via lateral rectus muscles. The facial nerve affects pain and temperature from the ear area, deep sensations in the face, and taste in the anterior two-thirds of the tongue. The trochlear nerve affects eye movement via superior oblique muscles.

In assessing a client with back pain, the nurse uses a paper clip bilaterally on each limb. What is the nurse assessing? A) Gait. B) Mobility. C) Sensation. D) Strength.

Answer: C. The nurse uses a paper clip bilaterally on each limb of the client with back pain to assess sensation. Both extremities may be checked for sensation by using a paper clip and a cotton ball for comparison of light and deep touch. The client may feel sensation in both limbs but may experience a stronger sensation on the unaffected side.Gait is assessed by having the client walk. Mobility is assessed by determining the client's ability to move on his/her own, turn or perform ADLs. Strength is measured by having the client perform bilateral grips.

The daughter of a client who has had a stroke asks the nurse for additional resources. What is the nurse's best response? A) "Call hospice." B) "Check the Internet." C) "The National Stroke Association has resources available." D) "The charge nurse at the desk has all of the information."

Answer: C. The nurse's best response about additional resources for stroke is the National Stroke Association. The National Stroke Association is a specific and reliable resource that can be recommended. Additional resources are frequently provided as part of the discharge teaching the nurse will provide.Hospice care is appropriate for clients who are terminally ill, not a client who has had a stroke necessarily. Sources on the Internet may be very broad and unreliable or lack evidence to support their recommendations. The role of the client's nurse is to advocate for the client and not to refer all questions to the charge nurse.

The home health nurse is checking in on a client with dementia and the client's spouse. The spouse confides to the nurse, "I am so tired and worn out." What is the nurse's best response? A) "Can't you take care of your spouse?" B) "Establishing goals and a daily plan can help." C) "Make sure you take some time off and take care of yourself too." D) "That's not a very nice thing to say."

Answer: C. The nurse's best response to the spouse of the client with dementia is to encourage the wife to take some time off to take care of herself. This response is supportive and reminds the spouse that he or she cannot care for the client when exhausted.Questioning the spouse's ability to provide care is not supportive and may offend the spouse. Establishing goals and a daily plan may be helpful to the situation but is not responding to the spouse's need. Reprimanding the spouse does not validate his or her feelings and does not allow the nurse to further explore the statement.

A client is admitted with an exacerbation of Guillain-Barré syndrome (GBS), presenting with dyspnea. Which intervention does the nurse perform first? A) Calls the Rapid Response Team (RRT) to intubate B) Instructs the client on how to cough effectively C) Raises the head of the bed to 45 degrees D) Suctions the client

Answer: C. The nurse's first action for a client with an exacerbation of GBS who now has dyspnea is to raise the head of the bed to 45 degrees. The head of the client's bed must be elevated to allow for increased lung expansion. This action helps improve the client's ability to breathe.Calling the RRT for intubation may be necessary if dyspnea is severe or oxygen saturation does not respond to oxygen therapy. Close monitoring of respiratory status is indicated because of the acute stages of GBS. Instructing the client on how to cough effectively is not the priority in this case. The client would be suctioned as needed but cautiously to avoid vagal stimulation.

The nurse is caring for a client postoperatively after an anterior cervical diskectomy and fusion. Which assessment finding is of greatest concern to the nurse? A) Neck pain is at a level 7 on a 0-to-10 scale. B) Serosanguineous fluid oozes onto the neck dressing. C) The client is reporting difficulty swallowing secretions. D) The client has numbness and tingling bilaterally down the arms.

Answer: C. The nursing assessment finding that is the greatest concern for a postoperative anterior cervical diskectomy client is the client reporting difficulty swallowing secretions. This may indicate swelling in the neck and the potential for compromise of the client's airway.Experiencing neck pain and numbness and tingling bilaterally down the arms are expected findings after this surgery. Serosanguineous fluid oozing onto the neck dressing is also a normal finding after this surgery.

A client has Parkinson's disease (PD). Which nursing intervention best protects the client from injury? A) Discouraging the client from activity B) Encouraging the client to watch the feet when walking C) Monitoring the client's sleep patterns D) Suggesting that the client obtain assistance in performing activities of daily living (ADLs)

Answer: C. The nursing intervention that best protects the PD client from injury is to monitor the client's sleep patterns. Clients with PD tend to not sleep well at night because of drug therapy and the disease itself. Some clients nap for short periods during the day and may not be aware that they have done so. This sleep misperception could put the client at risk for injury (e.g., falling asleep while driving).Active and passive range-of-motion exercises, muscle stretching, and activity are important to keep the client with PD mobile and flexible. The client with PD needs to avoid watching his or her feet when walking to prevent falls and would be encouraged to participate as much as possible in self-management, including ADLs. Occupational and physical therapists can provide training in ADLs and the use of adaptive devices, as needed, to facilitate independence.

The nurse is teaching a client newly diagnosed with multiple sclerosis (MS). Which statement by the client indicates a correct understanding of the pathophysiology of the disease? A) "I will die early." B) "I will have gradual deterioration with no healthy times." C) "Parts of my nervous system have plaques." D) "This was caused by getting too many x-rays as a child."

Answer: C. The statement that demonstrates that the newly diagnosed client with MS correctly understands the pathophysiology of the disease is "parts of my nervous system have plaques." MS is characterized by an inflammatory response that results in diffuse random or patchy areas of plaque in the white matter of the central nervous system.The client with MS has no decrease in life expectancy. Frequent times of remission are common in clients with MS. There is no known cause for MS.

A client admitted with cerebral edema suddenly begins to have a seizure while the nurse is in the room. What does the nurse do first? A) Administer phenytoin (Dilantin). B) Draw the client's blood. C) Establish an airway. D) Start an intravenous (IV) line.

Answer: C. When a client admitted with cerebral edema begins to have a seizure, the nurse must first establish an airway. The primary goal is to open and maintain an airway and then assess the client for the need of additional support during the seizure.Phenytoin (Dilantin) is administered to prevent the recurrence of seizures, not to treat a seizure already underway. Drawing blood or starting an IV is not the priority in this situation. Remember the ABCs during an emergency situation.

The nurse is administering the intake assessment for a newly admitted client with a history of seizures. The client suddenly begins to seize. What does the nurse do next? A) Documents the length and time of the seizure. B) Forces a tongue blade in the mouth. C) Positions the client on the side. D) Restrains the client.

Answer: C. When a newly admitted client with a history of seizures begins to seize, the nurse must turn the client on his/her side. Turning the client on the side during a generalized tonic-clonic or complex partial seizure is indicated because he or she may lose consciousness resulting in potential loss of a patent airway.Documenting the length and time of seizures is important, but not the priority intervention. Both forcing a tongue blade in the mouth and restraining the client can cause injury.

The nurse has just received report on a group of clients on the neurosurgical unit. Which client is the nurse's first priority? A) Client whose deep tendon reflexes have become hyperactive B) Client who displays plantar flexion when the bottom of the foot is stroked C) Client who consistently demonstrates decortication when stimulated D) Client whose Glasgow Coma Scale (GCS) has changed from 15 to 13.

Answer: D. After receiving report on a group of clients, the nurse's first priority is to assess the client whose GCS has changed from 15 to 13. A decrease of 2 or more points in the Glasgow Coma Scale total is clinically significant and indicates a major change in neurologic status. This finding must be reported immediately to the primary health care provider (PHCP).The client with hyperactive reflexes, the client displaying plantar flexion when the bottom of the foot is stroked, and the client with decortication upon stimulation will need to be assessed, but they do not require immediate attention.

The nurse is monitoring a client after supratentorial surgery. Which sign/symptom does the nurse report immediately to the provider? A) Periorbital edema B) Bilateral ecchymoses of both eyes C) Moderate amount of serosanguineous drainage on the head dressing D) Decorticate positioning

Answer: D. In a postoperative supratentorial client, the nurse must immediately report decorticate positioning to the provider. The major complications of supratentorial surgery are increased intracranial pressure from cerebral edema or hydrocephalus and hemorrhage. Decorticate positioning indicates damage to the pathway between the brain and the spinal cord.Periorbital edema and a small-to-moderate amount of serosanguineous drainage are expected after a craniotomy. Ecchymoses in the facial region, especially around the eyes, are expected after a craniotomy.

A client who has just undergone spinal surgery must be moved. How does the nurse plan to move this client? A) Getting the client up in a chair B) Keeping the client in the Trendelenburg position C) Lifting the client in unison with other health care personnel D) Log rolling the client

Answer: D. Log rolling the client who has undergone spinal surgery is the best way to keep the spine in alignment. The client who has undergone spinal surgery must remain straight and turned as a unit.The Trendelenburg position is not indicated for the client who has undergone spinal surgery, nor should the client be lifted or encouraged to get up in a chair.

A client with new-onset Bell's palsy is being dismissed from the hospital. Which statement made by the client demonstrates a need for further teaching by the nurse? A) "I'll need artificial tears at least four times a day." B) "I will eat a soft diet." C) "My eye must be taped or patched at bedtime." D) "Narcotics will be needed for pain relief."

Answer: D. Mild analgesics, not narcotics, are used for pain associated with Bell's palsy.Further teaching about Bell's palsy is needed when the client says that narcotics are needed for pain. Artificial tears need to be taken at least 4 times a day and taping the affected eye at night protects the cornea from drying out and potentially ulcerating. Drying out of the eyes occurs because of the eye's inability to close. Mastication is often impaired with Bell's palsy, so soft foods are indicated.

A client with myasthenia gravis is admitted with generalized fatigue, a weak voice, and dysphagia. Which client problem has the highest priority? A) Inability to tolerate everyday activities related to severe fatigue B) Inability to communicate verbally related to vocal weakness C) Inability to care for self-related to muscle weakness D) Potential for aspiration related to difficulty with swallowing

Answer: D. The client problem that has the highest priority for a client with MG is the risk for aspiration due to difficulty swallowing. The potential for aspiration is the highest priority client problem because the client's ability to maintain airway patency is compromised.Although important, an inability to tolerate everyday activities, an inability to communicate verbally related to vocal weakness, and an inability to care for oneself related to muscle weakness are not the nurse's highest priority.

A client with dementia and Alzheimer's disease is discharged to home. The client's daughter says, "He wanders so much, I am afraid he'll slip away from me." What resource does the nurse suggest? A) Alzheimer's Wandering Association B) Lost Family Members Tracking Association C) National Alzheimer's Group D) Safe Return Program

Answer: D. The discharge nurse suggests the Safe Return Program to the daughter of a client who wanders at home. The Safe Return Program, a national, government-funded program of the Alzheimer's Association assists in the identification and safe, timely return of those with dementia who wander off and become lost.The Alzheimer's Wandering Association, National Alzheimer's Group, and Lost Family Members Tracking Association do not exist.

A client is admitted with a stroke (brain attack). Which tool does the nurse use to facilitate a focused neurologic assessment of the client? A) Glasgow Coma Score (GCS) B) Intracranial pressure monitor C) Mini-Mental State Examination (MMSE; mini-mental status examination) D) National Institutes of Health Stroke Scale (NIHSS)

Answer: D. The nurse uses the NIHSS tool to perform a focused neurologic assessment. Health care providers and nurses at designated stroke centers use a specialized stroke scale such as the NIHSS to assess clients.The Glasgow Coma Score (GCS) provides a non-specific indication of level of consciousness. An intracranial pressure monitor would be requested by the health care specialist if signs and symptoms indicated increased intracranial pressure. The MMSE is used primarily to differentiate among dementia, psychosis, and affective disorders.

A client is eating a soft diet while recovering from a stroke. The client reports food accumulating in the cheek of the affected side. What is the nurse's best response? A) "Next time you eat, try lifting your chin when you swallow." B) "Let's advance your diet to solid food." C) "Let's see if the dietitian can help." D) "Let's see if the speech-language pathologist can help."

Answer: D. The nurse's best response about food gathering in the cheek of a stroke client is to see what the speech pathologist says may help. The speech-language pathologist identifies strategies to prevent food from accumulating in the cheek of the affected side of a client recovering from a stroke. The speech pathologist will assist the client with tongue exercises that will help move the food bolus to the unaffected side.Lifting the chin is not an appropriate technique. A solid diet would not necessarily be the best choice. The dietitian will be consulted to evaluate the nutritional status of the client as well as make recommendations regarding the correct diet.

A family member of a client with a recent spinal cord injury asks the nurse, "Can you please tell me what the real prognosis for recovery is? I don't feel like I'm getting a straight answer." What is the nurse's best response? A) "Every injury is different, and it is too soon to have any real answers right now." B) "Only time will tell." C) "The Health Insurance Portability and Accountability Act requires that I obtain the client's permission first." D) "Please request a meeting with the health care provider. I will help set that up."

Answer: D. The nurse's best response to a family member of a client with a recent spinal cord injury is, "Please request a meeting with the primary health care provider. I will set that up." Questions concerning prognosis and potential for recovery would be referred to the primary health care provider. The nurse can help facilitate the meeting however.The timing and extent of recovery are different for each client, but it is not the nurse's role to inform the client and family members of the client's prognosis. Telling the family that "only time will tell" is too vague and minimizes the family's concern. The client was informed of Health Insurance Portability and Accountability Act (HIPAA) rights on admission or when consciousness was established, so permission has already been granted by the client.

In the emergency department (ED), which is the nursing priority in assessing the client with a spinal cord injury? A) Indication of allergies. B) Level of consciousness. C) Loss of sensation. D) Patent airway.

Answer: D. The nursing priority when assessing a client with a spinal cord injury is a patent airway. Clients with injuries at or above T6 are at risk for respiratory complications. Assessing for a patent airway is essential.Asking the client about current medications and allergies is part of every trauma assessment. Assessing the level of consciousness utilizing the Glasgow Coma Score (GCS) is an important part of the trauma assessment. Determining the level of loss of sensation will be included in the neurological evaluation.

A client is admitted with a spinal cord injury at the seventh cervical vertebra secondary to a gunshot wound. Which nursing intervention is the priority for this client's care? A) Auscultating bowel sounds every 2 hours B) Beginning a bladder retraining program C) Monitoring nutritional status D) Positioning the client to maximize ventilation potential

Answer: D. The priority nursing intervention for a client with a spinal cord injury at the seventh cervical vertebra is to position the client to maximize ventilation potential. Airway management is the priority for the client with a spinal cord injury. The client with a cervical spinal cord injury is at high risk for respiratory compromise because the cervical spinal nerves (C3-C5) innervate the phrenic nerve, controlling the diaphragm.Auscultating bowel sounds is important since paralytic ileus can develop from a SCI; however this is not the priority intervention. Beginning bladder retraining and monitoring the nutritional status will be important for adequate healing and progress to rehabilitation. However, these interventions can be delayed until major life threats are addressed.\

A client is being discharged home after surgery for brain cancer. Which statement by the client's spouse indicates a correct understanding of the nurse's discharge teaching? A) "I will have to quit my job to care for my spouse." B) "Life will be back to normal soon." C) "The case manager will provide home care." D) "We can find a support group through the local American Cancer Society."

Answer: D. The statement by the spouse of a brain cancer client that shows correct understanding of discharge teaching is when the spouse says, "We can find a support group through the local American Cancer Society." The American Cancer Society is a good community resource for clients with malignant tumors and their families.It is not a requirement that the client's spouse quit his or her job but may need some assistance in home. A diagnosis of brain cancer is life changing and the client and spouse will find a "new normal"; however this will not happen immediately. The case manager helps coordinate care and will be able to locate home care but does not provide that care.

To prevent the leading cause of death for clients with spinal cord injury, collaboration with which component of the primary health care team is a nursing priority? A) Nutritional therapy. B) Occupational therapy. C) Physical therapy. D) Respiratory therapy.

Answer: D. To help prevent death for a client with spinal cord injury, collaboration with the Respiratory therapy team is a priority. A client with a cervical spinal cord injury is at risk for breathing problems including pneumonia and aspiration, resulting from the interruption of spinal innervation to the respiratory muscles. Collaboration with Respiratory therapy is crucial.Collaboration with nutritional therapy, occupational therapy, and physical therapy does not help prevent the leading cause of death in clients with spinal cord injury.

The nurse is teaching the spouse and client who has had a brain attack about rehabilitation. Which statement by the spouse demonstrates understanding of the nurse's instruction? A) "Frequent stimulation will help with the rehabilitation process." B) "My spouse will no longer need to take blood pressure medication." C) "Rehabilitation and physical therapy are the same thing." D) "The rehabilitation therapist will help identify changes needed at home."

Answer: D. Understanding instructions about brain attack is demonstrated by the statement that the rehabilitation therapist will help identify any needed home changes. The rehabilitation therapist and home health professionals assist the client and family in adapting the home environment to the client's needs and assess the client's need for therapy.An appropriate amount of stimulation based on the client's needs will be determined by the therapist and incorporated into a comprehensive plan. Any medication regimen established for the client after the brain attack must be maintained. Rehabilitation is much more comprehensive than physical therapy.

A client has returned to the unit after a thymectomy and is extubated. The client begins to report chest pain. What does the nurse do next? A) Calls the Rapid Response Team for immediate intubation B) Gives sublingual nitroglycerin (Nitrostat) C) Increases the intravenous (IV) rate D) Informs the surgeon immediately

Answer: D. When a postoperative thymectomy client complains of chest pain, the nurse's next step is to call the surgeon right away. The client's chest pain could be a symptom of a hemothorax or pneumothorax and must be reported to the surgeon immediately.It may be beneficial to notify the Rapid Response Team based on hospital policy; however this would not be the next step. Intubation is not absolutely indicated in this case. The cause of chest pain is unclear but likely related to the thymectomy; therefore, sublingual nitroglycerin would not be beneficial. Increasing the intravenous (IV) rate is not indicated.

_____: A class of drugs used to control seizures. Also called anticonvulsants.

Antiepileptic drugs (AEDs)

_____: Inability to use or comprehend spoken or written language due to brain injury or disease.

Aphasia

_____: The loss of the ability to carry out a purposeful motor activity.

Apraxia

_____: An abnormality that occurs during embryonic development, resulting in a tangled mass of malformed, thin-walled, dilated vessels. The congenital absence of a capillary network in these vessels forms an abnormal communication between the arterial and venous systems and increases the risk that the vessels may rupture, causing bleeding, such as into the subarachnoid space or into the intracerebral tissue with brain AVMs. In the absence of the capillary network, the thin-walled veins are subjected to arterial pressure.

Arteriovenous malformation (AVM)

_____: The surgical fusion of a joint.

Arthrodesis

_____: Groups of nerves that originate in the spinal cord and end in the brain.

Ascending tracts

_____: Dorsiflexion of the great toe and fanning of the other toes, which is an abnormal reflex in response to testing the plantar reflex with a pointed (but not sharp) object; indicates the presence of central nervous system disease. The normal response is plantar flexion of all toes.

Babinski's sign

_____: The point of division of a single structure into two branches.

Bifurcation

_____: Slow or no movement, as seen in a patient with Parkinson disease. Also called akinesia.

Bradykinesia

_____: The pressure gradient over which the brain is perfused. It is influenced by oxygenation, cerebral blood volume, blood pressure, cerebral edema, and intracranial pressure (ICP) and is determined by subtracting the mean ICP from the mean arterial pressure. A cerebral perfusion pressure above 70 mm Hg is generally accepted as an appropriate goal of therapy.

Cerebral perfusion pressure (CPP)

_____: Rapid, jerky movement.

Choreiform movements

_____: At the base of the brain, the ring formed by the anterior, middle, and posterior cerebral arteries where they are joined together by small communicating arteries.

Circle of Willis

_____: A type of oculotemporal or oculofrontal headache marked by unilateral, excruciating, nonthrobbing pain that is felt deep in and around the eye and may radiate to the forehead, temple, cheek, ear, occiput, or neck. Average duration is 10 to 45 minutes. Headaches occur every 8 to 12 hours and up to 24 hours daily at the same time for about 6 to 8 weeks (hence the term cluster), followed by remission for 9 months to a year. Cause and mechanism are unknown but have been attributed to vasoreactivity and oxyhemoglobin desaturation.

Cluster headche

_____: A way of helping brain-injured patients regain function in areas that are essential for a return to independence and a reasonable quality of life.

Cognitive rehabilitation

_____: Unconscious and cannot be aroused despite vigorous or noxious simulation.

Comatose

_____: Pertaining to the opposite side.

Contralateral

_____: A technique for assisting the tetraplegic patient to cough. Place his or her hands on either side of the rib cage or upper abdomen below the diaphragm; then, as the patient inhales, push upward to help expand the lungs and cough.

Cough assist

_____: A type of closed head injury that is usually related to high-speed acceleration/deceleration, as with motor vehicle crashes. There is significant damage to axons in the white matter, and there are lesions in the corpus callosum, midbrain, cerebellum, and upper brainstem. Patients with severe injury may present with immediate coma, and most survivors require long-term care.

Diffuse axonal injury (DAI)

_____: An inflammation of the brain parenchyma (brain tissue) and meninges that affects the cerebrum, brainstem, and cerebellum; usually caused by a virus.

Encephalitis

_____: Tests to measure the electrical signals to the brain generated by hearing, touch, or sight. Also called evoked response.

Evoked potentials

_____: A static traction device used for immobilization of the cervical spine. Four pins or screws are inserted into the skull, and a metal halo ring is attached to a plastic vest or cast when the spine is stable, allowing increased patient mobility.

Halo fixator

_____: A clear or yellowish ring surrounding a spot of blood.

Halo sign

_____: Blindness in half of the visual field of one or both eyes. Also called hemianopia.

Hemianopsia

_____: Paralysis on one side of the body.

Hemiplegia

_____: The protrusion (herniation) of the pulpy material from the center of a vertebral disk; herniated disks occur most often between the fourth and fifth lumbar vertebrae (L4-5) but may occur at other levels. A herniation in the lumbosacral area can press on the adjacent spinal nerve (usually the sciatic nerve), causing severe burning or stabbing pain into the leg or foot, or it may press on the spinal cord itself, causing leg weakness and bowel and bladder dysfunction. The specific area of pain depends on the level of herniation.

Herniated nucleus pulposus (HNP)

_____: Abnormal bony overgrowth, often into muscle; seen as a complication of prolonged immobility in patients with spinal cord injury.

Heterotropic ossification (HO)

_____: Condition in which there is blindness in the same side of both eyes.

Homonymous hemianopsia

_____: A hereditary disorder transmitted as an autosomal dominant trait at the time of conception (formerly called Huntington chorea). Men and women between 35 and 50 years of age are affected; clinical onset is gradual. The two main symptoms are progressive mental status changes (leading to dementia) and choreiform movements (rapid, jerky movements) in the limbs, trunk, and facial muscles.

Huntington disease

_____: Increased partial pressure of arterial carbon dioxide (PaCO2) levels.

Hypercarbia

_____: A mechanism of injury that occurs when a part of the body is suddenly accelerated and then decelerated, causing extreme extension.

Hyperextension

_____: A mechanism of injury that occurs when a part of the body is suddenly and forcefully accelerated forward, causing extreme flexion.

Hyperflexion

_____: A type of brain attack caused by occlusion of a cerebral artery by either a thrombus or an embolus. About 80% of all brain attacks are ischemic.

Ischemic stroke

_____: Turning technique in which the patient turns all at once while his or her back is kept as straight as possible.

Log rolling

_____: Pain in the lumbosacral region of the back caused by muscle strain or spasm, ligament sprain, disk degeneration, or herniation of the nucleus pulposus from the center of the disk. Herniated disks occur most often between the fourth and fifth lumbar vertebrae (L4-5) but may occur at other levels.

Low back pain (LBP)

_____: The insertion of a spinal needle into the subarachnoid space between the third and fourth (sometimes the fourth and fifth) lumbar vertebrae to withdraw spinal fluid for analysis.

Lumbar puncture (spinal tap)

_____: A noninvasive imaging technique that measures the magnetic fields produced by electrical activity in the brain via extremely sensitive devices such as superconducting quantum interference devices (SQUIDs).

Magnetoencephalography (MEG)

_____: A type of traumatic primary brain injury that occurs with a skull fracture or when the skull is pierced by a penetrating object. The integrity of the brain and the dura is violated, and there is exposure to outside contaminants, with damage to the underlying vessels, dural sinus, brain, and cranial nerves.

Open traumatic brain injury

_____: An individual who has consented to donating one or more organs when he or she dies.

Organ donor

_____: Ear discharge.

Otorrhea

_____: An acronym that stands for the phrase "Pupils should be equal in size, round and regular in shape, and react to light and accommodation."

PERRLA

_____: Edema and hyperemia of the optic disc; a sign of increased intracranial pressure found on ophthalmoscopic examination. Also called a choked disc.

Papiledema

_____: Edema and hyperemia of the optic disc; a sign of increased intracranial pressure found on ophthalmoscopic examination. Also called a choked disc.

Papilledema

_____: Weakness that involves only the lower extremities, as seen in lower thoracic and lumbosacral injuries or lesions.

Paraparesis

_____: Paralysis that involves only the lower extremities, as seen in lower thoracic and lumbosacral injuries or lesions.

Paraplegia

_____: Abnormal or unusual nerve sensations of touch, such as tingling and burning.

Paresthesia

_____: One of the three broad categories of seizure disorders along with generalized seizure and unclassified seizure. Partial seizures are of two types: complex and simple. Partial seizures begin in a part of one cerebral hemisphere; some can evolve into generalized tonic-clonic, tonic, or clonic seizures. They are most often seen in adults and in general are less responsive to medical treatment. Also called focal seizures or local seizures.

Partial seizure

_____: Procedure performed under general anesthesia to treat trigeminal neuralgia; a hollow needle is passed through the inside of the patient's cheek into the trigeminal nerve fibers, and a heating current (radiofrequency thermocoagulation) goes through the needle to destroy some of the fibers.

Percutaneous stereotactic rhizotomy (PSR)

_____: A type of multiple sclerosis (MS) that occurs in only 5% of patients with MS. It is characterized by the absence of periods of remission, and the patient's condition does not return to baseline. Progressive cumulative symptoms and deterioration occur over several years.

Progressive-relapsing MS (PRMS)

_____: Occurs in a patient with muscle weakness due to cerebral or brainstem reasons. The arm on the weak side tends to fall, or "drift," with the palm pronating (turning inward) after the patient has closed his or her eyes and held the arms perpendicular to the body with the palms up for 15 to 30 seconds; part of the neurologic assessment.

Pronator drift

_____: Chemicals that are produced in the cells and cause inflammation and swelling.

Prostaglandins

_____: Weakness that involves all four extremities; seen with cervical spinal cord injury.

Quadriparesis

_____: Recent memory, which can be tested during the history taking by asking about items such as the dates of clinic or physician appointments.

Recall memory

_____: A type of aphasia caused by injury to Wernicke's area in the temporoparietal area of the brain and characterized by an inability to understand the spoken and written word; reading and writing ability are equally affected. Although the patient can talk, the language is often meaningless and neologisms (made-up words) are common parts of speech.

Receptive (Wernicke's, or sensory) aphasia

_____: An intervention to help with communication problems in patients with dementia; consists of attracting the patient's attention before conversing, keeping the environment as free of distractions as possible, and speaking directly to the patient in a distinct manner using clear and short sentences.

Redirection

_____: A closed circuit of spinal and peripheral nerves that requires no control by the brain.

Reflex arc

_____: A type of multiple sclerosis that occurs in 85% of cases and is characterized by a mild or moderate course, depending on the degree of disability. Relapses develop over 1 to 2 weeks and resolve over 4 to 8 months, after which the patient returns to baseline.

Relapsing-remitting multiple sclerosis (RRMS)

_____: Long-term memory of events; can be tested by asking patients about their birth date, schools attended, city of birth, or anything from the past that can be verified.

Remote memory

_____: Swaying or falling when the patient is standing with arms at the sides, feet and knees close together, and eyes closed; a test of equilibrium in neurologic assessment.

Romberg sign

_____: A mechanism of injury in which the head is turned excessively beyond the normal range.

Rotation

_____: Acronym for systemic inflammatory response syndrome, an inflammatory state affecting the whole body.

SIRS

_____: Prolonged seizures lasting more than 5 minutes or repeated seizures over the course of 30 minutes; a potential complication of all types of seizures.

Status epilepticus

_____: An alternative to stereotactic pallidotomy as a surgical treatment for the patient with Parkinson disease; uses thermocoagulation of brain cells to reduce tremor. Usually only unilateral surgery is performed to benefit the side of the body most affected by the disease.

Thalamotomy

_____: Treatment that lowers the body core temperature to reduce the risk of cell, tissue, and organ damage from a low or absent blood flow. Usually follows cardiac arrest.

Therapeutic hypothermia

_____: Pertaining to a state of stiffening or rigidity of the muscles, particularly of the arms and legs, and immediate loss of consciousness of a tonic-clonic seizure.

Tonic phase

_____: Part of the neurologic examination. The patient closes his or her eyes while the practitioner touches the patient with a finger and asks that the patient point to the area touched.

Touch discrimination

_____: A position in which the patient lies in the semi-Fowler's position and flexes the knees to relax the muscles of the lower back and relieve pressure on the spinal nerve root. This is typically more comfortable and therapeutic for the patient with low back pain.

Williams position

Which statements about stroke prevention indicate a client's understanding of health teaching by the nurse? (Select all that apply.) A) "I will take aspirin every day." B) "I have decided to stop smoking." C) "I will try to walk at least 30 minutes most days of the week." D) "I need to cut down a lot on my drinking." E) "I'm going to decrease salt in my diet."

ANS: B, C, D, E. Smoking, lack of exercise, excessive drinking, and a high salt intake are all risk factors for developing a stroke. Therefore, choices B, C, D, and E are the correct answers that show that the client needs to make lifestyle changes in these areas. Daily aspirin should only be taken on the advice of a primary health care provider.

The nurse assesses an older adult with a diagnosis of severe, late-stage Alzheimer's disease. Which assessment findings would the nurse expect for this client? (Select all that apply.) A) Acute confusion B) Hallucinations C) Wandering D) Urinary incontinence E) Difficulty eating

ANS: B, D, E. The client with severe, late-stage Alzheimer's disease (AD) is not able to ambulate, so C is not a correct answer. They often become psychotic, are incontinent, and cannot perform ADLs, which makes B, D, and E the correct choices for this question. Clients with AD have chronic confusion, not acute confusion, so A is also not correct.

During a client's neurologic assessment, the nurse finds that the client continues to be drowsy but easily awakened. How does the nurse document this client's level of consciousness? A) Stuporous B) Lethargic C) Comatose D) Alert

ANS: B. A lethargic patient can be easily awakened; a stuporous patient requires painful or noxious stimulation to awaken. The comatose patient cannot be awakened despite stimulation.

The nurse is caring for a patient treated with alteplase following a stroke. What assessment finding is the highest priority for the nurse? A) Client's blood pressure is 144/90. B) Client is having epistaxis. C) Client ate only half of the last meal. D) Client continues to be drowsy.

ANS: B. Alteplase can cause bleeding and therefore the nurse assesses for indications of bleeding. Epistaxis is a nosebleed and therefore choice B is the correct answer. The client BP of 144/90 is acceptable for a stroke patient (A). The other assessments are important to document but not related to the administration of alteplase; they are also not potentially life-threatening (C and D).

A client returns from the postanesthesia care unit (PACU) after a surgical removal of a brainstem tumor. What position will the nurse place the client in at this time? A) Turn the patient from side to side to prevent aspiration. B) Keep the client flat in bed or up 10 degrees and reposition from side to side. C) Elevate the head of the bed to at least 30 degrees at all times. D) Keep the client in a sitting position in bed at all times.

ANS: B. The brainstem is located below the tentorium (infratentorial) and therefore the best practice position is side-lying and either flat or up 10 degrees (B). Choices C and D are not appropriate for this type of brain tumor surgery. Turning the patient side-to-side is appropriate for all types of surgery to prevent pneumonia and pressure injuries, not to prevent aspiration (A).

A client with early dementia asks the nurse to find her mother who is deceased. What is the nurse's most appropriate response? A) "We can call her in a little while if you want." B) "Your mother died over 20 years ago." C) "What did your mother look like?" D) "I'll ask your father to find her when he visits."

ANS: B. The client has early dementia which means that he or she experiences times of orientation and other times where memory is decreased. Therefore, the best response is to reorient the client to reality in this case.

The nurse is preparing a teaching plan for a client with migraine headaches. Which of these foods or food additives that may trigger a migraine will the nurse include in the teaching? (Select all that apply.) A) Sugar B) Salt C) Monosodium glutamate (MSG) D) Caffeine E) Wine F) Tyramine

ANS: C, D, E, F. As listed in Chart 42-7, common triggers for a migraine are MSG, caffeine, wine (any alcoholic beverage), and foods containing tyramine, such as aged cheese (C, D, E, F). Sugar and salt usually do not cause migraine headaches, but artificial sweeteners may trigger them.

The nurse is caring for a patient with expressive (Broca's) aphasia. Which nursing intervention is appropriate for communicating with the client? A) Refer the patient to the speech-language pathologist. B) Speak loudly to help the client interpret what is being said. C) Provide pictures to help the client communicate. D) Ask the client to read messages on a white board.

ANS: C. The client with expressive aphasia has difficulty speaking and writing, and usually little problem with understanding. Therefore, it would not be useful to ask the client to read from a white board (D). Speaking loudly is not appropriate because the client has a speech problem, not a hearing problem (B). While referral to a speech-language pathologist is an appropriate intervention, it does not help to communicate with the client at this time (A). Providing pictures to the client allows him or her to communicate graphically rather than by verbalizing; therefore, C is the best answer.

The nurse is caring for a client with chronic confusion who often yells and screams when touched. Which nursing intervention is most appropriate when caring for this client? A) Provide a large clock and calendar for the patient to read. B) Use removable restraints like a roll-waist belt to prevent wandering. C) Approach the patient so that the nurse can be seen clearly. D) Place the patient in a room close to the nurses' station for frequent observation.

ANS: C. The patient is likely afraid and cannot process being touched. While using a large clock and calendar is recommended for anyone who is confused, these devices will not prevent the patient from screaming when touched (A). Using restraints or putting the patient in a busy, noisy environment like the nurses' station is likely to increase unwanted behaviors (B and C). Instead, the best action is to approach the client calmly so that he or she can see the nurse before the nurse begins to provide care.

A family member asks the nurse about whether there would be any long-term psychological effects from a client's mild traumatic brain injury. What is the nurse's best response? A) "You need to talk with the client's primary health care provider." B) "Usually any effects last for only a few weeks or months." C) "Each person's reaction to brain injury is different." D) "You should expect a change in the client's personality."

ANS: C. While it is important to refer the client to the primary health care provider (A), the best response is to be realistic and explain that each person reacts differently to a mild TBI. Many clients experience some type of cognitive or emotional change(s), but some do not. Therefore, C is the best option.

The nurse is caring for a client with trigeminal neuralgia. Which patient problem is the priority for the nurse? A) Facial twitching B) Problems with communication C) Ptosis and diplopia D) Severe facial pain

ANS: D. The client with trigeminal neuralgia (TN) has severe burning or sharp pain that is worsened by facial movement or eating. While the client may also experience facial twitching, managing pain is the priority problem. The client with TN usually does not have problems with communication or facial paralysis.

The nurse is preparing a client diagnosed with rule-out meningitis for a lumbar puncture. Which interventions should the nurse implement? Select all that apply. 1. Obtain an informed consent from the client or significant other. 2. Have the client empty the bladder prior to the procedure. 3. Place the client in a side-lying position with the back arched. 4. Instruct the client to breathe rapidly and deeply during the procedure. 5. Explain to the client what to expect during the procedure.

ANSWER: 1, 2, 3, 5. 1. A lumbar puncture is an invasive procedure; therefore, an informed consent is required. 2. This could be offered for client comfort during the procedure. 3. This position increases the space between the vertebrae, which allows the HCP easier entry into the spinal column. 4. The client is encouraged to relax and breathe normally; hyperventilation may lower an elevated cerebrospinal fluid pressure. 5. The nurse should always explain to the client what is happening prior to and during a procedure. TEST TAKING HINT: This is an alternative-type question, which requires the test taker to select more than one answer option.

The client is being discharged following a transsphenoidal hypophysectomy. Which discharge instructions should the nurse teach the client? Select all that apply. 1. Sleep with the head of the bed elevated. 2. Keep a humidifier in the room. 3. Use caution when performing oral care. 4. Stay on a full liquid diet until seen by the HCP. 5. Notify the HCP if developing a cold or fever

ANSWER: 1, 2, 3, 5. 1. The client should sleep with the head of the bed elevated to promote drainage of the cerebrospinal fluid. 2. Humidified air will prevent drying of the nasal passages. 3. Because the incision for this surgery is just above the gumline, the client should not brush the front teeth. Oral care should be performed using a sponge until the incision has healed. 4. The client can eat a regular diet. 5. The HCP should be notified if the client develops an infection of any kind. A cold with sinus involvement and sneezing places the client at risk for opening the incision and developing a brain infection. TEST TAKING HINT: The test taker could choose option "5" because this is a standard instruction for any surgery. The test taker should look for more than one correct answer in an alternative-type question.

The nurse is admitting the client for ruleout encephalitis. Which interventions should the nurse assess to support the diagnosis of encephalitis? Select all that apply. 1. Determine if the client has recently received any immunizations. 2. Ask the client if he or she has had a cold in the last week. 3. Check to see if the client has active herpes simplex 1. 4. Find out if the client has traveled to the Great Lakes region. 5. Assess for exposure to soil with fungal spores

ANSWER: 1, 2, 3. 1. A complication of immunizations for measles, mumps, and rubella can be encephalitis. 2. Upper respiratory tract illnesses can be a precursor to encephalitis. 3. The herpes simplex virus, specifically type 1, can lead to encephalitis. 4. Fungal encephalitis is known to occur in certain regions, and the nurse should assess for recent trips to areas where these fungal spores exist, but the common areas are the southwest United States and central California. 5. Exposure to spores does not lead to encephalitis. TEST TAKING HINT: Encephalitis is inflammation of the brain caused by either a hypersensitivity reaction or a postinfectious state in which a virus reproduces in the brain. Encephalitis can be a life-threatening disease process. History is vital in the diagnosis.

Which intervention should the nurse implement when caring for the client diagnosed with encephalitis? Select all that apply. 1. Turn the client every two (2) hours. 2. Encourage the client to increase fluids. 3. Keep the client in the supine position. 4. Assess for deep vein thrombosis (DVT). 5. Assess for any alterations in elimination

ANSWER: 1, 2, 4, 5. 1. Clients with encephalitis should be treated for the disease process and also to prevent complications of immobility. Turning the client will prevent skin breakdown. 2. Increasing fluids helps prevent urinary tract infections and mobilize secretions in the lungs. 3. The client would be maintained in a slightly elevated position, semi-Fowler's, for gravity to assist the body in decreasing intracranial pressure. 4. Immobility causes clients to be at risk for deep vein thrombosis. Therefore, clients with encephalitis should be assessed for deep vein thrombosis. 5. Immobility causes the gastrointestinal tract to slow, resulting in constipation. Clients can have difficulty emptying their bladders, which can cause retention and urinary tract infections and stones. Assessing these systems can identify problems early. TEST TAKING HINT: Each option should be read carefully. If the test taker does not read each one carefully, the test taker could miss important words, such as "supine" in option "3," resulting in an incorrect answer.

Which intervention should the nurse implement to decrease increased intracranial pressure (ICP) for a client on a ventilator? Select all that apply. 1. Position the client with the head of the bed up 30 degrees. 2. Cluster activities of care. 3. Suction the client every three (3) hours. 4. Administer soapsuds enemas until clear. 5. Place the client in Trendelenburg position.

ANSWER: 1, 2. 1. Elevating the head of the bed 30 degrees will decrease ICP by using gravity to drain cerebrospinal fluid. 2. Minimizing disturbing the client and allowing rest in between activities will decrease ICP. 3. Suctioning increases ICP and should not be performed unless absolutely necessary. 4. Soapsuds enemas increase intra-abdominal pressure, which, in turn, increases ICP. 5. Trendelenburg position is head down, feet up. This would increase ICP.

The client has been newly diagnosed with epilepsy. Which discharge instructions should be taught to the client? Select all that apply. 1. Keep a record of seizure activity. 2. Take tub baths only; do not take showers. 3. Avoid over-the-counter medications. 4. Have anticonvulsant medication serum levels checked regularly. 5. Do not drive alone; have someone in the car.

ANSWER: 1, 3, 4. 1. Keeping a seizure and medication chart will be helpful when keeping follow-up appointments with the health-care provider and in identifying activities that may trigger a seizure. 2. The client should take showers, rather than tub baths, to avoid drowning if a seizure occurs. The nurse should also instruct the client never to swim alone. 3. Over-the-counter medications may contain ingredients that will interact with antiseizure medications or, in some cases, as with use of stimulants, possibly cause a seizure. 4. Most of the anticonvulsant medications have therapeutic serum levels that should be maintained, and regular checks of the serum levels help to ensure the correct level. 5. A newly diagnosed client would have just been put on medication, which may cause drowsiness. Therefore, the client should avoid activities that require alertness and coordination and should not be driving at all until after the effects of the medication have been evaluated. TEST TAKING HINT: The test taker must select all interventions that are appropriate for the question. A key word is the adverb "newly."

The intensive care nurse is caring for the client who has had intracranial surgery. Which interventions should the nurse implement? Select all that apply. 1. Assess for deep vein thrombosis. 2. Administer intravenous anticoagulant. 3. Monitor intake and output strictly. 4. Apply warm compresses to the eyes. 5. Perform passive range-of-motion exercises

ANSWER: 1, 3, 5. 1. Assessing for deep vein thrombosis, which is a complication of immobility, would be appropriate for this client. 2. Anticoagulants may cause bleeding; therefore, the client who has had surgery would not be prescribed this medication. 3. Monitoring of intake and output helps to detect possible complications of the pituitary gland, which include diabetes insipidus and syndrome of inappropriate antidiuretic hormone (SIADH). 4. The nurse should apply cool compresses to alleviate periocular edema. 5. The nurse does not want the client to be active and possibly increase intracranial pressure; therefore, the nurse should perform passive range-of-motion for the client.

The nurse in the neurointensive care unit is caring for a client with a new Cervical SCI who is breathing independently. Which nursing interventions should be implemented? Select all that apply. 1. Monitor the pulse oximetry reading. 2. Provide pureed foods six (6) times a day. 3. Encourage coughing and deep breathing. 4. Assess for autonomic dysreflexia. 5. Administer intravenous corticosteroids.

ANSWER: 1, 3, 5. 1. Oxygen is administered initially to prevent hypoxemia, which can worsen the spinal cord injury; therefore, the nurse should determine how much oxygen is reaching the periphery. 2. A C6 injury would not affect the client's ability to chew and swallow, so pureed food is not necessary. 3. Breathing exercises are supervised by the nurse to increase the strength and endurance of inspiratory muscles, especially those of the diaphragm. 4. Autonomic dysreflexia occurs during the rehabilitation phase, not the acute phase. 5. Corticosteroids are administered to decrease inflammation, which will decrease edema, and help prevent edema from ascending up the spinal cord, causing breathing difficulties. TEST TAKING HINT: The test taker must notice where the client is receiving care, which may be instrumental in being able to rule out incorrect answer options and help in identifying the correct answer. Remember Maslow's hierarchy of needs—oxygen and breathing are priority nursing interventions.

The client diagnosed with a right-sided cerebrovascular accident is admitted to the rehabilitation unit. Which interventions should be included in the nursing care plan? Select all that apply. 1. Position the client to prevent shoulder adduction. 2. Turn and reposition the client every shift. 3. Encourage the client to move the affected side. 4. Perform quadriceps exercises three (3) times a day. 5. Instruct the client to hold the fingers in a fist.

ANSWER: 1, 3. 1. Placing a small pillow under the shoulder will prevent the shoulder from adducting toward the chest and developing a contracture. 2. The client should be repositioned at least every two (2) hours to prevent contractures, pneumonia, skin breakdown, and other complications of immobility. 3. The client should not ignore the paralyzed side, and the nurse must encourage the client to move it as much as possible; a written schedule may assist the client in exercising. 4. These exercises are recommended, but they must be done at least five (5) times a day for 10 minutes to help strengthen the muscles for walking. 5. The fingers are positioned so that they are barely flexed to help prevent contracture of the hand. TEST TAKING HINT: Be sure to look at the intervals of time for any intervention; note that "every shift" and "three (3) times a day" are not appropriate time intervals for this client. Because this is a "select all that apply" question, the test taker must read each answer option and decide if it is correct; one will not eliminate another.

The male client is admitted to the emergency department following a motorcycle accident. The client was not wearing a helmet and struck his head on the pavement. The nurse identifies the concept as impaired intracranial regulation. Which interventions should the emergency department nurse implement in the first five (5) minutes? Select all that apply. 1. Stabilize the client's neck and spine. 2. Contact the organ procurement organization to speak with the family. 3. Elevate the head of the bed to 70 degrees. 4. Perform a Glasgow Coma Scale assessment. 5. Ensure the client has a patent peripheral venous catheter in place. 6. Check the client's driver's license to see if he will accept blood.

ANSWER: 1, 4, 5. 1. The first nursing action is to ensure that the client does not sustain further damage to the spinal cord. The nurse does this by placing sandbags around the client's head or by maintaining the client on a backboard with the head securely affixed to the board. 2. This will not occur until a full assessment is made and brain death is imminent. 3. The head of the bed has to be kept flat with the client's head stabilized until spinal damage has been ruled out. 4. The Glasgow Coma Scale is a systematic tool used to assess a client's neurological status. It gives health-care workers a standard method to determine the progress of a client's condition. 5. The client should have an access to be able to administer emergency medications. 6. In an emergency the nurse must concentrate on the immediate care of the client. If the client requires a blood transfusion, it will take time to have the type and crossmatch completed. TEST TAKING HINT: The nurse must remember that safety is a priority when caring for clients who are incapable or unable to protect themselves, as in option "1." Option "3" is an appropriate assessment method for a traumatic brain injury because the first step in the nursing process is assessment.

The client is diagnosed with Huntington's chorea. Which interventions should the nurse implement with the family? Select all that apply. 1. Refer to the Huntington's Chorea Foundation. 2. Explain the need for the client to wear football padding. 3. Discuss how to cope with the client's messiness. 4. Provide three (3) meals a day and no between-meal snacks. 5. Teach the family how

ANSWER: 1,2,3. 1. Foundations offer the family and client information about the disease, support groups, and up-to-date information on current research. 2. The use of padding will help prevent injury from the constant movement that occurs with this disease. 3. The constant movement causes the client to be messy when eating, dressing, or performing activities of daily living. 4. The constant movements expend more calories; therefore, the client should have three (3) meals plus between-meal snacks. 5. The client is at risk for choking; therefore, teaching the Heimlich maneuver is appropriate, but teaching chest percussion is not.

The occupational health nurse is concerned about preventing occupation-related acquired seizures. Which intervention should the nurse implement? 1. Ensure that helmets are worn in appropriate areas. 2. Implement daily exercise programs for the staff. 3. Provide healthy foods in the cafeteria. 4. Encourage employees to wear safety glasses.

ANSWER: 1. 1. Head injury is one of the main reasons for epilepsy that can be prevented through occupational safety precautions and highway safety programs. 2. Sedentary lifestyle is not a cause of epilepsy. 3. Dietary concerns are not a cause of epilepsy. 4. Safety glasses will help prevent eye injuries, but such injuries are not a cause of epilepsy. TEST TAKING HINT: The nurse must be aware of risk factors that cause diseases. If the test taker does not know the correct answer, thinking about which body system the question is asking about may help rule out or rule in some of the answer options. Only options "1" and "4" have anything to do with the head, and only helmets on the head are connected with the neurological system.

The nurse is caring for a client diagnosed with meningitis. Which collaborative intervention should be included in the plan of care? 1. Administer antibiotics. 2. Obtain a sputum culture. 3. Monitor the pulse oximeter. 4. Assess intake and output

ANSWER: 1. 1. A nurse administering antibiotics is a collaborative intervention because the HCP must write an order for the intervention; nurses cannot prescribe medications unless they have additional education and licensure and are nurse practitioners with prescriptive authority. 2. The nurse needs an order to send a culture to the laboratory for payment purposes, but the nurse can obtain a specimen without an order. A sputum specimen is not appropriate for meningitis. 3. A pulse oximeter measures the amount of oxygen in the periphery and does not require an HCP to order. 4. Intake and output are independent nursing interventions and do not require an HCP's order. TEST TAKING HINT: The test taker must note adjectives and understand that a collaborative nursing intervention is dependent on another member of the health-care team; an independent nursing intervention does not require collaboration

The client with end-stage ALS requires a gastrostomy tube feeding. Which finding would require the nurse to hold a bolus tube feeding? 1. A residual of 125 mL. 2. The abdomen is soft. 3. Three episodes of diarrhea. 4. The potassium level is 3.4 mEq/L.

ANSWER: 1. 1. A residual (aspirated gastric contents) of greater than 50 to 100 mL indicates that the tube feeding is not being digested and that the feeding should be held. 2. A soft abdomen is normal; a distended abdomen would be cause to hold the feeding. 3. Diarrhea is a common complication of tube feedings, but it is not a reason to hold the feeding. 4. The potassium level is low and needs intervention, but this would not indicate a need to hold the bolus tube feeding. TEST TAKING HINT: Knowing normal assessment data would lead the test taker to eliminate option "2" as a possible correct answer. Diarrhea and hypokalemia would not cause the client to not receive a feeding. Even if the test taker did not know what "residual" means, this would be the best option.

The significant other of a client diagnosed with a brain tumor asks the nurse for help identifying resources. Which would be the most appropriate referral for the nurse to make? 1. Social worker. 2. Chaplain. 3. Health-care provider. 4. Occupational therapist.

ANSWER: 1. 1. A social worker is qualified to assist the client with referrals to any agency or personnel that is needed. 2. The chaplain should be referred if spiritual guidance is required, but the stem did not specify this need. 3. The HCP also can refer to the social worker, but the nurse can make this referral independently. 4. The occupational therapist assists with cognitive functioning, activities of daily living (ADLs), and modification of the home, but the stem did not define these needs. TEST TAKING HINT: The test taker must decide what each discipline has to offer the client; an SW has the broadest range of referral capabilities

Which client would the nurse identify as being most at risk for experiencing a cerebrovascular accident (CVA)? 1. A 55-year-old African American male. 2. An 84-year-old Japanese female. 3. A 67-year-old Caucasian male. 4. A 39-year-old pregnant female.

ANSWER: 1. 1. African Americans have twice the rate of CVAs as Caucasians and men have a higher incidence than women; African Americans also suffer more extensive damage from a CVA than do people of other cultural groups. 2. Females are less likely to have a CVA than males, but advanced age does increase the risk for CVA. The Asian population has a lower risk, possibly as a result of their relatively high intake of omega-3 fatty acids, antioxidants found in fish. 3. Caucasians have a lower risk of CVA than do African Americans, Hispanics, and Native Pacific Islanders. 4. Pregnancy is a minimal risk for having a CVA. TEST TAKING HINT: Note the age of the client if this information is given, but take this information in context with the additional information provided in the answer options. The 84-year-old may appear to be the best answer but not if the client is a female and Asian, which rules out this option for the client most at risk.

The nurse asks the male client with epilepsy if he has auras with his seizures. The client says, "I don't know what you mean. What are auras?" Which statement by the nurse would be the best response? 1. "Some people have a warning that the seizure is about to start." 2. "Auras occur when you are physically and psychologically exhausted." 3. "You're concerned that you do not have auras before your seizures?" 4. "Auras usually cause you to be sleepy after you have a seizure."

ANSWER: 1. 1. An aura is a visual, an auditory, or an olfactory occurrence that takes place prior to a seizure and warns the client a seizure is about to occur. The aura often allows time for the client to lie down on the floor or find a safe place to have the seizure. 2. An aura is not dependent on the client being physically or psychologically exhausted. 3. This is a therapeutic response, reflecting feelings, which is not an appropriate response when answering a client's question. 4. Sleepiness after a seizure is very common, but the aura does not itself cause the sleepiness. TEST TAKING HINT: If the stem of the question has the client asking a question, then the nurse needs to give factual information, and option "3," a therapeutic response, would not be appropriate. Neither would option "2" or "4" because these options are worded in such a way as to imply incorrect information.

The client diagnosed with a mild concussion is being discharged from the emergency department. Which discharge instruction should the nurse teach the client's significant other? 1. Awaken the client every two (2) hours. 2. Monitor for increased intracranial pressure (ICP). 3. Observe frequently for hypervigilance. 4. Offer the client food every three (3) to four (4) hours.

ANSWER: 1. 1. Awakening the client every two (2) hours allows the identification of headache, dizziness, lethargy, irritability, and anxiety—all signs of post-concussion syndrome—that would warrant the significant other's taking the client back to the emergency department. 2. The nurse should monitor for signs of increased intracranial pressure (ICP), but a layman, the significant other, would not know what these signs and medical terms mean. 3. Hypervigilance, increased alertness and super-awareness of the surroundings, is a sign of amphetamine or cocaine abuse, but it would not be expected in a client with a head injury. 4. The client can eat food as tolerated, but feeding the client every three (3) to four (4) hours does not affect the development of postconcussion syndrome, the signs of which are what should be taught to the significant other. TEST TAKING HINT: Remember to pay close attention to answer options that have times (e.g., "every two [2] hours," "every three [3] to four [4] hours"). Also consider the likelihood of the options listed. Would a nurse teach the significant other terms such as increased intracranial pressure or hypervigilance? Probably not, so options "2" and "3" should be eliminated.

The client diagnosed with Parkinson's disease is prescribed carbidopa/levodopa (Sinemet). Which intervention should the nurse implement prior to administering the medication? 1. Discuss how to prevent orthostatic hypotension. 2. Take the client's apical pulse for one (1) full minute. 3. Inform the client that this medication is for short-term use. 4. Tell the client to take the medication on an empty stomach.

ANSWER: 1. 1. Because carbidopa/levodopa has been linked to hypotension, teaching a client given the medication ways to help prevent a drop in blood pressure when standing— orthostatic hypotension—decreases the risks associated with hypotension and falling. 2. The medication will not cause the heart rate to change, so taking the client's apical pulse for one (1) minute is not a priority. 3. This medication is prescribed for the client the rest of his or her life unless the medication stops working or the client experiences adverse side effects. 4. The medication should be administered with food to help prevent gastrointestinal distress.

The nurse performs an initial assessment on an older client. Which assessment findings would the nurse expect to be the result of normal physiologic aging? (Select all that apply.) A) Confusion B) Hearing loss C) Decerebrate positioning D) Slurred speech E) Constipation F) Urinary incontinence

ANS: B, E. Confusion, slurred speech, and incontinence are not normal changes of aging, although these findings are common in the older adult population. Changes in the bones of the ear and intestinal motility cause varying degrees of hearing loss and constipation.

Which intervention has the highest priority for the client in the emergency department who has been in a motorcycle collision with an automobile and has a fractured left leg? 1. Assessing the neurological status. 2. Immobilizing the fractured leg. 3. Monitoring the client's output. 4. Starting an 18-gauge saline lock

ANSWER: 1. 1. Assessment is the first step in the nursing process, and a client in a motorcycle accident must be assessed for a head injury. 2. Neurological assessment is a priority over a fractured leg. 3. The client's urinary output is not a priority over an assessment. 4. An 18-gauge IV access should be started in case the client has to go to surgery, but it is not a priority over an assessment.

The son of a client diagnosed with ALS asks the nurse, "Is there any chance that I could get this disease?" Which statement by the nurse would be most appropriate? 1. "It must be scary to think you might get this disease." 2. "No, this disease is not genetic or contagious." 3. "ALS does have a genetic factor and runs in families." 4. "If you are exposed to the same virus, you may get the disease."

ANSWER: 3. 1. The son is not sure if he may get ALS, so this is not an appropriate response. 2. This is incorrect information. 3. There is a genetic factor with ALS that is linked to a chromosome 21 defect. 4. ALS is not caused by a virus. The exact etiology is unknown, but studies indicate that some environmental factors may lead to ALS. TEST TAKING HINT: This question requires knowledge of ALS. There are some questions for which test-taking hints are not available.

_____: A group of drugs that help manage clinical depression.

Antidepressants

_____: The least common of the three types of migraine headaches, after migraines with aura and migraines without aura; the atypical category includes menstrual and cluster migraines.

Atypical migraine

_____: A sensation that signals the onset of a headache or seizure; the patient may experience visual changes, flashing lights, or double vision.

Aura

_____: A syndrome that affects the patient with an upper spinal cord injury; characterized by severe hypertension and headache, bradycardia, nasal stuffiness, and flushing; caused by a noxious stimulus, usually a distended bladder or constipation. This is a neurologic emergency and must be promptly treated to prevent a hypertensive brain attack.

Autonomic dysreflexia (AD)

_____: Abnormal formations of a mass of mineral salts that can occur in the body; forms in the kidney when excess calcium precipitates out of solution. Also called stones.

Calculi

_____: The sudden, brief, jerking contraction of a muscle or muscle group often seen in seizures. Also called myoclonus.

Clonus

_____: A type of traumatic primary brain injury that occurs as the result of blunt trauma; the integrity of the skull is not violated, and damage to brain tissue depends on the degree and mechanisms of injury.

Closed traumatic brain injury

_____: An accumulation of clotted blood resulting from arterial bleeding into the space between the dura and the skull; a neurosurgical emergency.

Epidural hematoma

_____: A chronic disorder characterized by recurrent, unprovoked seizure activity; may be caused by an abnormality in electrical neuronal activity, an imbalance of neurotransmitters, or a combination of both.

Epilepsy

_____: An applied science in which the workplace is designed to increase worker comfort (thus reducing injury) while increasing efficiency and productivity.

Ergonomics

_____: An increase in severity of a disease. Also called flare-up.

Exacerbation

_____: Located below the tentorium of the cerebellum.

Infratentorial

_____: Facilitating movement.

Motor

_____: Sensory neurons that respond to pain or other noxious stimuli.

Nociceptors

_____: Abnormal or unusual nerve sensations of touch, such as tingling and burning.

Paresthesias

_____: A debilitating neurologic disease that affects motor ability and is characterized by four cardinal symptoms: tremor, rigidity, akinesia (slow movement), and postural instability. It is the third most common neurologic disorder of older adults. Also called paralysis agitans.

Parkinson disease (PD)

_____: Injuries caused by piercing; classified by the velocity of the vehicle (e.g., knife or bullet) causing the injury. Low-velocity injuries from knife wounds cause damage directly at the site; high-velocity injuries from gunshot wounds cause both direct and indirect damage. Also called penetrating injury.

Penetrating trauma

_____: Paralysis of a part of the body that is characterized by spasticity of muscles due to hypertonia; may be seen in the patient who has experienced a brain attack.

Spastic paralysis

_____: Narrowing of the spinal canal; typically seen in people older than 60 years.

Spinal stenosis

_____: A continuous ringing or noise perception in the ears.

Tinnitus

_____: The inner part of the temporal lobe of the brain that can move downward and cause pressure on the brainstem; the vital sign center.

Uncus

_____: For the patient with moderate or severe Alzheimer's disease, the process of recognizing and acknowledging the patient's feelings and concerns without reinforcing an erroneous belief (e.g., if the patient is looking for his or her deceased mother).

Validation therapy

_____: An important speech area of the cerebrum. It is located in the temporal lobe and plays a significant role in higher-level brain function. It enables the processing of words into coherent thought and recognition of the idea behind written or printed words (language).

Wernicke's area

_____: Double vision.

Diplopia

_____: The degree of alertness or the amount of stimulation needed to engage a patient's attention and can range from alert to coma.

Level of consciousness (LOC)

_____: Pupil constriction in response to bringing a penlight in from the side of the patient's head and shining the light in the eye being tested as soon as the patient opens his or her eyes.

Direct response

_____: Disk inflammation

Diskitis

_____: Drugs that improve cholinergic neurotransmission in the central nervous system by delaying the destruction of acetylcholine by acetylcholinesterase, thus delaying the onset of cognitive decline. These are approved for symptomatic treatment of Alzheimer's disease but do not affect the course of the disease.

Cholinesterase inhibitors

The nurse is teaching a client about self-management measures to help prevent low back pain. Which teaching should be included? (Select all that apply.) A) "Losing weight can decrease strain on your back." B) "Avoid twisting at your waist." C) "Exercise on a regular basis, including walking." D) "Don't bend at your waist when lifting a heavy object." E) "Eat foods high in calcium and Vitamin D to prevent bone loss."

ANS: A, B, C, D, E. Preventing increased weight or obesity and moving carefully to prevent back strain are important measures to prevent back pain. Additionally, maintaining healthy habits such as regular exercise can strengthen muscles, including back muscles. Eating foods high in calcium and Vitamin D helps to prevent bone loss. Bone loss can lead to vertebral compression fractures which cause severe back pain.

The nurse is teaching a client about taking a new prescription for pyridostigmine. Which statements by the nurse indicate correct information about this drug? (Select all that apply.) A) "Avoid opioids and other sedating drugs when taking this medication." B) "Report increased mucous secretions and sweating immediately to the primary health care provider." C) "Take the prescribed medication after meals to increase intestinal absorption." D) "Avoid taking antibiotics, especially neomycin, while on this medication" E) "Maintain the exact same dose of this medication every day."

ANS: A, B, D. Choice A and D are correct due to potential drug-drug interactions with pyridostigmine. Choice B suggests possible cholinergic crisis which can occur if the dose of the medication is too high. The drug should be taken before meals to increase muscle tone needed to chew, swallow, and digest food. The drug dosing may vary depending on how the client is performing each day.

The nurse is caring for a client diagnosed with Guillain Barre syndrome. Which assessment findings require nursing action? (Select all that apply.) A) Blood pressure of 80/42 B) A respiratory rate of 24 C) Shallow breathing pattern D) A peripheral oxygen saturation (Spo2) of 85% E) Diminished breath sounds in all lung fields

ANS: A, C, D, E. All choices except B are abnormal assessment findings that can occur in clients with this disease. A respiratory rate of 24 is slightly elevated but does not require nursing action.

The nurse is teaching a client about what to expect during a cerebral angiographic exam. Which statement by the client indicates a need for further teaching? A) "I can't have this test because I am allergic to shellfish." B) "My head will be strapped in place so that I don't move." C) "I'll have to keep my leg very still after the procedure." D) "I'll have a temporary dressing on my groin."

ANS: A. Being allergic to seafood is no longer a contraindication to receiving iodine contrast materials. Steroids, such as prednisone, will be prescribed for the client in anticipation of the allergy. The other statements are true regarding the care and procedure for this diagnostic test.

The nurse provides health teaching for a client beginning glatiramer acetate therapy. Which statement by the client indicates a need for additional teaching? A) "I'll take this drug with food every morning." B) "I'll look for signs of skin reaction at the injection site." C) "I'll stay away from kids who have colds." D) "I'll avoid large crowds so that I don't get sick."

ANS: A. Glatiramer acetate is given by injection and therefore not an oral drug that would have to be taken with or without food. The client needs to check for signs of local injection skin reaction and report this problem to the primary health care provider. Like all immunomodulating drugs, glatiramer suppresses the immune system and makes the patient more likely to develop an infection. Therefore, the client needs to avoid large crowds of people, some of whom may have an infection, and children who are sick.

The nurse is preparing to administer Sinemet to a client whose most blood pressure is 88/50 while lying in bed. What is the nurse's priority action at this time? A) Instruct the client to get out of bed slowly. B) Withhold the drug until contacting the health care provider. C) Ask the client about the presence of hallucinations. D) Take the patient's apical pulse and temperature.

ANS: B. The client should not get out of bed because the systolic blood pressure will likely drop and the patient may become dizzy or perhaps have syncope (A). While Sinemet can cause hallucinations, it can also cause severe hypotension. Therefore, the priority action is for the nurse to withhold the drug until contacting the primary health care provider at this time, or choice B. Taking the pulse and temperature may be performed, but are not the priority action in response to a very low blood pressure (D).

A nurse is caring for a client who has a hard cervical collar for a complete cervical spinal cord injury. Which assessment finding will the nurse report to the health care provider? A) Purulent drainage from the pin sites on the patient's forehead B) Painful pressure ulcer under the collar C) Inability to move legs or feet D) Oxygen saturation of 95% on room air

ANS: B. The purpose of the cervical hard collar is to immobilize and support the spinal cord and vertebral column. There are no pins in this device. The client with a complete SCI is unable to move his or her legs or feet. An oxygen saturation of 95% is within normal or defined limits.

The nurse is assessing a client who opens both eyes when spoken to, obeys commands, and seems confused during conversation. What Glasgow Coma Score (GCS) will the nurse document? A) 15 B) 14 C) 11 D) 9

ANS: B. To assess this client, the GCS score would be as follows: Eye Opening 4; Motor Response 6; Verbal Response 4 (confused conversation); Total 14

A client who sustained a recent cervical spinal cord injury reports feeling flushed. His blood pressure is 180/100. What is the nurse's best action at this time? A) Perform a bladder assessment. B) Insert an indwelling urinary catheter. C) Turn on a fan to cool off the patient. D) Place the client in a sitting position.

ANS: D. The client is likely experiencing autonomic dysreflexia which is caused by an uncontrolled sympathetic nervous system response to one or more triggers, such as bladder distention, constipation, and temperature variations. However, until the nurse can assess and manage the cause, the best action is to make sure to sit the patient up to begin lowering the blood pressure and prevent further increase.

_____: Abnormal sensitivity to light.

Photophobia

_____: Awareness of body position and movement.

Proprioception (proprioceptive)

_____: A benign tumor of cranial nerve VIII; symptoms include damage to hearing, facial movements, and sensation. The tumor can enlarge into the brain, damaging structures in the cerebellum.

Acoustic neuroma

The nurse is caring for a client with Guillain-Barré syndrome (GBS) who is receiving intravenous immunoglobulin (IVIG). Which assessment finding warrants immediate evaluation? A) Chills. B) Generalized malaise. C) Headache with stiff neck. D) Temperature of 99° F (37° C)

Answer: C. Immediate evaluation is needed when a client with GBS receiving IVIG complains of a headache with stiff neck. This may be a sign of aseptic meningitis, a possible serious complication of IVIG therapy.Chills, generalized malaise, and a low-grade fever are minor adverse effects of IVIG therapy and do not indicate that the therapy must be stopped.

The nurse is developing a teaching plan for a client with a history of low back pain. Which instructions does the nurse plan to include in teaching the client about preventing low back pain and injury? (Select all that apply). A) "Do not wear high-heeled shoes." B) "Keep weight within 50% of ideal body weight." C) "Begin a regular exercise program." D) "When lifting something, the back should be straight and the knees bent." E) "Standing for long periods of time will help to prevent low back pain."

Answer: A, C, D. The nurse includes the following instructions into the low back pain client's teaching plan: don't wear high-heeled shoes, begin a regular exercise program, and keep the back straight and knees bent when lifting something. Wearing high-heeled shoes can increase back strain. Beginning a regular exercise program will help to promote back strengthening. Keeping the back straight while bending the knees is the proper way to lift objects and will help to prevent back injury.The client needs to avoid standing or sitting for long periods of time because this can cause further strain on the back. Weight needs to be kept within 10% of ideal body weight and not 50%.

_____: In the spinal cord, myelinated axons that surround the gray matter (neuron cell bodies).

White matter

_____: Gait disturbance or loss of balance.

Ataxia

_____: The part of the nervous system that is not under conscious control; consists of the sympathetic nervous system and the parasympathetic nervous system.

Autonomic nervous system (ANS)

_____: A mechanism of injury that involves vertical compression. An example is a diving accident, in which the blow to the top of the head causes the vertebrae to shatter and pieces of bone enter the spinal canal and damage the cord.

Axial loading or vertical compression

_____: Acute paralysis of cranial nerve VII; characterized by a drawing sensation and paralysis of all facial muscles on the affected side. The patient cannot close the eye, wrinkle the forehead, smile, whistle, or grimace. The face appears masklike and sags. Also called facial paralysis.

Bell's palsy (facial paralysis)

_____: Stroke; disruption in the normal blood supply to the brain, either as an interruption in blood flow (ischemic stroke) or as bleeding within or around the brain (hemorrhagic stroke). A medical emergency that occurs suddenly, a stroke should be treated immediately to prevent neurologic deficit and permanent disability. Formerly called cerebrovascular accident, the National Stroke Association now uses the term brain attack to describe stroke.

Brain attack

_____: In the patient with untreated increased intracranial pressure, protrusion (herniation) of the brain downward toward the brainstem or laterally from a unilateral lesion within one cerebral hemisphere, causing irreversible brain damage and possibly death.

Brain herniation syndromes

_____: An important speech area of the cerebrum. It is located in the frontal lobe and is composed of neurons responsible for the formation of words, or speech.

Broca's area

_____: Pertaining to the muscles involved in facial expression, chewing, and speech.

Bulbar

_____: An end-tidal carbon dioxide (EtCO2) monitor.

Capnography

_____: Enzymes that inactivate dopamine

Catechol-O-methyltransferases (COMTs)

_____: Visualization of the cerebral circulation (carotid and vertebral arteries) after injecting a contrast medium into an artery (usually the femoral).

Cerebral angiography (arteriography)

_____: Useful in evaluating cerebral vasospasm; can be measured in many areas of the brain with the use of radioactive substances.

Cerebral blood flow (CBF)

_____: Pertaining to a state of alternating muscle stiffness followed by rhythmic jerking motions, as in a tonic-clonic seizure.

Clonic (rhythmic)

_____: Form of hydrocephalus that occurs when the flow of cerebrospinal fluid (CSF) is blocked after it exits the ventricles; this form is "communicating" because CSF can still flow between the ventricles, which remain open.

Communicating hydrocephalus

_____: An injury in which the spinal cord has been severed or damaged in a way that eliminates all innervation below the level of the injury.

Complete spinal cord injry

_____: A fracture that is produced by a loading force applied to the long axis of cancellous bone. These fractures commonly occur in the vertebrae of patients with osteoporosis.

Compression fracture

_____: In assessing pupillary reaction to light, a slight constriction of the pupil of the eye not being tested when a penlight is brought in from the side of the patient's head and shined into the eye being tested as soon as the patient opens his or her eyes.

Consensual response

_____: A classic yet late sign of increased intracranial pressure (ICP) manifested by severe hypertension with a widened pulse pressure and bradycardia. As ICP increases, the pulse becomes thready, irregular, and rapid. Cerebral blood flow increases in response to hypertension.

Cushing's triad

_____: Superficial reflexes. Usually the plantar and abdominal reflexes are tested.

Cutaneous (superficial) reflexes

_____: Abnormal posturing and rigidity characterized by extension of the arms and legs, pronation of the arms, plantar flexion, and opisthotonos; usually associated with dysfunction in the brainstem area. Also called decerebration.

Decerebrate (decorticate) posturing (decerebration)

_____: Abnormal posturing seen in the patient with lesions that interrupt the corticospinal pathways. The arms, wrists, and fingers are flexed with internal rotation and plantar flexion of the legs.

Decorticate posturing (Decortication)

_____: Tested as part of the neurologic assessment. An intact reflex arc is indicated when the muscle contracts in response to the tendon being struck with a reflex hammer.

Deep tendon reflexes

_____: Destruction of myelin between the nodes of Ranvier; a major pathologic finding in multiple sclerosis or Guillain-Barré syndrome.

Demyelination

_____: Groups of nerves that begin in the brain and end in the spinal cord.

Descending tracts

_____: A class of drugs that mimic dopamine. Dopamine agonists stimulate dopamine receptors and are typically the most effective during the first 3 to 5 years of use. Prescribed for the patient with Parkinson disease to reduce dyskinesias (problems with movement).

Dopamine agonist

_____: Period of time lasting up to 10 days in which the patient with Parkinson disease receives no drug therapy.

Drug holiday

_____: Difficulty with movement.

Dyskinesia

_____: Problems understanding written language; caused by brain injury or disease.

Dyslexia

_____: The inability to direct or limit movement.

Dysmetria

_____: Difficulty in swallowing.

Dysphagia

_____: A form of myasthenia gravis that affects the muscles of the trunk and the pelvic and shoulder girdles; often observed in combination with small cell carcinoma of the lung. Although weakness increases after exertion, there may be a temporary increase in muscle strength during the first few contractions, followed by a rapid decline.

Eaton-Lambert syndrome

_____: A recording of the electrical activity of the cerebral hemispheres; it represents the voltage changes in various areas of the brain as determined by recording the difference between two electrodes.

Electroencephalography (EEG)

_____: A recording of the electrical activity of peripheral nerves by testing muscle activity.

Electromyography (EMG)

_____: Damage to the brain when a blood clot forms somewhere in the body (usually the heart) and travels through the bloodstream to block one or more of the arteries supplying the brain.

Embolic stroke

_____: Having uncontrollable emotions; for example, the patient laughs and then cries unexpectedly for no apparent reason.

Emotional lability

_____: A type of aphasia resulting from damage in Broca's area of the frontal lobe of the brain. A motor speech problem in which the patient understands what is said but is unable to communicate verbally and has difficulty writing; rote speech and automatic speech, such as responses to a greeting, are often intact. The patient is aware of the deficit and may become frustrated and angry.

Expressive (Broca's, or motor) aphasia

_____: A surgical procedure in which the surgeon performs a craniotomy and bypasses the blocked artery by making a graft (bypass) from the first artery to the second artery to establish blood flow around the blocked artery and re-establish blood flow to the involved areas.

Extracranial-intracranial bypass

_____: A combination of organic, psychological, and socioeconomic factors in patients for whom back surgery is not successful. Discouraged by repeated surgical procedures, these patients must continue long-term nonsurgical management of pain, including nerve blocks.

Failed back surgery syndrome (FBSS)

_____: Abnormal, involuntary twitching of a muscle.

Fasciculation

_____: A condition of excessive muscle tone, which tends to cause fixed positions or contractures of the involved extremities and restricted range of motion of the joints.

Hypertonia (spastic paralysis)

_____: PaCO2 less than 40 to 45 mm Hg or decreased partial pressure of carbon dioxide in arterial blood.

Hypocarbia

_____: Abnormally decreased sensation.

Hypoesthesia

_____: Paralysis of a part of the body that is characterized by loss of muscle tone due to hypotonia; may be seen in the patient who has experienced a brain attack.

Flaccid paralysis

_____: A structure within the brain; an integral part of autonomic nervous system control (controlling temperature and other functions) that is essential in intellectual function.

Hypothalamus

_____: An abnormal condition of inadequate muscle tone, with an inability to maintain balance.

Hypotonia (flaccid paralysis)

_____: One of the three broad categories of seizure disorders along with partial seizures and unclassified seizures. There are six types: tonic-clonic, tonic, clonic, absence, myoclonic, and atonic (akinetic).

Generalized seizure

_____: An objective and widely accepted tool for neurologic assessment and documentation of level of consciousness. It establishes baseline data for eye opening, motor response, and verbal response. The patient is assessed and assigned a numeric score for each of these areas. A score of 15 represents normal neurologic functioning, and a score of 3 represents a deep coma state.

Glasgow Coma Scale (GCS)

_____: In the spinal cord, neuron cell bodies.

Gray matter

_____: An acute autoimmune disorder characterized by varying degrees of motor weakness and paralysis. It may be referred to by a variety of other names, such as acute idiopathic polyneuritis and polyradiculoneuropathy.

Guillain-Barre syndrome (GBS)

_____: Abnormally increased sensation.

Hyperesthesia

_____: Short-term or new memory. Test by asking the patient to repeat two or three unrelated words to make sure they were heard; after about 5 minutes, while continuing the examination, ask the patient to repeat the words.

Immediate memory

_____: An injury in which the spinal cord has been damaged in a way that allows some function or movement below the level of the injury.

Incomplete spinal cord injury

_____: Necrosis, or cell death.

Infarction

_____: A tremor that occurs when performing an activity.

Intention tremor

_____: Cagelike spinal device that is implanted into the space where a disk was removed. Bone graft tissue grows into and around the cage and creates a stable spine at that level.

Interbody cage fusion

_____: Bleeding within the brain tissue caused by the tearing of small arteries and veins in the subcortical white matter.

Intracerebral hemorrhage (ICH)

_____: Referring to the spine

Intrathecal

_____: Occurring on the same side.

Ipsilateral

_____: Drowsy but easily awakened.

Lethargic

_____: The process of chewing.

Mastication

_____: A general term for the most interior portion of an organ or structure.

Medulla

_____: The immediate protective covering of the brain and the spinal cord.

Meninges

_____: A type of benign brain tumor that arises from the coverings of the brain (the meninges) and causes compression and displacement of adjacent brain tissue.

Meningioma

_____: Inflammation, usually bacterial or viral, of the arachnoid and pia mater of the brain and spinal cord and the cerebrospinal fluid. May be caused by bacteria or viruses; symptoms are the same regardless of the causative organism.

Meningitis

_____: A surgical procedure to relieve the pain of trigeminal neuralgia by relocating a small artery that compresses the trigeminal nerve as it enters the pons. The surgeon carefully lifts the loop of the artery off the nerve and places a small silicone sponge between the vessel and the nerve.

Microvascular decompression

_____: An episodic familial disorder manifested by a unilateral, frontotemporal, throbbing pain that is often worse behind one eye or ear. It is often accompanied by a sensitive scalp, anorexia, photophobia, and nausea with or without vomiting. Three categories of migraine headache are migraines with aura, migraines without aura, and atypical migraines.

Migraine headache

_____: Area in the frontal lobe of the brain that controls voluntary movement.

Motor cortex

_____: A chronic autoimmune disease that affects the myelin sheath and conduction pathway of the central nervous system. It is one of the leading causes of neurologic disability in persons 20 to 40 years of age.

Multiple sclerosis (MS)

_____: Muscle aches/muscle pain.

Myalgia

_____: A chronic autoimmune disease of the neuromuscular junction. It is characterized by remissions and exacerbations, with fatigue and weakness primarily in the muscles innervated by the cranial nerves and in the skeletal and respiratory muscles. It ranges from mild disturbances of the ocular muscles to a rapidly developing, generalized weakness that may lead to death from respiratory failure.

Myasthenia gravis (MG)

_____: Undermedication with cholinesterase inhibitors.

Myasthenic crisis

_____: A white, lipid covering of the axon.

Myelin sheath

_____: Radiography of the spine after injection of contrast medium into the subarachnoid space of the spine; used to visualize the vertebral column, intervertebral disks, spinal nerve roots, and blood vessels.

Myelography

_____: A group of drugs that block excess amounts of glutamate, which damages nerve cells in the brain; used to treat Alzheimer's disease.

N-methyl-D-aspartate (NMDA) receptor antagonist

_____: Degenerating nerve terminals found particularly in the hippocampus, an important part of the limbic system, and marked by increased amounts of an abnormal protein called beta amyloid; a characteristic change of the brain found in patients with Alzheimer's disease.

Neuritic plaques

_____: Tangled masses of fibrous elements throughout the neurons; a classic finding at autopsy in the brains of patients with Alzheimer's disease.

Neurofibrillary tangles

_____: Hypotension and bradycardia associated with cervical spinal injuries and caused by a loss of autonomic function. The patient is at greatest risk in the first 24 hours after injury.

Neurogenic shock

_____: Cells of varying size and shape that provide protection, structure, and nutrition for the neurons.

Neuroglia cells

_____: Excitable nerve cell that processes and transmits information through electrical and chemical signals.

Neuron

_____: Regulatory chemical that exerts inhibitory (slowing down) or excitatory (speeding up) activity at postsynaptic nerve cell membranes. Acetylcholine, norepinephrine, epinephrine, dopamine, and serotonin are neurotransmitters.

Neurotransmitter

_____: Stiff neck, which can be a sign of cerebrospinal fluid leak; nuchal rigidity is not checked until a spinal cord injury has been ruled out.

Nuchal rigidity

_____: Involuntary rapid eye movements.

Nystagmus

_____: In evaluating pupils for size and reaction to light, the midstage between a normal-size pupil and a dilated pupil; indicates the development of increased intracranial pressure.

Ovoid pupil

_____: The separation of plasma from whole blood, after which the blood cells are returned to the patient without the plasma to eliminate antibodies.

Plasmapheresis

_____: A group of clinical manifestations following a concussion that consist of personality changes, irritability, headaches, dizziness, restlessness, nervousness, insomnia, memory loss, and depression. The prolonged pattern is classified as post-trauma syndrome.

Post-concussion syndrome

_____: Referring to the time immediately after a seizure.

Postictal stage

_____: A type of multiple sclerosis (MS) that involves a steady and gradual neurologic deterioration without remission of symptoms. Patients with this type of MS are usually between 40 and 60 years of age at onset of the disease and experience progressive disability with no acute attacks.

Primary progressive MS (PPMS)

_____: Rare disease affecting the white matter of the brain caused by a virus that attacks the cells that make myelin; occurs most often in patients who are immunosuppressed.

Progressive multifocal leukoencephalopathy (PML)

_____: A diagnostic tool using a radiopharmaceutical (agent that enables radioisotopes to cross the blood-brain barrier) that is administered by IV injection, after which the patient is scanned.

Single-photon emission computed tomography (SPECT)

_____: Antipsychotic and neuroleptic drugs. These are appropriately given to patients with emotional and behavioral health problems (e.g., hallucinations and delusions) that accompany dementia but are sometimes inappropriately used for agitation, combativeness, or restlessness. They are considered chemical restraints because they decrease mobility and patients' ability to care for themselves.

Psychotropic drugs

_____: Referring to radicular pain; spinal nerve root involvement.

Radiculopathy

_____: Headache that occurs as a side effect of a drug that has relieved an initial migraine headache. Also called medication overuse headache.

Rebound headache (medication overuse headache)

_____: Special cells throughout the brainstem that constitute the system that controls awareness and alertness.

Reticular activating system (RAS)

_____: Watery drainage from the nose; a "runny" nose.

Rhinorrhea

_____: Changes in peripheral vision.

Scotomas

_____: A type of multiple sclerosis that begins with a relapsing-remitting course and later becomes steadily progressive. Attacks and partial recoveries may continue to occur.

Secondary progressive MS (SPMS)

_____: An abnormal, sudden, excessive, uncontrolled electrical discharge of neurons within the brain that may result in an alteration in consciousness, motor or sensory ability, and/or behavior. A single seizure may occur for no known reason; however, seizures may be due to a pathologic condition of the brain, such as a tumor.

Seizure

_____: Facilitating sensation.

Sensory

_____: A surgical treatment for the patient with Parkinson disease when drugs are ineffective in symptom management. An electrode is used to create a lesion in a targeted area within the pallidum, with the goal of reducing tremor and rigidity.

Stereotactic pallidotomy

_____: An invasive stimulation technique that provides pain control by applying an electrical field over the spinal cord.

Spinal cord stimulation

_____: A surgical procedure to stabilize the spine after repeated laminectomies have been unsuccessful. Chips of bone are removed (typically from the iliac crest) or are obtained from donor bone; the chips are grafted between the vertebrae for support and to strengthen the back.

Spinal fusion (arthrodesis)

_____: Loss of reflex activity below the level of a spinal lesion; occurs immediately after injury as a result of disruption in the communication pathways between the upper motor neurons and the lower motor neurons.

Spinal shock (spinal shock syndrome)

_____: Arousable only with vigorous or painful stimulation.

Stuporous

_____: Term for the space between the arachnoid mater and pia mater of the spinal cord. Also called subarachnoid.

Subarachnoid (subarachnoid space)

_____: The collection of clotted blood that typically results from venous bleeding into the space beneath the dura and above the arachnoid.

Subdural hematoma (SDH)

_____: Term for the space between the dura mater and the middle layer (arachnoid).

Subdural space

_____: In patients with Alzheimer's disease, increased confusion at night or when excessively fatigued.

Sundowning

_____: Located within the cerebral hemispheres, in the area above the tentorium of the cerebellum; the tentlike fold of dura that surrounds the cerebellar hemisphere and supports the occipital lobe.

Supratentorial

_____: The area through which impulses are transmitted to their eventual destination.

Synapse

_____: Transient loss of consciousness (blackouts), most commonly caused by decreased perfusion to the brain.

Syncope

_____: Another term for quadriplegia (paralysis that involves all four extremities).

Tetraplegia

_____: A structure within the brain; functions as the "central switchboard" for the central nervous system.

Thalamus

_____: Damage to the brain when blood flow is impaired from a clot, resulting in blockage to one or more of the arteries supplying blood to the brain.

Thrombotic stroke

_____: Removal of the thymus gland.

Thymectomy

_____: An encapsulated tumor of the thymus gland.

Thymoma

_____: The condition caused by excessive amounts of thyroid hormones.

Thyrotoxicosis

_____: A brief attack (lasting a few minutes to less than 24 hours) of focal neurologic dysfunction caused by a brief interruption in cerebral blood flow, possibly resulting from cerebral vasospasm or transient systemic arterial hypertension. Repeated attacks may damage brain tissue; multiple attacks indicate significant increased risk for brain attack.

Transient ischemic attack (TIA)

_____: One of the three broad categories of seizure disorders along with partial seizure and generalized seizure. They occur for no known reason, do not fit into the generalized or partial classifications, and account for about half of all seizure activity.

Unclassified (idiopathic) seizure

_____: In the patient who has had a brain attack, an unawareness of the existence of the paralyzed side. For example, the patient may believe he or she is sitting up straight when actually he or she is leaning to one side. Another typical example is the patient who washes or dresses only one side of the body.

Unilateral inattention (body neglect) syndrome

_____: The abnormal accumulation of cerebrospinal fluid within the skull.

hydrocephalus


Related study sets

Marketing Chapters 3-8 Practice Questions

View Set

Mood Disorders: Bipolar/Depression

View Set

Cognitive Psychology Chapter 1 Quiz

View Set

Chapter 11 Designing and Implementing Brand Architecture Strategies

View Set

active process and Mechanoelectrical Stimulation

View Set

Chapter 5 IT infrastructure full notes

View Set